Download as pdf or txt
Download as pdf or txt
You are on page 1of 260

TABLE OF CONTENTS

Part iv: the derivative……………………………………………………03


problem set 1…………………………………………………………………07
Problem set 2……………………………………………………………..….10
Part v: the stope:………………………………………………..……..…. 16
problem set 3………………………………………………………………… 21
Problem set 4………………………………………………………………… 25
Part vi: rate of change……………………………………………….… 28
Problem set 5………………………………………………………………… 33
problem set 6…………………………………………………………,……… 39
Part vii: the chain rule and general power rule............. 44
problem set 7………………………………………………………,………... 53
problem set 8……………………………………………………,…………… 61
Part viii: implicit differentiation………………..……,…………… 66
problem set 9…………………………………..…………………,……….… 75
problem set 10……………………………………………………….…….… 79
Part IX: Higher-Order Derivatives ………………….…,,………… 82
problem set 11…………………………………………………….…,……… 87
problem set 12………………………………………………………,….…… 92
Part x: polynomial curves………………………….…………,………134
problem set 13…………………………………………………………,...… 142
problem set 14……………………………………………………………..…03
Part xi: optimization problems………………….…………………..150
problem set 15……………………………………………………..………..159
problem set 16……………………………………………………………..…166
Part xii: related rates ……………………………………………..……173
problem set 17…………………………………………………………..……179
problem set 18………………………………………………………..………188
Part xiii: the differential…………………………………….…………203
problem set 19……………………………………………………………..…215
problem set 20………………………………………………………..……...221
problem set 21…………………………………………………………..……227
ANSWER KEY……………………………………………………………...…..234
references………………………………………………………………...…..260

2
PART iv: The
DERIVATIVE
The derivative of a function is one of the basic concepts of mathematics. Together with the
integral, derivative occupies a central place in calculus. The process of finding the derivative is
called differentiation. The inverse operation for differentiation is called integration.

The derivative of a function at some point characterizes the rate of change of the function at this
point. We can estimate the rate of change by calculating the ratio of change of the function 𝛥𝑦 to
the change of the independent variable 𝛥𝑥. In the definition of derivative, this ratio is considered
in the limit as 𝛥𝑥 → 0. Let us turn to a more rigorous formulation.

What is the formal definition of derivative?


Let f(x) be a function whose domain contains an open interval about some point 𝑥0 . Then the
function 𝑓(𝑥) is said to be differentiable at 𝑥0 , and the derivative of 𝑓(𝑥) at 𝑥 0 is given by

𝛥𝑦 𝑓(𝑥0 + 𝛥𝑥) − 𝑓(𝑥0 )


𝑓 ′(𝑥0 ) = 𝑙𝑖𝑚 = 𝑙𝑖𝑚 .
∆𝑥→0 𝛥𝑥 ∆𝑥→0 𝛥𝑥

Lagrange’s notation is to write the derivative of the function


𝑦 = 𝑓(𝑥) 𝑎𝑠 𝑓′(𝑥) 𝑜𝑟 𝑦′(𝑥).

𝑑𝑓 𝑑𝑦
Leibniz’s notation is also to write the derivative of the function 𝑦 = 𝑓(𝑥) 𝑎𝑠 𝑜𝑟 .
𝑑𝑥 𝑑𝑥

The steps to find the derivative of a function f(x) at the point x0 are as follows:
Δ𝑦 f(𝑥 +Δx)−f(𝑥 )
• Form the difference quotient Δ𝑥 = 0 𝛥𝑥 0 ;
• Simplify the quotient, cancel 𝛥𝑥 if possible;
• Find the derivative 𝑓′(𝑥0 ), applying the limit to the quotient. If this limit exists, then we
say that the function 𝑓(𝑥) is differentiable at 𝑥0 .

In the examples below, we derive the derivatives of the basic elementary functions using the
formal definition of derivative. These functions comprise the backbone in the sense that the
derivatives of other functions can be derived from them using the basic differentiation rules.

3
Examples:

1. Using the definition of derivative, prove that the derivative of a constant is 0.


Solution.
In this case, the function y(x) is always equal to to a constant C. Therefore, we can write
y(x) = C , y(x + Δx) = C.

It is clear that the increment of the function is identically equal to zero:


Δy = y(x + Δx) − y(x) = C – C ≡ 0.

Substituting this in the limit definition of derivative, we obtain:

Δy y(x + Δx) − y(x) 0


y′(x) = lim = lim = lim = lim 0 = 0 .
∆x→0 Δx ∆x→0 Δx ∆x→0 Δx ∆x→0

2.
Calculate the derivative of the function y = x.

Solution:
Δy
Following the above procedure, we form the ratio Δx and find the limit as Δx → 0:

Δy (x + Δx) − x Δx + Δx − x Δx
y′ (x) = lim = lim = lim = lim = lim 1 = 1
x→0 Δx Δx→0 Δx x→0 Δx Δx→0 Δx Δx→0

3.
Find the derivative of a linear function y = ax + b using the definition of derivative.

Solution.
We write the increment of the function corresponding to a small change in the argument Δx:
Δy = y(x + Δx) − y(x) = (a(x + Δx) + b) − (ax + b) = ax + aΔx + b − ax − b = aΔx.

Then the derivative is given by

Δy aΔx
y′(x) = lim = lim = lim a = a.
Δx→0 Δx Δx→0 Δx Δx→0

As it can be seen, the derivative of a linear function y = ax + b is always constant and equal to
the coefficient a.

4
4.

Using the limit definition find the derivative of the function f(x) = 3x + 2.

Solution.

Write the increment of the function:

Δy = y(x + Δx) − y(x) = [3(x + Δx) + 2] − [3x + 2] = 3x + 3Δx + 2 − 3x − 2 = 3Δx.

The difference ratio is equal to ΔyΔx = 3ΔxΔx = 3.


Δy
Then the derivative is given by f′(x) = lim = lim 3 = 3.
Δx→0 Δx Δx→0

What are the differentiation rules?


dy
Remember that if y = f(x) is a function then the derivative of y can be represented by dx or y′
df
or f’ or dx.
Rule 1: The Derivative of a Constant.
The derivative of a constant is zero.

Rule 2: The General Power Rule.


The derivative of x n is nx n−1

Rule 3: The Derivative of a Constant times a Function.


The derivative of kf(x), where k is a constant, is kf′ (x)

Rule 4: The Derivative of a Sum or a Difference.


df dh dg
If f(x) = h(x) ± g(x), then dx = dx ± dx.

Rule 5: The Product Rule.


The derivative of the product y = u(x)v(x), where u and v are both functions of x is
dy dv du
= u × + v ×
dx dx dx

Rule 6: The Quotient Rule.


u(x)
The derivative of the quotient (x) = , where u and v are both function of x is
v(x)
du du
df v× −u×
= dx dx
dx v 2

Rule 7: The Chain Rule.


If y is a function of u, i.e. y = f(u), and u is a function of x, i.e. u = g(x) then the derivative of
dy dy du
y with respect to x is dx = du × dx

5
Examples:
1
1.y = 14(x − 1)[2 + x 2 ]
SOLUTION:
1 1
y = 14 ( X3 − X2 + x − )
2 2
dy 1
∴ = 14 (3x 2 − 2x + )
dx 2
= 42x 2 − 28x + 7
x2 −5x+6
2. Find f′(x) if f(x) = .
x−2
x 2 − 5x + 6
f(x) =
x−2
(x − 3)(x − 2)
=
x−2
=x−3
f ′ (x) = 1
3. Find f′(y) if f(y) = √y.
f(y) = √y
1
= y2
1 1
f ′ (y) = y −2
2
1
=
2√y
4. Find f′(z) if f(z) = (z − 1)(z + 1).
f(z) = (z − 1)(z + 1)
= z2 − 1
f′(z) = 2z

6
Problem Set 1: Conversion
1. What is the temperature in degree Celsius of absolute zero?
A. -32 C. 273
B. 0 D. -273
2. How many degree Celsius is 100 degrees Fahrenheit?
A. 37.8°C C. 1.334°C
B. 2.667°C D. 13.34°C
3. A comfortable room temperature is 72°F. What is this temperature expressed In degrees
Kelvin?
A. 263 C. 295
B. 290 D. 275
4. 255°C is equivalent to:
A. 491°F C. 173.67°F
B. 427°F D. 109.67°F
5. At what temperature will be the °C and °F readings be equal?
A. 40° C.32°
B.-40° D.0°
6. How many degree Celsius is 80 degrees Fahrenheit
A. 26.67 C. 33.33
B. 86.4 D. 16.33
7. What is the absolute temperature of the freezing point of water in degree Rankine?
A. -32 C. 428
B. 0 D. 492
8. The angle of inclination of the road is 32°. What is the angle of inclination in mile?
A. 456.23 C. 125.36
B. 568.89 D. 284.44
9. An angle measures x degrees. What is its measure in radians?
A. 180°x/π C. 180° π/x
B. πx/180° D. 180° πx
10. Express 45° in mils.
A. 80 mils C.8000 mils
B.800 mils D.80000 mils
11. What is the value in degrees of π radians?
A. 90° C.180°
B.57.3° D.45°
12. How many degrees is 3200 mils
A. 360° C.180°
B.270° D.90°
13. An angular unit equivalent to 1/400 of the circumference of a circle is called
A. mil C. radian
B. grade D. degree

7
14. Carry out the following multiplication and express your answer in cubic meters: 3cm x 5mm
x 2m
A. 3 x 10-3 C. 8 x 10-2
-4
B. 3 x 10 D. 8 x 102
15. Add the following and express in meters: 3m + 2cm + 70 mm
A. 2.90m C.3.12m
B. 3.14m D.3.09m
16. One nautical mile is equivalent to:
A. 5280 ft. C. 1.256 statute mile
B. 6280 ft. C. 1.854
17. How many square feet is 100 square meters?
A. 328.10 C.1075.84
B.956.36 D.1563.25
18. A tank contains 1500 gallons of water. What is the equivalent in cubic meters?
A. 4.256 C.6
B.5.865 C.5.685
19. How many cubit feet is equivalent to 100 gallons of water?
A. 74.80 C.13.37
B.1.337 D.133.7
20. How many cubic meters is 100 gallons of liquid?
A. 0.1638cu.meter C.0.3785cu.meter
B.1.638cu.meter D.3.7850 cu.meter
21. The number of board feet in a plank 3 inches thick, 1 foot wide and 20 feet long is:
A. 30 C. 120
B. 60 D. 90
22. Which of the following is correct?
A. 1 horsepower = 746 kW C. 1 horsepower 0.746 kW
B. 1 horsepower = 0.746 watts D. 1 horsepower = 546 watts
23. The acceleration due to gravity in English unit is equivalent to:
A. 32.2ft/sec2 C. 9.81 ft/sec2
B. 3.22ft/sec2 D. 98.1 ft/sec2
24. The prefix nano is opposite to:
A. mega C. hexa
B. tera D. giga
th
25. 10 to the 12 power is the value of the prefix:
A. giga C. tera
B. pico D. peta
26. Convert 630 degrees to its value in centesimal system
A. 700 grade C.720 grade
B. 800 grade D. 850 grade
27. When rounded-off to four significant figures, 102.48886 becomes:
A. 102.4 C. 102.4888
B. 102.4889 D. 102.5

8
28. Which of the following is a prime number?
A. 221 C.523
B. 63 D. 703
29. The number of significant figures in the value 500.00 is:
A. one C. five
B. three D. six
30. A line on a map was drawn at a scale of 5:100,100. If a line in the map is 290mm long, the
actual length of the line is:
A. 4.8 C. 2.9km
B. 5.8km D. 6.4km
31. Which of the following is correct:
A. 0.001 has three significant figures
B. 100 has three significant figures
C. 100.00 has five significant figures
D. 0.000249 has six significant figures
32. The scale on the map is 1:x. A lot having an area of 640 sq. m. is represented by an area of
25.6cm2 on the map. What is the value of x?
A. 500 C. 50
B. 1000 D. 100

9
Problem Set 2: Exponents and Radicals

1. Solve for x:x = -(1/-27)-2/3


A. 9 C. -9
B. 1/9 D. -1/9
2. Solve for a in the equation: a= 64x4y
A. 4x+3y C. 5 x 3x-2
B. 43xy D. 13 x 3x
3. Simplify 3
4. Which of the following is true?
A. √−2 𝑥 √−2 = 2 C. √10 − √5 + √2
B. 24 = 4 √6 D. 55 + 55 + 55 + 55 + 55 = 56
5. Solve for x: x = √18 − √72 + √50
A. -2√2 C. 4
B. 2√2 D. 4√3
6. Solve for x: √𝑥 − √1 − 𝑥 = 1 − √𝑥
A. -16/25 & 0 C. -25/15 & 0
B. 25/16 & 0 D. 16/25 & 0
3 3 3
7. Simplify √𝑥 − √16𝑥 + 2 √54𝑥 4
4 4
3 3
A. 5√𝑥 4 C. 5 √2𝑥 4
3 3
B. 2 √5𝑥 4 D. 2√3𝑥 4
8. Solve for x: 3𝑥 5𝑥+1 = 6𝑥+2
A. 2.1455 C. 2.4154
B. 2.1445 D. 2.1544
2
(𝑎−2 𝑏3 )
9. Simplify
𝑎2 𝑏−1
−2 7
A. 𝑎 𝑏 C. 𝑎−6 𝑏7
B. 𝑎−2 𝑏5 D. 𝑎−6 𝑏5
10. (3𝑥 )𝑥 is equal to:
A. 3𝑥 2 C. 3𝑥 2
B. 3𝑥 𝑥 𝑥 B. 32𝑥
11. Solve for x: 3(7𝑥+1) = 6561
A. 1 C. 3
B. 2 D. 4
3𝑎2
12. If 3𝑎 = 7𝑏, then 7𝑏2 =
A. 1 C. 7/3
B. 3/7 D. 49/9

13. Solve for U if U = √1 − √1 − √1 − ⋯

10
A. 0.723 C. 0.852
B. 0.618 D. 0.453
14. If x to the ¾ power equals 8, then x equals:
A. -9 C. 9
b. 6 D. 16
15. If 33y = 1, what is the value of y/33?
A. 0 C. infinity
B. 1 D. 1/33
16. Find the value of x that will satisfy the following expression:
√𝑥 − 2 = −√𝑥 + 2
A. x = 3/2 C. x = 9/4
B. x= 18/6 D. none of these
−3
17. 𝑒 is equal to:
A. 0.048787 C. 0.049787
B. 0.049001 D. 0.048902
𝑚
18. B to the 𝑛𝑡ℎpower is equal to:
A. 𝑛𝑡ℎ root of b to the m power
B. 𝑏 to the 𝑚 + 𝑛 power
1
C. square root of 𝑏 to the 𝑚 power
𝑛
D. 𝑏 to the 𝑚 power over 𝑛
19. Find x from the follow equations:
27𝑥 = 9𝑦
81𝑦 3−𝑥 = 243
A. 2.5 C. 1
B. 2 D. 1.5
20. Solve for 𝑎 from the following equations:
𝑎𝑛
(𝑎𝑚 )(𝑎𝑛 ) = 100,000 𝑎𝑚𝑛 = 100,000 = 10
𝑎𝑚
A. 15.85 C.12
B. 10 D. 12.56
1
−3 (− )
(𝑥 2 𝑦 3 𝑧−2 ) (𝑥 −3 𝑦𝑧 3 ) 2
21. Simplify ( 5
(𝑥𝑦𝑧 −3 )−2
1 1
A. 𝑥2 𝑦7 𝑧 3 C. 𝑥2 𝑦5 𝑧3
1 1
B. 𝑥2 𝑦7 𝑧5 D. 𝑥5 𝑦 7𝑧 3
22. Simplify the following: 7𝑎+2 − 8(7𝑎+1 ) + 5(7𝑎 ) + 49(7𝑎−2 )
A. −5𝑎 C.−7𝑎
B. 3 D. 7𝑎
4 3
𝑥𝑦 −1 𝑥 2 𝑦 −2
23. Simplify: ( 3) + ( 3)
𝑥 −2 𝑦 𝑥 −3 𝑦
3
A. 𝑥𝑦 C. 𝑥 3 𝑦

11
𝑦 1
B. 𝑥3 D. 𝑥3 𝑦
√5−√3
24. Simplify the following:
√5+ √3
A. 4 + √15 C. 8 +√8
B. 4 – √15 D. 8 – √8
𝑚
25. Which of the following is equivalent to √ 𝑛√𝑎
𝑚
A. √𝑎𝑛 C. √𝑎𝑚𝑛
𝑛
B. √𝑎𝑚 D. 𝑚𝑛√𝑎
26. Find the value of x in (35 )(96 ) = 32𝑥
A. 8.5 C. 9.5
B. 9 D. 8
27. Solve for x:
𝑥
𝑥 𝑥 𝑥 = 10
A. 1.2589 C. 1.1745
B. 2.4156 D. Cannot be solved

12
-

PART v: THE
SLOPE
The slope describes the steepness of a line as a relationship between the change in y-
𝑦 −𝑦
values for a change in the x-values. Slope = 𝑥2 − 𝑥1
2 1
- What is the definition of Slope as the derivative of a function?
A derivative of a function is a representation of the rate of change of one variable in relation to
another at a given point on a function.

The derivative measures the steepness of the graph of a function at some particular point on the
graph. Thus, the derivative is a slope. (That means that it is a ratio of change in the value of the
function to change in the independent variable.)

If the independent variable happens to be "time", we often think of this ratio as a rate of change
(an example is velocity).

How to determine the slope of a curve at a given point?


To find the slope mm of a curve at a particular point, we differentiate the equation of the
𝑑𝑦
curve. If the given curve is 𝑦 = 𝑓 (𝑥 ), we evaluate 𝑑𝑥 or 𝑓′(𝑥 ) and substitute the value of
𝑥 to find the slope.
𝑦 = 𝑥4 − 𝑥3 𝑥 =1
𝑦 = 𝑥4 − 𝑥3
𝑑 𝑑
𝑦 = 𝑑𝑥 ( 𝑥 4 − 𝑥 3 )
𝑑𝑥
𝑑𝑦
= 4𝑥 3 − 3𝑥 2
𝑑𝑥
𝑑𝑦
= 4(1) − 3(1) x=1
𝑑𝑥
=4−3= 1
Therefore, the slope of the given curve at 𝑥 = 1 is 1.

13
Examples:
1. Find the slope of the curve 𝑦 = 2𝑥 3 − 8𝑥 2 + 1 at the point (2, −15).
𝑦 = 2𝑥 3 − 8𝑥 2 + 1
𝑑𝑦
= 6𝑥 2 − 16𝑥
𝑑𝑥
𝑑𝑦
= 6(2)2 – 16(2) = 24 − 32 = −8
𝑑𝑥

2. If the curve 𝑦 = 2𝑥 3 − 𝑏𝑥 + 𝑎 passes through (19, 2) and its slope at 𝑥 = 1 is 5, then what
are the values of 𝑎 and 𝑏?
𝑦 = 2𝑥 3 − 𝑏𝑥 + 𝑎 , 𝑥=1
𝑑𝑦
= 6𝑥 2 – 𝑏
𝑑𝑥

= 6(1)2 − 𝑏 = 6 − 𝑏 = 5 , 𝑏 = 1
𝑦 = 2𝑥 3 − 𝑏𝑥 + 𝑎 , (19, 2)
19 = 2(2)3 − 2 + 𝑎
19 = 16 −2 + 𝑎
19 = 14 + 𝑎 ⇾ 𝑎 = 5
Therefore 𝑎 = 5 and 𝑏 = 1
3. Find the slope of a line going through the point (1, 2) and the point (4, 3)
𝑦 −𝑦
Slope = 𝑥2 − 𝑥1
2 1
3−2 1 1
Slope = =3 Therefore, the slope is
4−1 3

4. Find the slope of a line going through the point (2, 1) and the point (3, 4)
𝑦 −𝑦
Slope = 𝑥2 − 𝑥1
2 1
2−3 −1 1 1
Slope = = −3 = 3 Therefore, the slope is
1−4 3

14
Exercises:
1. Find the slope of the curve 𝑦 = 7𝑥 9 + 8𝑥 at the point(6, −2).
2. Find the slope of the curve 𝑓(𝑥 ) = 𝑥 3 − 𝑥 + 2 at 𝑥 = 1.
3. Find the slope at points (4, -1) and (4, 4)
4. Find the slope at points (-1, 4) and (3, 2)

Solutions:
1. 𝑦 = 7𝑥 3 + 8𝑥 at the point(6, −2).
𝑦 = 7𝑥 3 + 8𝑥
𝑑𝑦
= 21𝑥 2 + 8
𝑑𝑥
𝑑𝑦
= 21(6)2 + 8 = 260
𝑑𝑥

2. 𝑓 (𝑥 ) = 𝑥 3 − 𝑥 + 2 at 𝑥 = 1
𝑓 (𝑥 ) = 𝑥 3 − 𝑥 + 2
𝑓 ′(𝑥) = 3𝑥 2 − 1
= 3(1)2 − 1 = 3 − 1 = 2

3. points (4, -1) and (4, 4) , slope = ?


𝑦2 − 𝑦1
Slope =
𝑥2 − 𝑥1

4−(−1) 5
Slope = = = 𝑈𝑁𝐷𝐸𝐹𝐼𝑁𝐸𝐷 (the slope is undefined because the line is parallel to
4−4 0
the y – axis)

4. points (-1, 4) and (3, 2), slope is


𝑦2 − 𝑦1
Slope = 𝑥2 − 𝑥1

2 − (4) −2 1
Slope = = = −2
3 –(−1) 4

15
Problem 3: Fundamentals in Algebra

1. Change 0.2272727…. to a common fraction.


A. 7/44 C. 5/22
B. 5/48 D.9/34
2. What is the value of 7! Or 7 factorial?
A. 5040 C. 5020
B. 2540 D. 2520
3. The reciprocal of 20 is:
A. 0.50 C. 0.20
B. 20 D. 0.05
4. If 𝑝 is an odd number and 𝑞 is an even number, which of the following expression must be
even?
A. 𝑝 + 𝑞 C. 𝑝𝑞
B. 𝑝 − 𝑞 D. p/q
5. MCMXCIV is a Roman Numeral equivalent to:
A. 2974 C. 2174
B. 3974 D. 1994
6. What is the lowest common factor of 10 & 32?
A. 320 C. 180
B. 2 D. 90
7. 4𝑥𝑦 − 4𝑥 2 − 𝑦 2 is equal to:
A. (2𝑥 − 𝑦)2 C. (−2𝑥 + 𝑦)2
B. (−2𝑥 − 𝑦)2 D.-(2𝑥 − 𝑦)2
8. Factor 𝑥 4 − 𝑦 2 + 𝑦 − 𝑥 2 as completely as possible.
A. (𝑥 2 + 𝑦)(𝑥 2 + 𝑦 − 1) C. (𝑥 2 − 𝑦)(𝑥 2 − 𝑦 + 1)
2 2
B. (𝑥 + 𝑦)(𝑥 − 𝑦 − 1) D. (𝑥 2 − 𝑦)(𝑥 2 + 𝑦 − 1)
9. Factor the expression 𝑥 2 + 6𝑥 + 8 as completely as possible.
A. (𝑥 + 8)(𝑥 − 2) C. (𝑥 + 4)(𝑥 − 2)
B. (𝑥 + 4)(𝑥 + 2) D. (𝑥 − 8)(𝑥 − 2)
3
10. Factor the expression 𝑥 + 8 as completely as possible:
A. (𝑥 − 2)(𝑥 2 + 2𝑥 + 4) C. (−𝑥 + 2)(−𝑥 2 + 2𝑥 + 2)
B. (𝑥 + 4)(𝑥 2 + 2𝑥 + 2) D. (𝑥 + 2)(𝑥 2 − 2𝑥 + 4)
11. Factor the expression (𝑥 4 − 𝑦 4 ) as completely as possible:
A. (𝑥 + 𝑦)(𝑥 2 + 2𝑥𝑦 + 𝑦 2 ) C. (𝑥 2 + 𝑦 2 )(𝑥 + 𝑦)(𝑥 − 𝑦)
B. (𝑥 2 + 𝑦 2 )(𝑥 2 − 𝑦 2 ) D. (1 + 𝑥 2 )(1 + 𝑦)(1 − 𝑦 2 )
12. Factor the expression 3𝑥 3 + 3𝑥 2 − 18𝑥 as completely as possible:
A. 3𝑥 (𝑥 + 2)(𝑥 − 3) C. 3𝑥 (𝑥 − 3)(𝑥 + 6)
B. 3𝑥 (𝑥 − 2)(𝑥 + 3) D. (3𝑥 2 − 6𝑥 )(𝑥 − 1)
13. Factor the expression 16 − 10𝑥 + 𝑥 2
A. (𝑥 + 8)(𝑥 − 2) C.(𝑥 − 8)(𝑥 + 2)

16
B. (𝑥 − 8)(𝑥 − 2) D. (𝑥 + 8)(𝑥 + 2)
6
14. Factor the expression 𝑥 − 1 as completely as possible:
A. (𝑥 + 1)(𝑥 − 1)(𝑥 4 + 𝑥 2 − 1)
B. (𝑥 + 1)(𝑥 − 1)(𝑥 4 + 2𝑥 2 + 1)
C. (𝑥 + 1)(𝑥 − 1)(𝑥 4 − 𝑥 2 + 1)
D. (𝑥 + 1)(𝑥 − 1)(𝑥 4 + 𝑥 2 + 1)
15. The roots of the equation (𝑥 − 4)2 (𝑥 + 2) = (𝑥 + 2)2 (𝑥 − 4)
A. 4 and -2 only C. -2 and 4 only
B. 1 only D. 1,-2, and 4 only
2
16. If 𝑓 𝑥 = 𝑥 + 𝑥 + 1, then 𝑓 𝑥 − 𝑓 (𝑥 − 1) =
( ) ( )
A. 0 C. 2x
B. x D. 3
17. Which of the following is not an identity?
A. (𝑥 − 1)2 = 𝑥 2 − 2𝑥 + 1
B. (𝑥 + 3)(2𝑥 − 2) = 2(𝑥 2 + 2𝑥 − 3)
C. 𝑥 2 − (𝑥 − 1)2 = 2𝑥 − 1
D. 2(𝑥 − 1) + 3(𝑥 + 1) = 5𝑥 + 4
𝑥+3 4𝑥 2 𝑥+9
18. Solve for x: 4 + 𝑥−3 − 𝑥2 −9 = 𝑥+3
A. -18 = -18 C. Any value
B. 12=12 or -3 = -3 D. -27 = -27 or 0 = 0
19. Solve the simultaneous equations: 3𝑥 − 𝑦 = 6; 9𝑥 − 𝑦 = 12
A. 𝑥 = 3; 𝑦 = 1 C. 𝑥 = 2; 𝑦 = 2
B. 𝑥 = 1; 𝑦 = −3 D. 𝑥 = 4; 𝑦 = 2
20. Solve algebraically: 4𝑥 + 7𝑦 2 = 32; 11𝑦 2 − 3𝑥 2 = 41
2

A. 𝑦 = 4, 𝑥 = ±1, 𝑎𝑛𝑑 𝑦 = −4, 𝑥 = ±1


B. 𝑦 = +2, 𝑥 = ±1, 𝑎𝑛𝑑 𝑦 = −2, 𝑥 = ±1
C. 𝑥 = 2, 𝑦 = 3 𝑎𝑛𝑑 𝑥 = −2, 𝑦 = −3
D. 𝑥 = 2, 𝑦 = −2, 𝑎𝑛𝑑 𝑥 = 2, 𝑦 = −2
21. Solve for w from the following equations;
3𝑥 − 2𝑦 + 𝑤 = 11 𝑥 + 5𝑦 − 2𝑤 = −9
2𝑥 + 𝑦 − 3𝑤 = −6
A. 1 C. 3
B. 2 D. 4
22. When (𝑥 + 3)(𝑥 − 4) + 4 is divided by x-k, the remainder is k. Find the value of k
A. 4 or 2 C. 4 or -2
B. 2 or -4 D. -4 or -2
2
23. Find k in the equation 4𝑥 + 𝑘𝑥 + 1 = 0 so that it will only have one real root.
A. 1 C. 3
B. 2 D. 4
12
24. Find the remain when (𝑥 + 2) is divided by (𝑥 − √3)
A. 652 C. 231
B. 731 D. 851

17
25. If 3𝑥 2 − 4𝑥 2 𝑦 + 5𝑥𝑦 2 + 6𝑦 3 is divided by (𝑥 2 02𝑥𝑦 + 3𝑦 2 ) the remainder is:
A. 0 C. 2
B. 1 D. 3
3 2
26. If 4𝑦 + 8𝑦 + 18𝑦 − 4 is divided by (2𝑦 + 3), the remainder is
( )
A. 10 C.12
B. 11 D. 13
27. Given: 𝑓 (𝑥 ) = (𝑥 + 3)(𝑥 − 4) + 4 when divided by (𝑥 − 𝑘 ), the remainder is k. Find k
A. 2 C. 4
B. 3 D. -3
28. The polynomial 𝑥 3 + 4𝑥 2 − 3𝑥 + 8 is divided by 𝑥 − 5. What is the remainder?
A. 281 C. 218
B. 812 D. 182
29. Find the quotient of 3𝑥 − 4𝑥 + 2𝑥 2 + 36𝑥 + 48 divided by 𝑥 3 − 2𝑥 2 + 6
5 3

A. −3𝑥 2 − 4𝑥 + 8 C. 3𝑥 2 − 4𝑥 − 8
B. 3𝑥 2 + 4𝑥 + 8 D. 3𝑥 2 + 6𝑥 + 8
1 1
30. If 𝑥 = 𝑎 + 𝑏 and 𝑦 = 𝑎 − 𝑏 then 𝑥 − 𝑦 is equal to:
A. ½𝑎 C. 2𝑎 / (𝑎2 − 𝑏2 )
B. ½𝑏 D. 2𝑏 (𝑎 2 − 𝑏 2 )
1
31. If x-1/x=1, find the value of 𝑥 3 − 𝑥3
A. 1 C. 3
B. 2 D. 4
1 1 2 1
32. If 𝑥 + 𝑦 = 3 and 𝑥 − 𝑦 = 1, then x is equal to:
1 3
A. 2 C. 4
2 4
B. D.
3 3
33. Simplify the following expression:
5𝑥 𝑥+3 2𝑥 + 1
2
− 2 + 2
2𝑥 + 7𝑥 + 3 2𝑥 − 3𝑥 − 2 𝑥 + 𝑥 − 6
A. 2 / (x-3) C. (x +3)/(x-1)
B. (x-3)/5 D. 4 / (x+3)
3𝑥 2
34. If 3𝑥 = 4𝑦 then 4𝑦2 is equal to:
A. 3/4 C. 2/3
B. 4/3 D. 3/2
1 2 1 2
35. Simplify: (𝑎 + 𝑎) − (𝑎 − 𝑎)
A. -4 C. 4
2
B. 0 D. − 𝑎2
36. The quotient of (𝑥 5 + 32) by (𝑥 + 2) is:
A. 𝑥 4 − 𝑥 2 + 8 C. 𝑥 4 − 2𝑥 3 + 4𝑥 2 − 8𝑥 + 16
3 3
B. 𝑥 + 2𝑥 − 8𝑥 + 4 D. 𝑥 4 + 2𝑥 3 + 𝑥 2 + 16𝑥 + 8

18
37.Solve the simultaneous equations:
𝑦 − 3𝑥 + 4 = 0
𝑥 2 24
𝑦+ =
𝑦 𝑦
−6+2√14 −6−2√14 6+2√14 6−2√14
A. 𝑥 = or C. 𝑥 = or
5 5 5 5
2+6√14 −2+6√14 −2+6√14 −2−6√14
y = 5 𝑜𝑟 y= 𝑜𝑟
5 5 5
6+2√15 6−2√15 6+2√14 6−2√14
B. x = 𝑜𝑟 D. x = 𝑜𝑟
5 5 5 5
−2+6√14 −2−6√15 −6−2√14 −6+2√14
y= 𝑜𝑟 y= 𝑜𝑟
5 5 5 5
38. Find the value of A in the equation
𝑥 2 +4𝑥+10 𝐴 𝐵(2𝑥+2) 𝐶
= 𝑥 + 𝑥2 +2𝑥+5 + 𝑥2 +2𝑥+5
𝑥 3 +22 +5𝑥
1
A. 2 C. −
2
1
B. -2 D. 2
𝑥+10 𝐴 𝐵
39. Find A and B such that = +
𝑥 2 −4 𝑥−2 𝑥+2
A. A= -3; B = 2 C. A = 3; B = -2
B. A = -3; B = -2 D. A = 3; B =2
(𝑥+2)
40. Resolve 𝑥2 −7𝑥+12 into partial fraction.
6 2 6 5
A. 𝑥−4 − 𝑥−3 C. 𝑥−4 − 𝑥−3
6 7 6 5
B. 𝑥−4 + 𝑥−3 D. 𝑋−4 + 𝑥−3
41. The arithmetic mean of 80 numbers is 55. If two numbers namely 250 and 850 are removed,
what is the arithmetic mean of the remaining numbers?
A. 42.31 C. 50
B. 57.12 D. 38.62
42. The arithmetic mean of 6 numbers is 17. If two numbers are added to the progression, the
new set of number will have an arithmetic mean of 19. What are the two numbers if their
difference is 4?
A. 21,29 C. 24,26
D. 23,27 D. 22,28
43. If 2𝑥 − 3𝑦 = 𝑥 + 𝑦, then 𝑥 : 𝑦 2 =
2

A. 1:4 C. 1:16
B. 4:1 D. 16:1
44. If 1/c : 1/b : 1/c =2 : 3 : 4, then (𝑎 + 𝑏 + 𝑐 ): (𝑏 + 𝑐) is equal to
A. 13:7 C. 10:3
B. 15:6 D. 7:9
45. Find the mean proportional to 5 and 20
A. 8 C. 12
B. 10 D. 14
46. Find the fourth proportional of 7,12, and 21

19
A. 36 C. 32
B. 34 D. 40
47. If (x + 3): 10 = (3x – 2):8, find (2x – 1)
A. 1 C. 3
B. 2 D.4
48. Solve for x: -4 < 3x-1 < 11
A. 1 <x<-4 C. 1<x<4
B. -1<x<4 D. -1<x<-4
49. Solve for x: 𝑥 2 + 4𝑥 > 12
A. -6 > x > 2 C. -6 > x > -2
B. 6 > x > -2 D. 6 > x > 2
50. When the expression 𝑥 4 + 𝑎𝑥 3 + 5𝑥 2 + 𝑏𝑥 + 𝑏 is divded by (x – 2), the remainder is 16.
When it is divided by (x + 1) the remainder is 10. What is the value of the constant a?
A. -5 C. 7
B. -9 D. 8

20
PROBLEMS – SET 4
LOGARITHM, BIOMIAL THEOREM,
QUADRATIC EQUATION
log 𝑥
1. If 1−log10 = 2, what is the value of x?
10 2
1
A. C. 4
4
B. 25 D. 5
2. Solve for x: log 6 + 𝑥 log 4 = log 4 + log(32 + 4𝑥 )
A. 1 C. 3
B. 2 D. 4
3. Which of the following cannot be used as a base of a system of logarithm?
A. e C. 2
B. 10 D. 1
4. If log 5.2 1000 = 𝑥, what is the value of x?
A. 4.19 C. 3.12
B. 5.23 D. 4.69
5. Find the value of a in the equationlog 𝑎 2187 = 7/2.
A. 3 C. 9
B. 6 D. 12
6. If log 2 = 𝑥 and log 3 = 𝑦, find log 1.2
A. 2x + y C. 2x + y – 1
B. 2xy/10 D. xy – 1
log 𝑥 𝑦
7. is equal to:
log 𝑦 𝑌
𝑥𝑦
A. 𝑦𝑌
C. (𝑦 log 𝑥)/ (𝑥 log 𝑦)
B. (𝑦 log 𝑥) − (𝑥 log 𝑦) D. 1
𝑎𝑥+𝑏
8. If10 = 𝑃, what is the value of x?
1 1
A. (𝑎)(log 𝑃 − 𝑏) C. (𝑎)𝑃10−𝑏
1 1
B. (𝑎) log(𝑃 − 𝑏) D. (𝑎) log 𝑃10
9. Find the value of log(𝑎𝑎 ) ^𝑎
A. 2𝑎 log 𝑎 C. 𝑎 log 𝑎2
2
B. 𝑎 log 𝑎 D. (𝑎 log 𝑎)𝑎
10. Solve for x: 𝑥 = 𝑙𝑜𝑔𝑏 𝑎 x 𝑙𝑜𝑔𝑐 𝑑 x 𝑙𝑜𝑔𝑑 𝑐
A. 𝑙𝑜𝑔𝑏 𝑎 C. 𝑙𝑜𝑔𝑏 𝐶
B. 𝑙𝑜𝑔𝑎 𝐶 D. 𝑙𝑜𝑔𝑑 𝑎
11. Find the positive value of 𝑥 if 𝑙𝑜𝑔𝑥 36 = 2
A. 2 C. 6
B. 4 D. 8
12. Find 𝑥 if 𝑙𝑜𝑔𝑥 27 + 𝑙𝑜𝑔𝑥 3 = 2

21
A. 9 C. 8
B. 12 D. 7
13. Find 𝑎 if 𝑙𝑜𝑔2 (𝑎 + 2) + 𝑙𝑜𝑔2 (𝑎 − 2) = 5.
A. 2 C. 6
B. 4 D. 8
14. Solve for x 𝑙𝑜𝑔5 𝑥 = 3.
A. 115 C. 135
B. 125 D. 145
15. Find log 𝑃 if ln 𝑃 = 8.
A. 2980.96 C. 3.47
B. 2542.33 D. 8.57
16. If 𝑙𝑜𝑔8 𝑥 = −𝑛, then 𝑥 is equal to:
A. 8𝑛 C. 1 / 8𝑛
−𝑛
B. 1 / 8 D. 81 / 𝑛
17. If3𝑙𝑜𝑔10 𝑥 − 𝑙𝑜𝑔10 𝑦 = 0, find 𝑦 in terms of x.
A. 𝑦 = 3√𝑥 C. 𝑦 = 𝑥³
B. 𝑦 = √𝑥³ D. 𝑦 = 𝑥
18. Which of the following is correct?
1 1
A. −2 log 7 = C. 𝑙𝑜𝑔7 ( ) = −2
49 49
1
B. 𝑙𝑜𝑔7 (−2) = 1/49 D. 𝑙𝑜𝑔7 (49) = 2
19. Log of the 𝑛th root of 𝑥 equals log of 𝑥 to the 1/ 𝑛 power and also equal to:
log (𝑥) log (𝑥)1/𝑛
A. 𝑛 C. 𝑛
B. 𝑛 log(𝑥) D. (𝑛 − 1) log(𝑥)
20. What is the natural logarithm of 𝑒 to the 𝑥𝑦 power?
A. 1/𝑥𝑦 C. 𝑥𝑦
B. 2.718/𝑥𝑦 D. 2.718𝑥𝑦
21. What expression is equivalent to log 𝑥 − log(𝑦 + 𝑧)?
A. log 𝑥 + log 𝑦 + log 𝑧 C. log 𝑥 − log 𝑦 − log 𝑧
𝑥
B. log[𝑦+𝑧] D. log 𝑦 + log(𝑥 + 𝑧)
22. What is the value of log to the base 10 of10003.3?
A. 9.9 C. 10.9
B. 99.9 D. 9.5
23. If 𝑙𝑜𝑔𝑥 2 + 𝑙𝑜𝑔2 𝑥 = 2, then the value of x is:
A. 1 C. 3
B. 2 D. 4
24. 𝑙𝑜𝑔6 845 =?
A. 4.348 C. 5.912
B. 6.348 D. 3.761
25. The logarithms of the quotient and the product of two numbers are 0.352182518 and
1.556302501, respectively:
A. 9 C. 11
B. 10 D. 12

22
26. The sum of the logarithms of two numbers is 1.748188 and the difference of their
logarithms is -0.0579919. One of the number is:
A. 9 C. 8
B. 6 D. 5
𝑒𝑥
27. Solve for 𝑦: 𝑦 = 𝑙𝑛 𝑒𝑥−2
A. 2 C. -2
B. 𝑥 D. 𝑥 − 2
28. What is the value of (log 5 to the base of 2) + (log 5 to the base 3)?
A. 3.97 C. 9.37
B. 7.39 D. 3.79
29. The logarithm of negative number is:
A. Irrational number C. imaginary number
B. Real number D. complex number
30. 38.5 to the 𝑥 power = 6.5 to the 𝑥 − 2 power, solve for 𝑥 using logarithms.
A. 2.70 C. -2.10
B. 2.10 D. -2.02
31. Find the 6 term of the expansion of (1/2𝑎 − 3)16
th
22113 22113
A. − 256𝑎11 C. − 128𝑎11
66339 66339
B. 128𝑎11 D. − 256𝑎11
12
32. In the expansion of 𝑥 + 4𝑦) , the numerical coefficient of the 5th term is
A. 253,440 C. 63,360
B. 126,720 D. 506,880
33. The middle term in the expansion of (𝑥 2 − 3)8 is:
A. −70𝑥 8 C. −5670𝑥 8
B. 70𝑥 8 D. 5670𝑥 8
34. The term involving 𝑥 in the expansion of (𝑥 2 + 2/𝑥)12 is:
9

A. 25434𝑥 9 C. 25344𝑥 9
9
B. 52344𝑥 D. 23544𝑥 9
1
35. The constant term in the expansion of (𝑥 + 3 )15 is:
𝑥2
A. 3003 C. 6435
B. 5005 D. 7365
36. Find the sum of the coefficients in the expansion of (𝑥 + 2𝑦 − 𝑧)8
A. 256 C. 1
B. 1024 D. 6
37. The sum of the coefficients in the expansion of (𝑥 + 2𝑦 + 𝑧)4 (𝑥 + 3𝑦)5 is:
A. 524,288 C. 131,072
B. 65,536 D. 262,144
38. The sum of the coefficients in the expansion of (𝑥 + 𝑦 − 𝑧)8 is:
A. Less than 2 C. from 2 to 5
B. Above 10 D. from 5 to 10
39. What is the sum of the coefficients of the expansion of (2𝑥 − 1)20?
A. 1 C. 215
B. 0 D. 225
40. In the quadratic equation 𝐴𝑥 2 + 𝐵𝑥 + 𝐶 = 0, the product of the roots is:

23
A. C/A C. –C/A
B. –B/A D. B/A
41. If ¼ and -7/2 are the roots of the quadratic equation𝐴𝑥 2 + 𝐵𝑥 + 𝐶 = 0, what is the
value of𝐵?
A. -28 C. -7
B. 4 D. 26
42. In the equation3𝑥 2 + 4𝑥 + (2ℎ − 5) = 0, find ℎ if the product of the roots is 4.
A. -7/2 C. 17/2
B. -10/2 D. 7/2
43. If the roots of 𝑎𝑥² + 𝑏𝑥 + 𝑐 = 0 are 𝑢 and 𝑣, then the roots of 𝑐𝑥² + 𝑏𝑥 + 𝑎 = 0 are:
A. 𝑢 and 𝑣 C. 1/ 𝑢 and 1/𝑣
B. -𝑢 and 𝑣 D. −1/𝑢 and -1/𝑣
44. If the roots of the quadratic equation 𝑎𝑥² + 𝑏𝑥 + 𝑐 = 0 are 3 and 2 and 𝑎, 𝑏 and 𝑐 are
all whole numbers, find 𝑎 + 𝑏 + 𝑐.
A. 12 C. 2
B. -2 D. 6
45. The equation whose roots are the reciprocals of the roots of 2𝑥 2 − 3𝑥 − 5 = 0 is:
A. 5𝑥 2 + 3𝑥 − 2 = 0 C. 5𝑥 2 − 2𝑥 − 3 = 0
2
B. 3𝑥 − 5𝑥 − 2 = 0 D. 2𝑥 2 − 5𝑥 − 3 = 0
46. The roots of a quadratic equations are 1/3 and 1/4. What is the equation?
A. 12𝑥 2 + 7𝑥 + 1 = 0 C. 12𝑥 2 − 7𝑥 + 1 = 0
B. 12𝑥 2 + 7𝑥 − 1 = 0 D. 12𝑥 2 − 7𝑥 − 1 = 0
2
47. Find 𝑘 so that the expression 𝑘𝑥 − 3𝑘𝑥 + 9 is a perfect square.
A. 3 C. 12
B. 4 D. 6
2
48. Find 𝑘 so that the expression 4𝑥 + 𝑘𝑥 + 1 = 0 will only have one real solution.
A. 1 C. 3
B. 4 D. 2
49. The only root of the equation 𝑥 2 − 6𝑥 + 𝑘 = 0 is:
A. 3 C. 6
B. 2 D. 1
50. Two engineering students are solving a problem leading to a quadratic equation. One
student made a mistake in the coefficient of the first-degree term, got roots of 2 and -
3. The other student made a mistake in the coefficient of the constant term got roots
of -1 and 4. What is the correct equation?
A. 𝑥 2 − 6𝑥 − 3 = 0 C. 𝑥 2 + 3𝑥 + 6 = 0
2
B. 𝑥 + 6𝑥 + 3 = 0 D. 𝑥 2 − 3𝑥 − 6 = 0

24
- Part vi:
rate of change

A rate of change is a rate that describes how one quantity changes in relation to another
quantity. If x is the independent variable and y is the dependent variable, then
𝑐ℎ𝑎𝑛𝑔𝑒 𝑖𝑛 𝑦
𝑟𝑎𝑡𝑒 𝑜𝑓 𝑐ℎ𝑎𝑛𝑔𝑒 =
𝑐ℎ𝑎𝑛𝑔𝑒 𝑖𝑛 𝑥

Rates of change can be positive or negative. This corresponds to an increase or decrease in the y
-value between the two data points. When a quantity does not change over time, it is called zero
rate of change.
Positive rate of change

- When the value of x increases, the value of y increases and the graph slants upward.
Negative rate of change
- When the value of x increases, the value of y decreases and the graph slants downward.
Zero rate of change
- When the value of x increases, the value of y remains constant. That is, there is no change in y
value and the graph is a horizontal line.
Examples:
- 𝑇𝑖𝑚𝑒 𝐷𝑟𝑖𝑣𝑖𝑛𝑔 (ℎ)𝑥 𝐷𝑖𝑠𝑡𝑎𝑛𝑐𝑒 𝑇𝑟𝑎𝑣𝑒𝑙𝑙𝑒𝑑 (𝑚𝑖 )𝑦
3 180
6 270
9 360
𝑐ℎ𝑎𝑛𝑔𝑒 𝑖𝑛 𝑦 𝑐ℎ𝑎𝑛𝑔𝑒 𝑖𝑛 𝑑𝑖𝑠𝑡𝑎𝑛𝑐𝑒 270−180 90
𝑟𝑎𝑡𝑒 𝑜𝑓 𝑐ℎ𝑎𝑛𝑔𝑒 = = 𝑟𝑎𝑡𝑒 𝑜𝑓 𝑐ℎ𝑎𝑛𝑔𝑒 = = = 30
𝑐ℎ𝑎𝑛𝑔𝑒 𝑖𝑛 𝑥 𝑐ℎ𝑎𝑛𝑔𝑒 𝑖𝑛 𝑡𝑖𝑚𝑒 6−3 3
The rate of change is 30. This means a vehicle is traveling at a rate of 30 miles per hour

25
Average Rate of Change
It is a measure of how much the function changed per unit, on average, over that interval.
It is derived from the slope of the straight line connecting the interval's endpoints on the
function's graph.
The average rate of change of function f over the interval 𝑎 ≤ 𝑥 ≤ 𝑏 is given by this expression:
𝑓 (𝑏) − 𝑓(𝑎)
𝑏−𝑎
Finding Average Rate of Change from Equation
1. 𝑔(𝑥 ) = 𝑥 3 − 9𝑥 over the interval 1 ≤ 𝑥 ≤ 6.
𝑔(1) = 13 − 9(1) = −8
𝑔(6) = 63 − 9(6) = 1
𝑔(6)−𝑔(1) 162−(−8)
Average rate of change = = = 34
6−1 5

Finding Average Rate of Change from Graph


1. Find the average rate of change of f over the interval 0 ≤ 𝑥 ≤ 9.

The graph shows 𝑓(0) = −7 𝑎𝑛𝑑 𝑓(9) = 3


𝑓(9)−𝑓(0) 3−(−7) 10
Average rate of change = = =
9− 0 9 9
Instantaneous Rate of Change
The instantaneous rate of change measures the rate of change, or slope, of a curve at a certain instant.
Thus, the instantaneous rate of change is given by the derivative.
The Formula of Instantaneous Rate of Change represented with limit exists in,
∆𝑦 𝑡(𝑎+ℎ)−(𝑡(𝑎))
𝑓 ′(𝑎) = lim = lim
∆𝑥→0 ∆𝑥 𝑥→0 ℎ
with respect to x, when 𝑥 = 𝑎 𝑎𝑛𝑑 𝑦 = 𝑓(𝑥)

26
Exercises:
1 – 2. Let 𝒚 = 𝒙𝟐 − 𝟐
A. Find the average rate of change of y with respect to x over the interval [2,5].
B. Find the instantaneous rate of change of y with respect to x at point x = 4.

3. What is the average rate of change of 𝑔 over the interval −8 ≤ 𝑥 ≤ −2 ?

4. Compute the Instantaneous rate of change of the function 𝑓 (𝑥 ) = 9𝑥 2 + 12𝑥 𝑎𝑡 𝑥 = 2


Solutions:
1-2.
A. Find the average rate of change of y with respect to x over the interval [2,5].
𝑦 = 𝑥 2 − 2 = 𝑓 (𝑥 ) = 𝑥 2 − 2

𝑓 (2) = 22 − 2 = 2
𝑓 (5) = 52 − 2 = 23

𝑓(5)−𝑓(2) 23−2 21
Average rate of change = = = =7
5−2 3 3

B. Find the instantaneous rate of change of y with respect to x at point x = 4.


𝑦 = 𝑥 2 − 2 = 𝑓 (𝑥 ) = 𝑥 2 − 2
𝑓 ′(𝑥 ) = 2𝑥 = 2(4) = 8
3. The graph shows 𝑔(−8) = −8 𝑎𝑛𝑑 𝑔(−2) = 3
𝑔(−2)−𝑔(−8) 3−(−8) 11
Average rate of change = = =
−2−(−8) 6 6

4. Compute the Instantaneous rate of change of the function 𝑓 (𝑥 ) = 9𝑥 2 + 12𝑥 𝑎𝑡 𝑥 = 2


𝑓 (𝑥 ) = 9𝑥 2 + 12𝑥
′( )
𝑓 𝑥 = 18𝑥 + 12 = 18(2) + 12 = 48

27
Problem set 5 – age, mixture, work,
clock, number problems
1. Two times the father’s age is 8 more than six times his son’s age. Ten years ago, the sum
of their ages was 44. The age of the son is:
A. 49 C. 20
B. 15 D. 18
2. Peter’s age 13 years ago was 1/3 of his age 7 years hence. How old is Peter?
A. 15 C. 23
B. 21 D. 27
3. A man is 41 years old and in seven years he will be four times as old as his son is at that
time. How old is his son now?
A. 9 C. 5
B. 4 D. 8
4. A father is three times as old as his son. Four years ago, he was four times as old as his
son was at that time. How old is his son?
A. 36 years old C. 32 years old
B. 24 years old D. 12 years old
5. The ages of the mother and her daughter are 45 and 5 years, respectively. How many
years will the mother be three times as old as her daughter?
A. 5 C. 15
B. 10 D. 20
6. Mary is 24 years old. Mary is twice is as old as Ana was when Mary was as old as Ana is
now. How old is Ana?
A. 16 C. 19
B. 18 D. 20
7. The sum of the parent’s ages is twice the sum of their children’s ages. Five years ago, the
sum of the parent’s ages is four times the sum of their children’s ages. In fifteen years,
the sum of the parent’s ages will be equal to the sum of their children’s age. How many
children were in the family?
A. 2 C. 4
B. 3 D. 5
8. Two thousand (2000) kg of steel containing 8% nickel is to be made by mixing steel
containing 14% nickel with another steel containing 6% nickel. How much of the steel
containing 14% nickel is needed?
A. 1500 kg C. 750 kg
B. 800 kg D. 500 kg
9. A 40-gram alloy containing 35% gold is to be melted with a 20-gram alloy containing
50% gold. How much percentage of gold is the resulting alloy?
A. 40% C. 45%
B. 30% D. 35%

10. In what ratio must a peanut costing P 240.00 per kg be mixed with a peanut costing P
340.00 per kg so that a profit of 20% is made by selling the mixture at P 360.00 per kg.?

28
A. 1:2 C. 2:3
B. 3:2 D. 3:5
11. A 100-kilogram salt solution originally 4% by weight. Salt in water is boiled to reduce
water content until the concentration is 5% by weight salt. How much water is
evaporated?
A. 10 C. 20
B. 15 D. 25
12. A pound of alloy of lead and nickel weighs 14.4 ounces in water, where lead losses 1/11
of its weight and nickel losses 1/9 of its weight. How much of each metal is in the alloy?
A. Lead = 7.2 ounces; Nickel = 8.8 ounces
B. Lead = 8.8 ounces; Nickel = 7.2 ounces
C. Lead = 6.5 ounces; Nickel = 5.4 ounces
D. Lead = 7.8 ounces; Nickel = 4.2 ounces
13. An alloy of silver and gold weighs 15 oz. in air and 14 oz. in water. Assuming that silver
losses 1/10 of its weight in water and gold losses 1/18 of its weight, how many oz. at each
metal are in the alloy?
A. silver = 4.5 oz.; gold = 10.5 oz.
B. silver = 3.75 oz.; gold = 11.25 oz.
C. silver = 5 oz.; gold = 10 oz.
D. silver = 2.75 oz.; gold = 12.25 oz.
14. A pump can pump out a tank in 11 hours. Another pump can pump out the same tank in
20 hours. How long will it take both pumps together to pump out the tank?
A. ½ hours C. 6 hours
B. ½ hours D. 7 hours?
15. Mr Brown can wash his car in 15 minutes, while his son takes twice as long to do the
same job. If they work together, how may minutes can they do the washing?
A. 6 C. 10
B. 8 D. 12
16. One pipe can fill a tank in 5 hours and another pipe can fill the same tank in 4 hours. A
drainpipe can empty the full content of the tank in 20 hours. With all the three pipes
open, how long will it take to fill the tank?
A. 2 hours C. 1.92 hours
B. 2.5 hours D. 1.8 hours
17. A swimming pool is filled through its inlet pipe and then emptied through its outlet pipe
in a total of 8 hours. If water enters through inlet and simultaneously allowed to leave
through its outlet, the pool is filled in 7 ½ hours. Find how long will it take to fill the pool
with the outlet closed
A. 6 C. 3
B. 2 D. 5
18. Three persons can do a piece of work alone in 3 hours, 4 hours, and 6 hours, respectively.
What fraction of the job can they finish in one hour working together?
A. 3/4 C. ½
B. 4/3 D. 2/3
19. A father and his son can dig a well if the father works 6 hours and his son works 12 hours
or they can do it if the father works 9 hours and the son works 8 hours. How long will it
take for the son to dig the well alone?

29
A. 5 hours C. 15 hours
B. 10 hours D. 20 hours
20. Peter and Paul can do a certain job in 3 hours. On a given day, they worked together for
1 hour then Paul left and Peter finishes the rest of the work in 8 more hours. How long
will I take for Peter to do the job alone?
A. 10 hours C. 12 hours
B. 11 hours D. 13 hours
21. Pedro can paint a fence 50% faster than Juan and 20% faster than Pilar and together they
can paint a given fence in 4 hours. How long will it take Pedro to paint the same fence if
he had to work alone?
A. 10 hrs. C. 13 hrs.
B. 11 hrs. D. 15 hrs.
22. Nonoy can finish a certain job in 10 days if Imelda will help for 6 days. The same work
can be done by Imelda in 12 days if Nonoy helps for 6 days. If they work together, how
long will it take for them to do the job?
A. 8.9 C. 9.2
B. 8.4 D. 8
23. A pipe can fill up a tank with the drain open in three hours. If the pipe runs the drain open
for one hour and then the drain is closed, it will take 45 more minutes for the pipe to fill
the tank. If the drain will be closed right at the start of filling, how long will it take for the
pipe to fill the tank?
A. 1.15 hrs C. 1.325 hrs
B. 1.125 hrs D. 1.525 hrs
24. Delia can finish a job in 8 hours. Daisy can do it in 5 hours. If Delia worked for 3 hours
and then Daisy was asked to help her finish it, how long will Daisy have to work with
Delia to finish the job?
A. 2/5 hours C. 28 hours
B. 25/14 hours D. 1.923 hours
25. A job could be done by eleven workers in 15 days. Five workers started the job. They
were reinforced with for more workers at the beginning of the 6 th day. Find the total
number of days it took them to finish the job.
A. 22.36 C. 23.22
B. 21.42 D. 20.56
26. On one job, two power shovels excavate 20,000m³ of earth, the larger shovel working for
40 hours and the smaller for 35 hours. Another job, they removed 40,000m³ with the
larger shovel working 70 hours and the smaller working 90 hours. How much earth can
the larger shovel move in one hour?
A. 173.91 C. 368.12
B. 347.83 D. 162.22

27. A and B can do a piece of work in 42 days, B and C in 31 days, and A and C in 20 days.
Working together, how many days can all of them finish the work?
A. 18.9 C. 17.8
B. 19.4 D. 20.9
28. Eight men can dig 150 ft of trench in 7 hrs. Three men can backfill 100 ft of the trench in
4 hrs. The time that it will take 10 men to dig and fill 200 ft of trench is:

30
A. 9.867 hrs C. 8.967 hrs
B. 9.687 hrs D. 8.687 hrs
29. In two hours, the minute hand of the clock rotates through an angle of:
A. 45° C. 360°
B. 90° D. 720°
30. In one day (24 hours), how many times will the hour-hand and minute-hand of a
continuously driven clock be together?
A. 21 C. 23
B. 22 D. 24
31. How many minutes after 3:00 PM will the minute hand of the clock overtakes the hour
hand?
A. 14/12 minutes C. 16-4/11 minutes
B. 16-11/12 minutes D. 14/11 minutes
32. How many minutes after 10:00 o’clock will the hands of the clock be opposite each other
for the first time?
A. 21.41 C. 21.81
B. 22.31 D. 22.61
33. What time between the hours of 12:00 noon and 1:00 pm would the hour-hand and the
minute-hand of a continuously driven clock be in straight line?
A. 12:33 pm C. 12:37 pm
B. 12:30 pm D. 12:287 pm
34. At what time after 12:00 noon will the hour hand and the minute hand of a clock first
form an angle of 120°?
A. 21.818 C. 21.181
B. 12:21.181 D. 12:21.818
35. From the time 6:15 PM to the time 7:45 PM of the same day, the minute hand of a
standard clock describe an arc of:
C. 360° C. 540°
D. 120° D. 720°
36. It is now between 3 and 4 o’clock and in twenty minutes the minute-hand will be as much
as the hour-hand as it is now behind it. What is the time now?
A. 3:06.36 C. 3:09.36
B. 3:07.36 D. 3:08.36
37. A man left his home at past 3:00 o’clock PM as indicated in his wall clock. Between two
to three hours after, he returned home and noticed that the hands of the clock
interchanged. At what time did he left his home?
A. 3:27.27 C. 3:22.22
B. 3:31.47 D. 3:44.44
38. The sum of the reciprocals of two numbers is 11. Three times the reciprocal of one of the
numbers is three more than twice the reciprocal of the other number. Find the numbers.
A. 5 and 6 C. 1/5 and 1/6
B. 7 and 4 D. 1/7 and ¼
39. If a two-digit number has x for its unit’s digit and y for its ten’s digit, represent the
number/
A. 𝑦𝑥 C. 10𝑥 + 𝑦
B. 10𝑦 + 𝑥 D. 𝑥 + 𝑦

31
40. One number if five less than the other number. If their sum is 135, what are the numbers?
A. 70 & 75 C. 65 & 70
B. 60 & 65 D. 75 & 80
41. In a two-digit number, the unit’s digit is 3 greater than the ten’s digit. Find the number if
it is 4 times as large as the sum of its digits.
A. 47 C. 63
B. 58 D. 25
42. Find two consecutive even integers such that the square of the larger is 44 greater than
the square of the smaller integer.
A. 10 & 12 C. 8 & 10
B. 12 & 14 D. 14 & 16
43. Twice the middle digit of a three-digit number is the sum of the other two. If the number
is divided by the sum of its digit, the answer is 56 and the remainder is 12. If the digits
are reversed, the number becomes smaller by 594. Find the number.
A. 258 C. 852
B. 567 D. 741
44. The product of three consecutive integers in 9240. Find the third integer.
A. 20 C. 22
B. 21 D. 23
45. The product of two numbers is 1400. If three (3) is subtracted from each number, their
product becomes 1175. Find the bigger number.
A. 28 C. 32
B. 50 D. 40
46. The sum of the digits of a three-digit number is 14. The hundreds digit being 4 times the
units digit. If 594 is subtracted from the number, the order of the digits will be reversed.
Find the number.
A. 743 C. 653
B. 563 D. 842
47. The sum of two numbers is 21, and one number is twice the other. Find the numbers.
A. 7 and 14 C. 8 and 13
B. 6 and 15 D. 9 and 12
48. Ten less than four times a certain number is 14. Determine the number.
A. 4 C. 6
B. 5 D. 7
49. The denominator of a certain fraction is three more than twice the numerator. If 7 are
added to both terms of the fraction, the resulting fraction is 3/5. Find the original fraction.
A. 8/5 C. 13/5
B. 5/13 D. 3/5
50. Three times the first of three consecutive odd integers is three more than twice the third.
Find the integer.
A. 9 C. 13
B. 11 D. 15

32
Problem set 6 – motion, variation,
percent, miscellaneous problems
1. Nonoy left Pikit to drive to Davao at 6:15 PM and arrived at 11:45 PM. If he averaged 30
mph and stopped 1 hour for dinner, how far is Davao from Pikit.
A. 128 C. 160
B. 135 D. 256
2. A man fires a target 420 m away and hears the bullet strike 2 seconds after he pulled the
trigger. An observer 525 m away from the target and 455 m from the man heard the bullet
strike the target one second after he heard the report of the rifle. Find the velocity of the
bullet.
A. 525 m/s C. 350 m/s
B. 360 m/s D. 336 m/s
3. A man travels in a motorized banca at the rate of 12 kph from his barrio to the poblacion
and come back to his barrio at the rate of 10 kph. If his total time of travel back and forth
is 3 hours and 10 minutes, the distance from the barrio to the poblacion is:
A. 17.27 km C. 12.77 km
B. 17.72 km D. 17.32 km
4. It takes Michael 60 seconds to run around a 440-yard track. How long does it take Jordan
to run around the track if they meet in 32 seconds after they start together in a race
around the track in opposite directions?
A. 58.76 seconds C. 65.87 seconds
B. 68.57 seconds D. 86.57 seconds
5. Juan can walk from his home to his office at the rate of 5 mph and back at the rate of 2
mph. What is his average speed in mph?
A. 2.86 C. 4.12
B. 3.56 D. 5.89
6. Kim and Ken travelled at the same time at the rate of 20m/min, from the same point on a
circular track of radius 600 m. If Kim along the circumference and Ken towards the
centre, find their distance after 10 minutes.
A. 193 m C. 241 m
B. 202 m D. 258 m
7. Two ferryboats ply back and forth across a river with constant but different speeds,
turning at the riverbanks without loss of time. They leave the opposite shores at the same
instant, meet for the first time 900 meters from one shore, and meet for the second time
500 meters from the opposite shore. What is the width of the river?
A. 1500 m C. 2000 m
B. 1700 m D. 2200 m
8. A boat takes 2/3 as much time to travel downstream from C to D, as to return. If the rate
of the river’s current is 8 kph, what is the speed of the boat in still water?
A. 38 C. 40
B. 39 D. 41
9. A man rows downstream at the rate of 5mph and upstream at the rate of 2mph. How far
downstream should he go if he is to return in 7/4 hours after leaving?
A. 2 mi C. 3 mi

33
B. 3.5 mi D. 2.5 mi
10. A jogger starts a course at a steady rate of 8kph. Five minutes later, a second jogger the
same course at 10kph. How long will it take for the second jogger to catch the first?
A. 20 min C. 30 min
B. 25 min D. 15 min
11. At 2:00 pm, an airplane takes off at 340mph on an aircraft carrier. The aircraft carrier
moves due South at 25kph in the same direction as the plane. At 4:05 pm, the
communication between the plane and the aircraft carrier was lost. Determine the
communication range in miles between the plane and the carrier.
A. 656 miles C. 557 miles
B. 785 miles D. 412 miles
12. A boat going across a lake 8 km wide proceed 2 km at a certain speed and then completes
the trip at a speed ½ kph faster. By doing this, the boat arrives 10 minutes earlier than if
the original speed had been maintained. Find the original speed of the boat.
A. 2 kph C. 9 kph
B. 4 kph D. 5 kph
13. Given that w varies directly as the product of x and y and inversely as the square of z and
that w = 4, when x = 2, y = 6, and z = 3. Find w when x = 1, y = 4, and z =2.
A. 4 C. 1
B. 2 D. 3
14. If x varies directly as y and inversely as z, and x = 14 when y = 7 and z = 2, find x when z
= 4 and y = 16.
A. 14 C. 16
B. 4 D. 8
15. The electrical resistance of a cable varies directly as its length and inversely as the square
of its diameter. If a cable 600 meters long and 25 mm in diameter has a resistance of 0.1
ohm, find the length of the cable 75 mm in diameter with resistance of 1/6 ohm.
A. 6000 m C. 8000 m
B. 7000 m D. 9000 m
16. The electrical resistance offered by an electric wire varies directly as the length and
inversely as the square of the diameter of the wire. Compare the electrical resistance
offered by two pieces of wire of the same material, one being 100 m long and 5 mm in
diameter, and the other is 50 m long and 3 mm in diameter.
A. 𝑅1 = 0.57 𝑅2 C. 𝑅1 = 0.84 𝑅2
B. 𝑅1 = 0.72 𝑅2 D. 𝑅1 = 0.95 𝑅2
17. The time required for an elevator to lift a weight varies directly with the weight and the
distance through which it is to be lifted and inversely as the power of the motors. If it
takes 20 seconds for a 5-hp motor to lift 50 lbs. through 40 feet, what weight can an 80-
hp motor lift through a distance of 40 feet within 30 seconds?
A. 1000 lbs. C. 1175 lbs.
B. 1150 lbs. D. 1200 lbs.
18. The time required by an elevator to lift a weight, vary directly with the weight and the
distance through which it is to be lifted and inversely as the power of the motor. If it
takes 30 seconds for a 10-hp motor to lift 100 lbs. through 50 feet, what size of motor is
required to lift 800 lbs. in 40 seconds through a distance of 40 feet?
A. 48 hp C. 56 hp

34
B. 50 hp D. 58 hp
19. In a certain department store, the monthly salary of a saleslady is partly constant and
varies as the value of her sales for the month. When the value of her sales for the month
is P10, 000.00, her salary for that month is P900.00. When her sales goes up to P12,
000.00, her monthly salary goes up to P1, 000.00. What must be the value of her sales for
the month so that her salary for that month will be P2, 000.00.
A. P25,000.00 C. P32,000.00
B. P28,000.00 D. P36,000.00
20. A man sold 100 eggs. Eighty of them were sold at a profit of 30% while the rest were
sold at a loss of 40%. What is the percentage gain or loss on the whole stock?
A. 14% C. 16%
B. 15% D. 17%
21. The population of the country increases 5% each year. Find the percentage it will
increase in three years.
A. 5% C. 15.15%
B. 15% D. 15.76%
22. Pedro bought two cars, one for P600, 000.00 and the other for P400, 000.00. He sold the
first at a gain of 10% and the second at a loss of 12%. What was his total percentage gain
or loss?
A. 6% gain C. 1.20% gain
B. 0% gain D. 6% loss
23. A grocery owner raises the prices of his goods by 10%. Then he starts his Christmas sale
by offering the customers a 10% discount. How many percent of discount does the
customers actually get?
A. Nothing C. 9% discount
B. 1% discount D. they pay 1% more
24. Kim sold a watch for P3,500.00 at a loss of 30% on the cost price. Find the corresponding
loss or gain if he sold it for P5,050.00
A. 1% loss C. 1% gain
B. 10% loss D. 10% gain
25. By selling balut at P5.00 each, a vendor gains 20%. The cost price of egg rises by 12.5%.
If he sells the balut as the same price as before, find his new gain in percent.
A. 7.5% C. 8%
B. 5% D. 6.25%
26. The enrolment at college A and college B both grew up by 8% from 1980 to 1985. If the
enrolment in college A grew up by 800 and the enrolment in college B grew up by 840,
the enrolment was how much greater than the enrolment in college A in 1985?
A. 650 C. 483
B. 504 D. 540
27. A group consists of 𝑛 boys and 𝑛 girls. If two of the boys are replaced by two other girls,
then 49% of the group members will be boys. Find the value of𝑛.
A. 100 C. 50
B. 49 D. 51
28. On his Christmas Sale, a merchant marked a pair of slipper P180.00, which is 20% off
the normal retail price. If the retail price is 50% higher than the wholesale price, what is
the wholesale price of the slipper?

35
A. P18.00 C. P15.00
B. P17.00 D. 22.50
29. A certain Xerox copier produces 13 copies every 10 seconds. If the machine operates
without interruption, how many copies will it produce in an hour?
A. 780 C. 1825
B. 46800 D. 4680
30. At a certain printing plant, each of the machines prints 6 newspapers every second. If all
machines work together but independently without interruption, how many minutes will
it take to print the entire run of 18000 newspapers?
A. 50 x C. 50 / x
B. 3000 / x D. 3000 x
31. A manufacturing firm maintains one product assembly line to produce signal generators.
Weekly demand for the generators is 35 units. The line operates for 7 hours per day, 5
days per week. What is the maximum production time per unit in hours required for the
line to meet the demand?
A. 1 hour C. 3 hours
B. 0.75 hours D. 2.25 hours
32. Of the 316 people watching a movie, there are 78 more children than women and 56
more women the men. The number of men in the movie house is:
A. 176 C. 42
B. 98 D. 210
33. A certain department store has an inventory of Q units of a certain product at time t = 0.
The store sells the product at a steady rate of Q/A units per week, and exhausts the
inventory in A weeks. The amount of product in inventory at any time t is:
A. Q – (Q / A) t C. Q t – Q / A
B. Q + (Q / A) t D. Q t – (Q / A) t
34. A merchant has three items on sale: namely, a radio for P50, a clock for P30, and a
flashlight for P1. At the end of the day, she has sold a total of 100 of the three items and
has taken exactly P1000 on the total sales. How many radios did she sell?
A. 80 C. 16
B. 4 D. 20
35. The price of 8 calculators ranges from P200 to P1000. If their average price is P950, what
is the lowest possible price of any one of the calculators?
A. 500 C. 600
B. 550 D. 650
36. A deck of 52 playing cards is cut into two piles. The first pile contains 7 times as many
black cards as red cards. The second pile contains the number of red cards that is an exact
multiple as the number of black cards. How many cards are there in the first pile.
A. 14 C. 16
B. 15 D. 17
37. The Population of the Philippines doubled in the last 30 years from 1967 to 1997.
Assuming that the rate of population increase will remain the same in what year will the
population triple?
A. 2030 C. 2021
B. 2027 D. 2025
38. Determine the unit’s digit in the expansion of3855.

36
A. 3 C. 7
B. 9 D. 1
39. Find the 1987th digit in the decimal equivalent of 1785/9999 starting from the decimal
point.
A. 1 C. 8
B. 7 D. 5
40. Find the sum of all positive integral factors of 2048.
A. 4095 C. 4560
B. 3065 D. 1254
41. In how many ways can two integers be selected from the numbers 1, 2, 3,….50 so that
their difference is exactly 5?
A. 50 C. 45
B. 5 D. 41
42. A box contains 8 white balls, 15 green balls, 6 black balls, 8 red balls, and 13 yellow
balls. How many balls must be drawn to ensure that there will be three balls of the same
colour?
A. 8 C. 10
B. 9 D. 11
43. A shoe store sells 10 different sizes of shoes, each in both high-cut and low-cut variety,
each either rubber or leather, and each with white or black colour. How many different
kinds of shoes does he sell?
A. 64 C. 72
B. 80 D. 92
44. An engineer was told that a survey had been made on a certain rectangular field but the
dimensions had been lost. An assistant remembered that if the field had been 100 ft
longer and 25 ft narrower, the area would have been increased by 2500 sq. ft, and that if
it had been 100 ft shorter and 50 ft wider, the area would have been decreased 5000 sq. ft.
What was the area of the field?
A. 25,000 ft² C. 20,000 ft²
B. 15,000 ft² D. 22,000 ft²
45. A 10-meter tape is 5 mm short. What is the correct length in meters.
A. 9.995 m C. 9.95 m
B. 10.05 m D. 10.005 m
46. The distance between two points measured with a steel tape was recorded as 916.58 ft.
Later, the tape was checked and found to be only 99.9 ft long. What is the true distance
between the points?
A. 935.66 ft C. 955.66 ft
B. 966.15 ft D. 915.66 ft
47. A certain steel tape is known to be 100,000 feet long at a temperature of 70°F. When the
tape is at a temperature of 10°F, what tape reading corresponds to a distance of 90 ft?
Coefficient of linear expansion of the tape is 5.833 x 10-6 per °F.
A. 85.931 C. 90.031
B. 88.031 D. 93.031
48. A line was measured with a steel tape when the temperature was 30°C. The measured
length of the line was found to be 1,256,271 feet. The tape was afterwards tested when

37
the temperature was 10°C and it was found to be 100,042 feet long. What was the true
length of the line if the coefficient of expansion of the tape was 0.000011 per °C?
A. 1,275.075 feet C. 1,256.547 feet
B. 1,375.575 feet D. 1,249.385 feet
49. The standard deviation of the numbers 1, 4, and 7 is:
A. 2.3567 C. 3.2256
B. 2.4495 D. 3.8876
50. Three cities are connected by roads forming a triangle, all of different lengths. It is 30 km
around the circuit. One of the roads is 10 km long and the longest is 10 km longer than
the shortest. What is the length of the longest road?
A. 5 km C. 15 km
B.10 km D. 20 km

38
PART VII: chain RULE
and the general power rule

In order to know what is the chain rule and the general power rule, Let’s review first what are the
basic derivative rules as follows:

BASIC DERIVATIVE RULES

39
The chain rule states that the derivative of 𝑓(𝑔(𝑥)) 𝑖𝑠 𝑓′(𝑔(𝑥)) ⋅ 𝑔′(𝑥). In other words, it helps
us differentiate composite functions. For example, 𝑠𝑖𝑛(𝑥²) is a composite function because it can
be constructed as 𝑓(𝑔(𝑥)) for 𝑓(𝑥) = 𝑠𝑖𝑛(𝑥) and 𝑔(𝑥) = 𝑥².

Suppose that we have two functions 𝑓(𝑥)and 𝑔(𝑥) and they are both differentiable.

1. If we define 𝐹(𝑥) = (𝑓 ∘ 𝑔)(𝑥 ) then the derivative of 𝐹(𝑥) is,

𝐹′(𝑥) = 𝑓′(𝑔(𝑥))𝑔′(𝑥)

If we have 𝑦 = 𝑓(𝑢) and 𝑢 = 𝑔(𝑥) then the derivative of y is, 𝑑𝑦/𝑑𝑥 = 𝑑𝑦/𝑑𝑢 du/dx

Example:
1. 𝑓(𝑡) = (2𝑡 3 + 𝑐𝑜𝑠(𝑡))50

In this case the outside function is the exponent of 50 and the inside function is all the stuff on
the inside of the parenthesis. The derivative is then.

𝑓′(𝑡) = 50(2𝑡 3 + 𝑐𝑜𝑠(𝑡))49(6𝑡 2 − 𝑠𝑖𝑛(𝑡))


= 50(6𝑡 2 − 𝑠𝑖𝑛(𝑡))(2𝑡 3 + 𝑐𝑜𝑠(𝑡))49

2.ℎ(𝑤) = 𝑒𝑤 4 − 3𝑤 2 + 9
Distinguishing the outside function in the past two was genuinely straightforward since it truly
was the "outside" function in some sense. For this situation we should be somewhat cautious.

40
Review that the outside function is the last activity that we would act in an assessment. For this
situation if we somehow managed to evaluate this function the last activity would be the
exponential. Subsequently, the outside function is the exponential function and within work is its
exponent.
ℎ′(𝑤) = 𝑒𝑤 4 − 3𝑤 2 + 9(4𝑤 3 − 6𝑤)
= (4𝑤 3 − 6𝑤)𝑒𝑤 4 − 3𝑤 2 + 9
The general power rule is a special case of the chain rule. It is useful when finding the derivative
of a function that is raised to the nth power. The general power rule states that this derivative is
𝑛 times the function raised to the (𝑛 − 1) the power times the derivative of the function.

Here is the Power Rule with some sample values. See the pattern?

POSITIVE PATTERN NEGATIVE PATTERN


𝑛 𝑛−1
𝑓 𝑓’(𝑥 ) = 𝑛𝑥 𝑓’ 𝑓 𝑓’(𝑥 𝑛 ) = 𝑛𝑥 𝑛−1 𝑓’
𝑥 1𝑥 1−1 = 𝑥 0 1 𝑥 −1
−1𝑥 −1−1 = −𝑥 −2 −𝑥 −2
𝑥2
2𝑥 2−1 = 2𝑥 1 2𝑥 𝑥 −2 −2𝑥 −2−1 = −2𝑥 −3 −2𝑥 −3
𝑥3 3𝑥 3−1 = 3𝑥 2 3𝑥 2 𝑥 −3 −3𝑥 −3−1 = −3𝑥 −4 −3𝑥 −4
𝑥4 4𝑥 4−1 = 4𝑥 3 4𝑥 3 𝑥 −4 −4𝑥 −4−1 = −4𝑥 −5 −4𝑥 −5
so on and so forth so on and so forth

Examples:

1. Find the derivative of the function 𝑓(𝑥) = (3𝑥 − 2𝑥2)3

SOLUTION
If we let 𝑢 = (3𝑥 − 2𝑥 2 ) 𝑡ℎ𝑒𝑛 𝑓(𝑥) = 𝑢3 and by the General Power rule, the derivative is
𝑓 ′ (𝑥) = 3(3𝑥 − 2𝑥 2 )2 (3 − 4𝑥)
= (9 − 12𝑥 )(3𝑥 − 2𝑥 2 )2

2. Find the derivative of the function


𝑓(𝑥 ) = 𝑥 2 (1 − 𝑥 2 )

41
SOLUTION
1
𝑓((𝑥)) = 𝑥 2 (1 − 𝑥 2 )2
1 1
𝑓 ′(𝑥 ) = 𝑥 2 [12(1 − 𝑥 2 )−2 (−2𝑥 )] + (1 − 𝑥 2 )2 (2𝑥 )
1
= −𝑥 3 (1 − 𝑥 2 )−2 [−𝑥 2 (1) + 2(1 − 𝑥 2 )]
1
= 𝑥 (1 − 𝑥 2 )−2 (2 − 3𝑥 2 ) = 𝑥 (2 − 3𝑥 2 )1 − 𝑥 2

3.
Find the derivative of the function
𝑓(𝑥) = ( 3𝑥 − 1 𝑥 2 + 3 )2

SOLUTION
Following the Quotient rule, the Power rule and the Chain rule we have
f ' (x) = 2( 3𝑥 − 1 𝑥 2 + 3 ) [ (𝑥 2 + 3) (3) − (3𝑥 − 1 ) (2𝑥) ( 𝑥 2 + 3 ) 2 ]

= 2(3𝑥 − 1) (−3𝑥 2 + 2𝑥 + 9) ( 𝑥 2 + 3 )2

Exercises:

1. 𝑓(𝑥) = 𝑠𝑖𝑛(3𝑥 2 + 𝑥)𝑓(𝑥) = 𝑠𝑖𝑛 (3𝑥 2 + 𝑥)

2. 𝑓(𝑡) = (2𝑡 3 + 𝑐𝑜𝑠(𝑡))50𝑓(𝑡) = (2𝑡 3 + 𝑐𝑜𝑠 (𝑡))50

3. ℎ(𝑤) = 𝑒𝑤 4 − 3𝑤 2 + 9ℎ(𝑤) = 𝑒𝑤 4 − 3𝑤 2 + 9

4. 𝑦 = 𝑠𝑒𝑐(1 − 5𝑥)

Answers:

1. It looks like the outside function is the sine and the inside function is 3x2+x. The derivative is then.
f′(x) = (6x + 1)cos(3x 2 + x)
2. In this case the outside function is the exponent of 50 and the inside function is all the stuff on the
inside of the parenthesis. The derivative is then.
𝑓′(𝑡) = 50(2𝑡 3 + 𝑐𝑜𝑠(𝑡))49(6𝑡 2 − 𝑠𝑖𝑛(𝑡))
= 50(6𝑡 2 − 𝑠𝑖𝑛(𝑡))(2𝑡 3 + 𝑐𝑜𝑠(𝑡))49
3. Identifying the outside function in the previous two was fairly simple since it really was the “outside”
function in some sense. In this case we need to be a little careful. Recall that the outside function is the
last operation that we would perform in an evaluation. In this case if we were to evaluate this function

42
the last operation would be the exponential. Therefore, the outside function is the exponential function
and the inside function is its exponent. Here’s the derivative:
ℎ′(𝑤) = 𝑒𝑤 4 − 3𝑤 2 + 9(4𝑤 3 − 6𝑤)

= (4𝑤 3 − 6𝑤)𝑒𝑤 4 − 3𝑤 2 + 9
4. In this case the outside function is the secant and the inside is the 1−5x.
𝑦′ = 𝑠𝑒𝑐(1 − 5𝑥)𝑡𝑎𝑛(1 − 5𝑥)(−5)

= −5𝑠𝑒𝑐(1 − 5𝑥)𝑡𝑎𝑛(1 − 5𝑥)

43
PROBLEM SET 7: PROGRESSION, MATRIX,
DETERMINANT, VENN DIAGRAM
1. How many terms of the sequence -9, -6, -3, … must be taken so that the sum is 66?
a. 13
b. 12
c. 4
d. 11
2. The sum of the progression 5, 8, 11, 14 … is 1025. How many terms are there?
a. 22
b. 23
c. 24
d. 25
3. There are seven arithmetic means between 3 and 35. Find the sum all the terms.
a. 169
b. 171
c. 167
d. 173
4. There are nine (9) arithmetic means between 11 and 51. The sum of the progression is:
a. 279
b. 341
c. 376
d. 254
5. The sum of all even number from 0 to 420 is:
a. 43410
b. 44300
c. 44310
d. 44130
6. Which of the following numbers should be changed to make all the numbers form an arithmetic
progression when properly arranged?
a. 27/14
b. 33/28
c. 45/28
d. 20/14
7. The first term of an arithmetic progression (A.P.) is 6 and the 10 th term is 3 times the second
term. What is the common difference?
a. 1
b. 2

44
c. 3
d. 4
8. The sum of five arithmetic means between 34 and 42 is:
a. 150
b. 160
c. 190
d. 210
9. The positive value of a so that 4𝑥, 5𝑥 + 4, 32 − 1 will be in arithmetic progression is:
a. 2
b. 3
c. 4
d. 5
10. Solve for 𝑥 if 𝑥 + 3𝑥 + 5𝑥 + 7𝑥 + ⋯ + 49𝑥 = 625.
a. ¼
b. ½
c. 1
d. 1¼
11. The 10𝑡ℎ term of the series a, a – b, a-2b, … is:
a. a – 6b
b. a – 9b
c. 2a – b
d. a + 9b
12. If the sum of the first 13 terms of two arithmetic progressions are in the ratio 7:3, find the ratio
of their corresponding 7𝑡ℎ term.
a. 3:7
b. 1:3
c. 7:3
d. 6:7
13. If 1/x, 1/y, 1/z are arithmetic progression, then y is equal to:
a. x–z
b. ½ (x + 2z)
c. (x + z) / 2xz
d. 2 xz / (x + z)
14. Find the 30𝑡ℎ term of the A.P, 4,7,10 …
a. 88
b. 91
c. 75
d. 90

45
15. Find the 100𝑡ℎ term of the sequence in 1.01, 1.00, 0.99, …
a. 0.05
b. 0.04
c. 0.03
d. 0.02
16. The sum of all numbers between 0 and 10,000 which is exactly divisible by 77 is:
a. 546,546
b. 645,568
c. 645,645
d. 645,722
17. What is the sum of the following finite sequence of terms? 18, 25, 32, 39, …, 67.
a. 234
b. 181
c. 213
d. 340
18. Find 𝑥 in the series: 1, 1/3, 0.2, x.
a. 1/6
b. 1/8
c. 1/7
d. 1/9
19. Find the fourth term of the progression ½, 0.2, 0.125, …
a. 0.102
b. 1/10
c. 1/11
d. 0.099
20. The 10𝑡ℎ term of the progression 6/5, 4/3, 3/2, … is:
a. 12
b. 10/3
c. 12/3
d. 13/3
21. The geometric mean of 4 and 64 is:
a. 48
b. 16
c. 34
d. 23

46
22. The geometric mean of a and b is:
a. √𝑎𝑏
b. (a + b)/2
c. 1/b
d. ab/2
23. Determine the sum of the infinite geometric series of 1, -1/5, +1/25, …?
a. 4/5
b. 5/7
c. 4/6
d. 5/6
24. There are 6 geometric means between 4 and 8748. Find the sum of all the terms.
a. 13120
b. 15480
c. 10250
d. 9840
25. Find the sum of the infinite geometric progression 6, -2, 2/3 …
a. 5/2
b. 9/2
c. 7/2
d. 11/2
26. Find the sum of the first 10 terms of the Geometric Progression 2, 4, 8, 16, …
a. 1023
b. 2046
c. 1596
d. 225
27. The 1𝑠𝑡 , 4𝑡ℎ , 𝑎𝑛𝑑 8𝑡ℎ terms of an A.P. are themselves geometric progression (G.P.). What is
the common ratio of the G.P.
a. 4/3
b. 5/3
c. 2
d. 7/3
28. Determine x so that 𝑥, 2𝑥 + 7, 10𝑥 − 7 will form a geometric progression.
a. -7
b. 6
c. 7
d. -6

47
29. The fourth term of a geometric progression is 189 and the sixth term is 1701, the 8𝑡ℎ term is:
a. 5103
b. 1240029
c. 45927
d. 15309
30. The sum of three numbers in arithmetical progression is 45. If 2 is added to the first number,
3 to the second, and 7 to the third, the new numbers will be in geometrical progression. Find the
common difference in A.P.
a. -5
b. 1-
c. 6
d. 5
31. The geometric mean and the harmonic mean of two numbers are 12 and 36/5 respectively.
What are the numbers?
a. 36 & 4
b. 72 & 8
c. 36 & 8
d. 72 & 4
32. If 𝑥, 4𝑥 + 8, 30𝑥 + 24 are in geometric progression, find the common ratio.
a. 2
b. 4
c. 6
d. 8
33. A besiege fortress is held by 5700 men who have provisions for 66 days. If the garrison loses
20 men each day, for how many days can the provision hold out?
a. 60
b. 72
c. 76
d. 82
34. If one third of the air in the tank is removed by each stroke of an air pump, what fractional
part of the total air is removed in 6 strokes?
a. 0.9122
b. 0.0877
c. 0.8211
d. 0.7145
35. A rubber ball is dropped from a height of 15 m. On each rebound, it rises 2/3 of the height
from which it last fell. Find the distance traveled by the ball before it comes to rest.
a. 75 m
b. 96 m

48
c. 100 m
d. 85 m
36. In the recent Bosnia conflict, the NATO forces captured 6400 soldiers. The provisions on hand
will last for 216 meals while feeding 3 meals a day. The provisions lasted 9 more days because of
daily death. At an average, how many died per day?
a. 15.2
b. 17.8
c. 18.3
d. 19.4
37. To build a dam, 60 men must work 72 days. If all 60 men are employed at the start but the
number is decreased by 5 men at the end of each 12-day period, how long will it take to complete
the dam?
a. 108 days
b. 9 days
c. 94 days
d. 60 days
38. In a benefit show, a number of wealthy men agreed that the first one to arrive would pay 10
centavos to enter and each later arrival would pay twice as much as the preceding man. The total
amount collected from all of them was P104,857.50. How many wealthy men had paid?
a. 18
b. 19
c. 20
d. 21
7 8
39. Evaluate the following determinant: | |
9 4
a. 64
b. 44
c. 54
d. -44
40. The following equation involves two determinants:
3 𝑥 2 −1
| |= | |
2 2 𝑥 −3
The value of x is:
a. 1
b. 3
c. 4
d. 3

49
41. Evaluate the following determinant:
1 5 −2
|2 1 −3|
3 −2 1
a. -24
b. 24
c. -46
d. 46
42. Compute the value of x from the following:
4 −1 2 3
𝑥 = |2 0 2 1 |
10 3 0 1
14 2 4 5
a. 27
b. -28
c. 26
d. -29
43. Evaluate the following determinant:
1 4 2 −1
𝐷 = |2 −1 0 −3|
−2 3 1 2
0 2 1 4
a. 5
b. -4
c. 4
d. -5
1 3
44. Given: Matrix 𝐴 = [ ]
−2 1
−1 −2
Matrix𝐵 = [ ]
−1 1
Find A+2B.
1 0
a. [ ]
2 1
1 0
b. [ ]
1 3
−1 3
c. [ ]
0 1
−1 −1
d. [ ]
0 3

50
1 2
45. Elements of 𝑀𝑎𝑡𝑟𝑖𝑥 𝐵 = [ ]
0 −5
3 6
Elements of 𝑀𝑎𝑡𝑟𝑖𝑥 𝐶 = [ ]
4 1
Find the elements of the product of the two matrices, matrix BC.
11 8
a. [ ]
20 −5
15 9]
b. [
−22 4
12 10
c. [ ]
20 −4
15 15
d. [ ]
−17 −6
46. Solve for 𝑥 and 𝑦 from the given relationship:
1 1 𝑥 2
[ ] |𝑦| = [ ]
3 2 0
a. 𝑥 = −2; 𝑦 = 6
b. 𝑥 = 2; 𝑦 = 6
c. 𝑥 = −2; 𝑦 = −6
d. 𝑥 = 2; 𝑦 = −6
47. In a class of 40 students, 27 students like Calculus and 25 like Geometry. How many students
like both Calculus and Geometry?
a. 10
b. 14
c. 11
d. 12
48. A class of 40 took examination in Algebra and Trigonometry. If 30 passed Algebra, 36 passed
Trigonometry, and 2 failed in both subjects, the number of students who passed the two subject is:
a. 2
b. 8
c. 28
d. 25
49. The probability for the ECE board examinees from a certain school to pass the Mathematics
subject is 3/7 and that for the communications subject is 5/7. If none of the examinees failed n both
subject and there are 4 examinees who pass both subjects, How many examinees from the school
took the examination?
a. 28
b. 27
c. 26
d. 32

51
50. In a commercial survey involving 1000 persons on brand preferences, 120 were found to prefer
brand x only, 200 persons prefer brand y only, 150 persons prefer brand z only, 370 prefer either
brand x or y but not z, 450 prefer brand y or z but not x, and 370 prefer either brand z or x but not
y, and none prefer all the three brands at a time. How many persons have no brand preference with
any of the three brands?
a. 120
b. 280
c. 70
d. 320

52
PROBLEM SET 8: PERMUTATION,
COMBINATION, PROBABILITY
1. How many permutations can be made out of the letters in the word ISLAND taking four letters
at a time?
a. 360
b. 720
c. 120
d. 24
2. How many 4 digit numbers can be formed without repeating any digit, from the following digits
1, 2, 3, 4 and 6.
a. 150
b. 120
c. 140
d. 130
3. How many permutations can be made out of the letters of the word ENGINEERING?
a. 39,916,800
b. 277,200
c. 55,440
d. 3,326,400
4. How many ways can 3 men and 4 women be seated on a bench if the women are to be together?
a. 720
b. 576
c. 5040
d. 1024
5. In how many ways can 5 people line up to pay their electric bills?
a. 120
b. 1
c. 72
d. 24
6. In how many ways can 5 people line up to pay their electric bills if two particular persons refuse
to follow each other?
a. 120
b. 72
c. 90
d. 140

53
7. How many ways can 7 people be seated at a round table?
a. 5040
b. 120
c. 720
d. 840
8. In how many relative orders can we seat 7 people at a round table with a certain 3 people side
by side.
a. 144
b. 5040
c. 720
d. 1008
9. In how many ways can we seat 7 people in a round table with a certain 3 people not in
consecutive order?
a. 576
b. 3960
c. 5320
d. 689
10. The captain of a baseball team assigns himself to the 4 th place in a battling order. In how many
ways can he assign the remaining places to his eight teammates if just three men are eligible for
the first position?
a. 2160
b. 40320
c. 5040
d. 15120
11. In how many ways can PICE chapter with 15 directors choose a president, a vice-president, a
secretary, a treasurer, and an auditor, if no member can hold more than one position?
a. 630630
b. 3300
c. 360360
d. 3003
12. How many ways can a committee of five may be selected from an organization with 35
members?
a. 324632
b. 425632
c. 125487
d. 326597

54
13. How many line segments can be formed by 13 distinct point?
a. 156
b. 36
c. 98
d. 78
14. In how many wars can a hostess select six luncheon guests from 10 women if she is to avoid
having a particular two of them together at the luncheon?
a. 210
b. 84
c. 140
d. 168
15. A semiconductor company will hire 7 men and 4 women. In how many ways can the company
choose from 9 men and 6 women who qualified for to position?
a. 680
b. 840
c. 480
d. 540
16. How many ways can you invite one or more of five friends to a party?
a. 25
b. 15
c. 31
d. 62
17. A bag contains 4 red balls, 3 green balls, and 5 blue balls. The probability of not getting a red
ball in the first draw is:
a. 2
b. 2/3
c. 1
d. 1/3
18. Which of the following cannot be a probability?
a. 1
b. 0
c. 1/e
d. 0.434343
19. A bag contains 3 white and 5 black balls. If two balls are drawn in succession without
replacement, what is the probability that both ball are black?
a. 5/28
b. 5/16
c. 5/32
d. 5/14

55
20. A bag contains 3 white and 5 red balls. If two balls are drawn at random, find the probability
that both are white.
a. 3/28
b. 3/8
c. 2/7
d. 5/15
21. In Problem 8 – 20, find the probability that one ball is white and the other is red.
a. 15/56
b. 15/28
c. ¼
d. 225/784
22. In problem 8-20, find the probability that all are of the same color.
a. 13/30
b. 14/29
c. 13/28
d. 15/28
23. The probability that both stages of two-stage rocket to function correctly is 0.92. The reliability
of the first stage is 0.97. The reliability of the second stage is:
a. 0.948
b. 0.958
c. 0.968
d. 0.8924
24. Ricky and George each throw two dice. If Ricky gets a sum of 4, what is the probability that
George will get less?
a. ½
b. 5/6
c. 9/11
d. 1/12
25. Two fair dice are thrown. What is the probability that the sum shown on the dice is divisible
by 5?
a. 7/36
b. 1/9
c. 1/12
d. ¼
26. An urn contains 4 black balls and 6 white balls. What is the probability of getting one black
ball and one white ball in two consecutive draws from the urn?
a. 0.24
b. 0.27
c. 0.53

56
d. 0.04
27. If three balls are drawn in succession from 5 white and 6 black balls in a bag, find the
probability that all are of one color, if the first ball is replaced immediately whiel the second is not
replaced before the third draw.
a. 10/121
b. 18/121
c. 28/121
d. 180/14641
28. A first bag contains 5 white balls and 10 black balls and a second bag contains 20 white and
10 black balls. The experiment consists of selecting a bag and then drawing a ball from the selected
bag. Find the probability of drawing a white ball.
a. 1/3
b. 1/6
c. ½
d. 1/18
29. In problem 8 -28, find the possibility of drawing a white ball from the first bag.
a. 5/6
b. 1/6
c. 2/3
d. 1/3
30. If seven coins are tossed simultaneously, find the probability that there will be at least six tails.
a. 33/128
b. 35/128
c. 30/129
d. 37/129
31. If seven coins are tossed simultaneously, find the probability that there will be at least six tails.
a. 2/128
b. 3/128
c. 1/6
d. 2/16
32. A face coin is either head or tail. If three coins are tossed, what is the probability of getting
three tails?
a. 1/8
b. ½
c. ¼
d. 1/6

57
33. The face of a coin is either head or tail. If three coins are tossed, what is the probability of
getting three tails or three heads?
a. 1/8
b. ½
c. ¼
d. 1/6
34. Five fair coins were tossed simultaneously. What is the probability of getting three heads and
two tails?
a. 1/32
b. 1/16
c. 1/8
d. ¼
35. Throw a fair coin five times. What is the probability of getting three head and two tails?
a. 5/32
b. 5/16
c. 1/32
d. 7/16
36. The probability of getting a credit in an examination is 1/3. If three students are selected at
random, what is the probability that at least one of them got a credit?
a. 19/27
b. 8/27
c. 2/3
d. 1/3
37. There are three short questions in mathematics test. For each question, one mark will be
awarded for a correct answer and no mark for a wrong answer. If the probability that Mary
correctly answers a question in a test is 2/3, determine the probability that Mary gets two marks.
a. 4/27
b. 8/27
c. 4/9
d. 2/9
38. A marksman hits 75% of all his targets. What is the probability that he will hit exactly 4 of his
next 10 shots?
a. 0.01622
b. 0.4055
c. 0.004055
d. 0.001622

58
39. A two-digit number is chosen randomly. What is the probability that it is divisible by 7?
a. 7/50
b. 13/90
c. 1/7
d. 7/45
40. One box contains four cards numbered, 1,3, 5 and 6. Another box contains three cards
numbered 2,4, and 7. One card is drawn from each bag. Find the probability that the sum is even.
a. 5/12
b. 3/7
c. 7/12
d. 5/7
41. Two people are chosen randomly from 4 married couples. What is the probability that they
are husband and wife?
a. 1/28
b. 1/14
c. 3/28
d. 1/7
42. One letter is taken from each of the words PARALLEL and LEVEL at random. What is the
probability of getting the same letter?
a. 1/5
b. 1/20
c. 3/20
d. ¾
43. In a shooting game, the probabilities that Botoy and Toto will hit a target is 2/3 and ¾
respectively. What the probability that the target is hit when both shoot at it once?
a. 13/5
b. 5/13
c. 7/12
d. 11/12
44. A standard deck of 52 playing cards is well shuffled. The probability that the first four cards
dealt from the deck will be the four aces is closest to:
a. 4 × 10−6
b. 2 × 10−6
c. 3 × 10−6
d. 8 × 10−6

59
45. A card is chosen from a pack of playing cards. What is the probability that it is either red or a
picture card.
a. 8/13
b. 10/13
c. 19/26
d. 8/15
46. In a poker game consisting of 5 cards, what is the probability of holding 2 aces and 2 queens?
a. 4! / 52!
b. 5/52
c. 33/54145
d. 1264/45685
47. Dennis Rodman sinks 50% of all his attempts. What is the probability that he will make exactly
3 of his next 10 attempts?
a. 1/256
b. 3/8
c. 30/128
d. 15/128
48. There are 10 defectives per 1000 items of a product in a long run. What is the probability that
there is one and only one defective in a random lot of 100.
a. 0.3697
b. 0.3967
c. 0.3796
d. 0.3679
49. The UN forces for Bosnia uses a type of missile that hits the target with a probability of 0.3.
How many missiles should be fire so that there is at least an 80% probability of hitting the target?
a. 2
b. 4
c. 5
d. 3
50. In a dice game, one fair dice is used. The player wins P20.00 if he rolls either 1 or 6. He loses
P10.00 if he turns up any other face. What is the expected winning for one roll of the dice?
a. P40.00
b. P0.00
c. P20.00
d. P10.00

60
Part viii: implicit
differentiation

To this point we’ve done quite a few derivatives, but they have all been derivatives of functions
of the form 𝑦 = 𝑓(𝑥). Unfortunately, not all the functions that we’re going to look at will fall
into this form.

There are actually two solution methods for this problem.

Solution 1:

This is the simple way of doing the problem. Just solve for y to get the function in the form that
we’re used to dealing with and then differentiate.

y = 1/x to y′ = −1/x 2

So, that’s easy enough to do. However, there are some functions for which this can’t be done.
That’s where the second solution technique comes into play.

Solution 2:
In this case we’re going to leave the function in the form that we were given and work with it in
that form. However, let’s recall from the first part of this solution that if we could solve
for y then we will get y as a function of x. In other words, if we could solve for y (as we could in
this case but won’t always be able to do) we get y = y(x). Let’s rewrite the equation to note this.

𝑥𝑦 = 𝑥𝑦(𝑥) = 1

The next step in this solution is to differentiate both sides with respect to x as follows,

𝑑/𝑑𝑥(𝑥𝑦(𝑥)) = 𝑑/𝑑𝑥(1)

The right side is easy. It’s just the derivative of a constant. The left side is also easy, but we’ve
got to recognize that we’ve actually got a product here, the 𝑥 and the 𝑦(𝑥). So, to do the
derivative of the left side we’ll need to do the product rule. By doing this we have,

(1)𝑦(𝑥) + (𝑥)𝑑/𝑑𝑥(𝑦(𝑥)) = 0

61
Now, recall that we have the following notational way of writing the derivative.

𝑑/𝑑𝑥(𝑦(𝑥)) = 𝑑𝑦/𝑑𝑥 = 𝑦′

Using this we get the following,

𝑦 + 𝑥𝑦′ = 0𝑦 + 𝑥𝑦′ = 0

Note that we dropped the (𝑥) on the 𝑦 as it was only there to remind us that the 𝑦 was a function
of 𝑥 and now that we’ve taken the derivative it’s no longer really needed. We just wanted it in
the equation to recognize the product rule when we took the derivative.

So, let’s now recall just what were we after. We were after the derivative 𝑦′ and notice that there
is now a 𝑦′ in the equation. So, to get the derivative all that we need to do is solve the equation
for 𝑦’.

𝑦′ = −𝑦/𝑥

There it is. Using the second solution technique this is our answer. This is not what we got from
the first solution however. Or at least it doesn’t look like the same derivative that we got from
the first solution. Recall however, that we really do know what y is in terms of xx and if we plug
that in, we will get

1 −1
𝑦′ = − 𝑥
= 2
𝑥 𝑥
Examples:
I.
Find the equation of the tangent line 𝑡𝑜𝑥2 + 𝑦2 = 9𝑥2 + 𝑦2 = 9

at the point (2, √5)(2,5).

First note that unlike all the other tangent line problems we’ve done in previous sections we need
to be given both the 𝑥 and the 𝑦 values of the point. Notice as well that this point does lie on the
graph of the circle (you can check by plugging the points into the equation) and so it’s okay to
talk about the tangent line at this point.

Recall that to write down the tangent line all we need is the slope of the tangent line and this is
nothing more than the derivative evaluated at the given point. We’ve got the derivative from the
previous example so all we need to do is plug in the given point.

𝑚 = 𝑦′ ∣∣ 𝑥 = 2, 𝑦 = √5 = −2√5𝑚 = 𝑦′|𝑥 = 2, 𝑦 = 5 = −25

The tangent line is then.

62
𝑦 = √5 − 2√5(𝑥 − 2)
𝒂 𝑥 3 𝑦 5 + 3𝑥 = 8𝑦 3 + 1𝑥 3 𝑦 5 + 3𝑥 = 8𝑦 3 + 1
First differentiate both sides with respect to 𝑥 and remember that each 𝑦 is really 𝑦(𝑥) we just
aren’t going to write it that way anymore. This means that the first term on the left will be a
product rule.
3𝑥 2 𝑦 5 + 5𝑥 3 𝑦 4 𝑦′ + 3 = 24𝑦 2 𝑦′
Now all that we need to do is solve for the derivative, 𝑦′. This is just basic solving algebra that
you are capable of doing. The main problem is that it’s liable to be messier than what you’re
used to doing. All we need to do is get all the terms with 𝑦′ in them on one side and all the terms
without 𝑦’ in them on the other.
Then factor 𝑦′ out of all the terms containing it and divide both sides by the “coefficient” of
the 𝑦′. Here is the solving work for this one,

3𝑥 2 𝑦 5 + 3 = 24𝑦 2 𝑦′ − 5𝑥 3 𝑦 4 𝑦′
3𝑥 2 𝑦 5 + 3 = (24𝑦 2 − 5𝑥 3 𝑦 4 )𝑦′
𝑦′ = (3𝑥 2 𝑦 5 + 3)/(24𝑦 2 − 5𝑥 3 𝑦 4 )
II.

𝑎.(5𝑥 3 − 7𝑥 + 1)5[𝑓(𝑥)]5[𝑦(𝑥)]5

𝑑/𝑑𝑥[(5𝑥 3 − 7𝑥 + 1)5] = 5(5𝑥 3 − 7𝑥 + 1)4 (15𝑥2 − 7)

𝑑/𝑑𝑥[𝑓(𝑥)]5 = 5[𝑓 (𝑥 )]4 𝑓′(𝑥)


𝑑/𝑑𝑥[𝑦(𝑥)]5 = 5[𝑦(𝑥 )]4 𝑦′(𝑥)

𝑏.𝑠𝑖𝑛(3 − 6𝑥), 𝑠𝑖𝑛(3 − 6𝑥)

𝑑/𝑑𝑥[𝑠𝑖𝑛(3 − 6𝑥)] = −6𝑐𝑜𝑠(3 − 6𝑥)

𝑑/𝑑𝑥[𝑠𝑖𝑛(𝑦(𝑥))] = 𝑦′(𝑥)𝑐𝑜𝑠(𝑦(𝑥))

Exercises:
1. Assume that y is a function of x Find y' = 𝑑𝑦/𝑑𝑥 for 𝑥 3 + 𝑦 3 = 4
2. Assume that y is a function of x Find y' = 𝑑𝑦/𝑑𝑥 for (𝑥 − 𝑦)2 = 𝑥 + 𝑦 − 1
3. Assume that y is a function of x Find y' = 𝑑𝑦/𝑑𝑥 for y = 𝑥 2 𝑦 3 + 𝑥 3 𝑦 2
4. Assume that y is a function of x Find y' = 𝑑𝑦/𝑑𝑥 for (𝑥 − 𝑦 3 )/(𝑦 + 𝑥 2 ) = 𝑥 + 2
Answers:
1. Begin with 𝑥 3 + 𝑦 3 = 4 Differentiate both sides of the equation, getting

63
𝐷 ( 𝑥3 + 𝑦3 ) = 𝐷 ( 4 )
𝐷( 𝑥 3 ) + 𝐷( 𝑦 3 ) = 𝐷 ( 4 ) ,
Remember to use the chain rule on D( 𝑦 3 )
3𝑥 2 + 3𝑦 2 𝑦′ = 0
so that (Now solve for y')
3𝑦 2 𝑦′ = − 3𝑥 2
𝑦′ = (−3𝑥 2 )/(3𝑦 2 )
= 𝑥 2 /𝑦 2
2. Begin with (𝑥 − 𝑦)2 = 𝑥 + 𝑦 – 1, Differentiate both sides of the equation, getting
𝐷 (𝑥 − 𝑦 )2 = 𝐷 ( 𝑥 + 𝑦 − 1 )
𝐷 (𝑥 − 𝑦 )2 = 𝐷 ( 𝑥 ) + 𝐷 ( 𝑦 ) − 𝐷 ( 1 )
(Remember to use the chain rule on 𝐷 (𝑥 − 𝑦)2
2(𝑥 − 𝑦) (1 − 𝑦′) = 1 + 𝑦′
so that (Now solve for y')
2 (𝑥 − 𝑦) − 2 (𝑥 − 𝑦) 𝑦′ = 1 + 𝑦′
− 2(𝑥 − 𝑦) 𝑦′ − 𝑦′ = 1 − 2 (𝑥 − 𝑦)
(Factor out y')
𝑦′[−2(𝑥 − 𝑦) − 1] = 1 − 2 (𝑥 − 𝑦)
𝑦′ = (1 − 2(𝑥 − 𝑦))/(−2(𝑥 − 𝑦) − 1)
= (2𝑦 + 2𝑥 + 1)/(2𝑦 − 2𝑥 − 1)
3. Begin with 𝑦 = 𝑥 2 𝑦 3 + 𝑥 3 𝑦 2 Differentiate both sides of the equation, getting
𝐷(𝑦) = 𝐷 ( 𝑥 2 𝑦 3 + 𝑥 3 𝑦 2 )
𝐷(𝑦) = 𝐷 ( 𝑥 2 𝑦 3 ) + 𝐷 ( 𝑥 3 𝑦 2 )
𝑦 ′ = 3𝑥 2 𝑦 2 𝑦 ′ + 2𝑥𝑦 3 + 2𝑥 3 𝑦 𝑦′ + 3𝑥 2 𝑦 2
so that (Now solve for y')
𝑦′ − 3𝑥 2 𝑦 2 𝑦′ − 2𝑥 3 𝑦𝑦′ = 2𝑥𝑦 3 + 3𝑥 2 𝑦 2
(Factor out y')
𝑦′ [ 1 − 3𝑥 2 𝑦 2 − 2𝑥 3 𝑦 ] = 2𝑥𝑦 3 + 3𝑥 2 𝑦 2
𝑦′ = (2𝑥𝑦 3 + 3𝑥 2 𝑦 2 )/(1 − 3𝑥 2 𝑦 2 − 2𝑥 3 𝑦)
4. Clear the fraction by multiplying both sides of the equation by 𝑦 + 𝑥 2 getting
𝑥 − 𝑦 3 = 𝑥𝑦 + 2𝑦 + 𝑥 3 + 2𝑥 2
Now differentiate both sides of the equation, getting
𝐷 ( 𝑥 − 𝑦 3 ) = 𝐷 ( 𝑥𝑦 + 2𝑦 + 𝑥 3 + 2𝑥 2 )
𝐷 ( 𝑥 ) − 𝐷 (𝑦 3 ) = 𝐷 ( 𝑥𝑦 ) + 𝐷( 2𝑦 ) + 𝐷( 𝑥 3 ) + 𝐷( 2𝑥 2 )

64
Remember to use the chain rule on 𝐷 (𝑦 3 )
1 − 3 𝑦 2 𝑦′ = ( 𝑥𝑦′ + (1)𝑦 ) + 2𝑦′ + 3𝑥 2 + 4𝑥
so that (Now solve for y')
1 − 𝑦 − 3𝑥 2 − 4𝑥 = 3𝑦 2 𝑦′ + 𝑥𝑦′ + 2 𝑦′
(Factor out y')
1 − 𝑦 − 3𝑥 2 − 4𝑥 = (3𝑦 2 + 𝑥 + 2) 𝑦′ ,
𝑦′ = (1 − 𝑦 − 3𝑥 2 − 4𝑥)/(3𝑦 2 + 𝑥 + 2)

65
Problem set 9: complex numbers,
vectors, elements,
1. In the complex number 3 + 4𝑖, the absolute value is:
a. 10
b. 7.211
c. 5
d. 5.689
2. In the complex number 8 − 2𝑖, the amplitude is:
a. 104.04°
b. 14.04°
c. 345.96°
d. 165.96°
3. (6 cis 120°) (4 cis 30°) is equal to?
a. 10 cis 150°
b. 24 cis 150°
c. 10 cis 90°
d. 24 cis 90°
30 𝑐𝑖𝑠 80°
4. 10 𝑐𝑖𝑠 50° is equal to:

a. 20 cis 30°
b. 3 cis 130°
c. 3 cis 30°
d. 20 cis 130°
5. The value of 𝑥 + 𝑦 in the complex equation 3 + 𝑥𝑖 = 𝑦 + 2𝑖 is:
a. 5
b. 1
c. 2
d. 3
6. Multiply (3 − 2𝑖 )(4 + 3𝑖 ).
a. 12 + 𝑖
b. 18 + 𝑖
c. 6+𝑖
d. 20 + 𝑖

66
4+3𝑖
7. Divide 2−𝑖
11+10𝑖
a. 5
b. 1+2i
c. 1
d. -1
8. Find the value of 𝑖 ′9 .
a. 𝑖
b. −𝑖
c. 1
d. -1
9. Simplify 𝑖 ′997 + 𝑖 ′999, where i is an imaginary number
a. 1+𝑖
b. 𝑖
c. 1−𝑖
d. 0
3
10. Expand (2 + √−9)
a. 46 + 9𝑖
b. 46 − 9𝑖
c. −46 − 9𝑖
d. −46 + 9𝑖
11. Write -4 + 3i in polar form.
a. 5∠36.87°
b. 5∠216.87°
c. 5∠323.13°
d. 5∠143.13°
12. Simplify: 𝑖 30 − 2𝑖 25 + 3𝑖 17.
a. i+1
b. -1 – 2i
c. -1 + i
d. -1 + 5i
13. Evaluate the value of √−10 × √−7
a. Imaginary
b. −√70
c. √17
d. √70

67
3
14. Perform the indicated operation: √−9 × √−343
a. 21
b. 21i
c. -21i
d. -21
15. What is the quotient when 4 + 8𝑖 is divided by 𝑖 3 ?
a. 8 + 4𝑖
b. −8 + 4𝑖
c. 8 − 4𝑖
d. −8 − 4𝑖
16. What is the exponential form of the complex number 4 + 3𝑖?
a. 5 𝑒 𝑖 53.13°
b. 5 𝑒 𝑖 36.87°
c. 7 𝑒 𝑖 53.13°
d. 7 𝑒 𝑖 36.87°
17. What is the algebraic form of the complex number 13 𝑒 𝑖 67.38° ?
a. 12 + 5i
b. 5 – 12i
c. 12 – 5i
d. 5 + 12i
18. Solve for x that satisfy the equation 𝑥 2 + 36 = 9 − 2𝑥 2
a. ±6𝑖
b. ±3𝑖
c. 9𝑖
d. -9i
19. Evaluate ln (5 + 12i).
a. 2.565 + 1.176 𝑖
b. 2.365 − 0.256 𝑖
c. 5.635 + 2.112 𝑖
d. 3.214 − 1.254
20. Add the given vectors: (-4, 7) + (5, -9)
a. (1,16)
b. (1, -2)
c. (9, 2)
d. (1, 2)

68
21. Find the length of the vector (2, 1, 4).
a. √17
b. √21
c. √20
d. √19
22. Find the length of vector (2, 4, 4).
a. 8.75
b. 6.00
c. 7.00
d. 5.18
23. What is the magnitude of the vector 𝐹 = 2𝑖 + 5𝑗 + 6𝑘?
a. 6.12
b. 7.04
c. 6.18
d. 8.06
24. Find the value of x in the complex equation: (𝑥 + 𝑦𝑖 )(1 − 2𝑖 ) = 7 − 4𝑖.
a. 1
b. 3
c. 4
d. 2
25. A statement the truth of which is admitted without proof is called:
a. An axiom
b. A postulate
c. A theorem
d. A corollary
26. In a proportion of four quantities, the first and the fourth terms are referred to as the:
a. Mean
b. Denominators
c. Extremes
d. Numerators
27. Convergent series is a sequence of decreasing numbers or when the succeeding term is ______
than the preceding term
a. Ten times more
b. Greater
c. Equal
d. Lesser

69
28. It is the characteristic of a population which is measurable.
a. Frequency
b. Distribution
c. Sample
d. Parameter
29. The quartile deviation is a measure of:
a. Division
b. Central tendency
c. Certainty
d. Dispersion
30. In complex algebra, we use a diagram to represent a complex plane commonly called:
a. De Moivre’s Diagram
b. Funicular Diagram
c. Argand Diagram
d. Venn Diagram
31. A series of number which are perfect square numbers (i.e. 1, 4, 9, 16, … ) is called:
a. Fourier Series
b. Fermat’s number
c. Euler’s number
d. Fibonacci Numbers
32. A sequence of numbers where every term is obtained by adding all the preceding terms such
as 1, 5, 14, 39, … is called:
a. Triangular number
b. Pyramidal Number
c. Tetrahedral Number
d. Euler’s Number
33. The graphical representation of the commulative frequency distribution in a set of statistical
data is called:
a. Ogive
b. Histogram
c. Frequency Polyhedron
d. Mass diagram
34. A sequence of numbers where the succeeding term is greater than the preceding term is called:
a. Dissonant series
b. Convergent series
c. Isometric series
d. Divergent series

70
35. The number 0.123123123 … is
a. Irrational
b. Surd
c. Rational
d. Transcendental
36. An array of 𝑚 × 𝑛 quantities which represent a single number system composed of elements
in rows and column is known as:
a. Transpose of matrix
b. Determinant
c. Co-factor of a matrix
d. Matrix
37. The characteristic is equal to the exponent of 10, when the number is written in:
a. Exponential
b. Scientific notation
c. Logarithmic
d. Irrational
38. Terms that differ only in numeric coefficients are known as:
a. Unequal terms
b. Unlike terms
c. Like terms
d. Equal term
39. _________ is a sequence of terms whose reciprocals form an arithmetic progression.
a. Geometric progression
b. Harmonic progression
c. Algebraic progression
d. Ratio and proportion
40. The logarithm of a number to the bas e (2.718281828 …) is called:
a. Naperian logarithm
b. Characteristic
c. Mantissa
d. Briggsian Logarithm
41. The ratio or product of two expressions in direct or inverse relation with each other is call:
a. Ratio and proportion
b. Constant of variation
c. Means
d. Extremes

71
42. In any square matrix, when the elements of any two rows are exactly the same the determinant
is:
a. Zero
b. Positive integer
c. Negative integer
d. Unity
43. Two or more equation are equal if and only if they have the same:
a. Solution set
b. Degree
c. Order
d. Variable set
44. What is a possible outcome of an experiment called?
a. A sample space
b. A random point
c. An even
d. A finite set
45. If the roots of an equation are zero, then they are classified as:
a. Trivial Solution
b. Extraneous Roots
c. Conditional Solutions
d. Hypergolic Solutions
46. A complex number associated with a phase-shifted sine wave in a polar form whose magnitude
is in RMS and angle is equal to the angle of the phase-shifted sine wave is known as:
a. Argand’s number
b. Imaginary number
c. Phasor
d. Real number
47. In a raw data, the term, which occurs most frequently, is known as:
a. Mean
b. Median
c. Mode
d. Quartile
48. Infinity minus infinity is:
a. 1
b. Infinity
c. Zero
d. Indeterminate

72
49. Any number divided by infinity is equal to:
a. 1
b. Infinity
c. Zero
d. Indeterminate
50. The term in between any to terms of an arithmetic progression is called:
a. Arithmetic mean
b. Median
c. Middle terms
d. Mean
51. Any equation which, because of some mathematical process, has acquired an extra root is
sometimes called a:
a. Redundant equation
b. Literal equation
c. Linear equation
d. Defective equation
52. A statement that one mathematical expression is greater than or less than another is called:
a. Inequality
b. Non-absolute condition
c. Absolute Condition
d. Conditional Expression
53. A relation in which every ordered pair (x, y) has one and only one value of y that corresponds
to the values of x, is called:
a. Function
b. Range
c. Domain
d. Coordinates
54. An equation in which a variable appears under the radical sign is called:
a. Literal equation
b. Radical equation
c. Irradical equation
d. Irrational equation
55. The number of favorable outcomes divided by the number of possible outcomes is:
a. Permutations
b. Probability
c. Combination
d. Change

73
56. Two factors are considered essentially the same if:
a. One is merely the negative of the other
b. One is exactly the same as other
c. Both of them are negative
d. Both of them are positive
57. An integer is said to be prime if:
a. It is factorable by any value
b. It is an odd integer
c. It has no other integer as a factor except itself or 1
d. It is an even integer
58. Equations in which the members are equal for all permissible values of unknowns are called:
a. A conditional equation
b. An identity
c. A parametric equation
d. A quadratic equation
59. Equation which satisfy only for some values of unknown are called:
a. A conditional equation
b. An identity
c. A parametric equation
d. A quadratic equation
60. The logarithm of 1 to any base is:
a. Indeterminate
b. Zero
c. Infinity
d. One

74
Problem set 10: angles, trigonometric
identities and equations
1.Find the supplement of an angle whose compliment is 62°
A. 28° B. 118° C.152° D. none of these
2. A certain angle has a supplement five times its compliment, Find the angle.
A. 67.5° B. 157.5° C. 168.5° D. 186°
3. The sum of the two interior angles of the triangle is equal to the third angle and the difference
of the two angles is equal to 2/3 of the third angle. Find the third angle.
A. 15° B. 75° C. 90° D. 120°
1
4. The measure of 12 revolutions counter-clockwise is:

A. 540° B. 520° C.+90° D.-90°


5. The measure of 2.25 revolutions counterclockwise is:
A. -835 degrees B. -810 degrees C. 805 degrees D. 810 degrees
6. Solve for θ: sin 0 − sec θ + csc θ − tan θ = −0.0866
A. 40° B. 41° C. 47° D. 43°
7. What are the exact values of the cosine and tangent trigonometric functions of acute angle A,
given that sin A = 3/7?
A. cos 𝐴 = 7/2 √10; tan 𝐴 = 2√10/3
B. cos 𝐴 = 2 √10/7; tan 𝐴 = 3√10/20
C. cos 𝐴 = 2 √10/3; tan 𝐴 = 7/2√10
D. cos 𝐴 = 2 √10/3; tan 𝐴 = 7√10/20
8. Given three angles A, B, and C whose sum is 180°. If the tan 𝐴 + tan 𝐵 + tan 𝐶 = 𝑥, find the
value of tan 𝐴 x tan 𝐵 x tan 𝐶.
A. 1 − 𝑥 B. √𝑥 C. 𝑥/2 D. 𝑥
9. What is the sine of 820°?
A. 0984 B. -0.866 C. 0.866 D. -0.500
10. Csc 270° = ?

A. −√3 B. -1 C. √3 D. 1

75
11. If coversine θ is 0.134, find the value θ.
A. 60° B. 45° C. 30° D. 20°
12. Solve for cos 72° if the given relationship is cos 2 𝐴 = 2 cos 2 𝐴 − 1
A. 0.309 B. 0.258 C. 0.268 D. 0.315
13. If sin 3𝐴 = cos 6𝐵 then:
A. 𝐴 + 𝐵 = 180° B. 𝐴 + 2𝐵 = 30° C. 𝐴 − 2𝐵 = 30°
D. 𝐴 + 𝐵 = 30
14. Find the value of ssin(arccos 15/17).
A. 8/17 B. 17/ 9 C. 8/21 D. 8/9
15. Find the value cos[arcsin(1/3) + arctan(2/√5)]

A. (2/9)(1 + √10) B. (2/9)(√10 − 11) C. (2/9)(√10 + 1) D. (2/9)(√10 − 1)


16. If sin 40 ° + sin 20° = sin θ, find the value of θ.
A. 20° B. 80° C. 120° D. 60°
17. How many different values of x from 0° to 180° for the equation (2 sin 𝑥 − 1)(cos 𝑥 + 1) =
0?
A. 3 B. 0 C. 1 D. 2
18. For what value of θ (less that 2𝜋) will the following equation be satisfied?
A. 𝜋 B. 𝜋/4 C. 3𝜋/2 D. 𝜋/2
19. Find the value of x in the equation csc 𝑥 + cot 𝑥 = 3.
A. 𝜋/4 B. 𝜋/3 C. 𝜋/2 D. 𝜋/5
20. If sec2 𝐴 is 5/2, the quantity 1 − sec2 𝐴 is equivalent to:
A. 2.5 B. 0.6 C. 1.5 D. 0.4
21. Find sin 𝑥 if 2 sin 𝑥 + 3 cos 𝑥 − 2 = 0
A. 1 & -5/13 B. -1 & 5/13 C. 1 & 5/13 D. -1 & -5/13
22. If sin 𝐴 = 4/5, 𝐴 in quadrant II, sin 𝐵 = 7/25, 𝐵 in quadrant I, find sin(𝐴 + 𝐵).
A. 3/5 B. 2/5 C. 3/4 D. 4/5
23. If sin 𝐴 = 2.571𝑥, cos 𝐴 = 3.06𝑥, and sin 2𝐴 = 3.939𝑥, find the value of 𝑥.
A. 0.350 B. 0.250 C. 0.100 D. 0.150

76
24. If cos θ = √3/2, then find the value of 𝑥 if 𝑥 = 1 − tan2 θ.
A. -2 B. -1/3 C. 4/3 D. 2/3
25. If sin θ − cos θ = −1/3, what is the value of sin 2θ?
A. 1/3 B. 1/9 C. 8/9 D. 4/9
26. If x cos θ + y sin θ = 1 and 𝑥 sin θ − y cos θ = 3, what is the relationship between 𝑥 and 𝑦?
A. 𝑥 2 + 𝑦 2 = 20 B. 𝑥 2 + 𝑦 2 = 5 C. 𝑥 2 + 𝑦 2 = 16 𝑥 2 + 𝑦 2 = 10
27. If sin 𝑥 + 1/ 𝑠𝑖𝑛 𝑥 = √2, then sin2 𝑥 is equal to:
A. √2 B. 1 C. 2 D. 0
28. The equation 2 sin θ + 2 cos θ − 1 = √3 is:
A. an identity B. a parametric equation C. a conditional equation
D. a quadratic equation
sin 𝑥 tan 𝑦
29. If 𝑥 + 𝑦 = 90°, then sin 𝑦 tan 𝑥 is equal to:

A. tan 𝑥 B. cos 𝑥 C. cot 𝑥 D. sin 𝑥


30. If cos θ = x/2 then 1 − tan2 θ is equal to:
A. (2𝑥 2 + 4)/𝑥 2 B. (4 − 2𝑥 2 )/𝑥 2 C. (2𝑥 2 − 4)/𝑥 D. (2𝑥 2 − 4)/𝑥 2
31. Find the value in degrees of arccos(tan 24°)
A. 61.48 B. 62.35 C. 63.56 D. 60.84
32. arctan[2 cos(arcsin √3 /2)] is equal to:
A. 𝜋/3 B. 𝜋/4 C. 𝜋/6 D. 𝜋/2
33. Solve for x in the equation arctan(2𝑥 ) + arctan(𝑥 ) = 𝜋/4
A. 0.821 B. 0.218 C. 0.281 D. 0.182
34. Solve for x from the given trigonometric equation: arctan(1 − 𝑥 ) + arctan(1 + 𝑥 ) =
arctan 1/8
A. 4 B. 6 C. 8 D. 2
35. Solve for y if 𝑦 = (1/ sin 𝑥 − 1/ tan 𝑥 )(1 + cos 𝑥 )
A. sin 𝑥 B. cos 𝑥 C. tan 𝑥 D. sec2 𝑥
36. Solve for 𝑥: 𝑥 = (tan θ + cot θ)2 sec2 θ − tan2 θ.
A. sin θ B. cos θ C. 1 D. 2

77
37. Solve for 𝑥: 𝑥 = 1 − (sin θ −cos θ)2 .
A. sin θ cos θ B. −2 cos θ C. cos 2θ D. sin 2θ
38. Simplify cos 4 θ − sin4 θ.
A. 2 B. 1 C. 2 sin2 θ + 1 D. 2 cos 2 θ − 1
1−tan2 𝑎
39. Solve for 𝑥: 𝑥 = 1+tan2 𝑎

A. cos 𝑎 B. sin 2𝑎 C. cos 2𝑎 D. sin 𝑎


40. Which of the following in different from the others?
A. 2 cos 2𝑥 − 1 B. cos 4𝑥 − sin 4𝑥 C. cos 3𝑥 − sin 3𝑥 D. 1 − 2 sin 2𝑥
41. Find the value of 𝑦: 𝑦 = (1 + cos 2θ) tan θ.
A. cos θ B. sin θ C. sin 2θ D. cos 2θ
42. The equation 2 sinh 𝑥 cosh 𝑥 is equal to:
A. 𝑒 𝑥 B. 𝑒 −𝑥 C. sin2 θ sec2 θ D. cos 2 θ
43. Simplifying the equation sin2 θ(1 + cot −1 θ) gives:
A. 1 B. sin2 θ C. sin2 θ sec2 θ D. cos 2 θ
44. Find the value of sin(90° + 𝐴)
A. cos 𝐴 B. − cos 𝐴 C. sin 𝐴 D. − sin 𝐴
45. Which of the following expressions is equivalent to sin 2 θ
A. 2 sin θ cos θ B. sin2 θ sec2 θ C. cos 2 θ − sin2 θ D. sin θ cos θ
46. If tan θ = 𝑥 2 , which of the following is incorrect?

A. sin θ = 1√1 + 𝑥 4 B. sec θ = √1 + 𝑥 4

C. cos θ = 1 /√1 + 𝑥 4 D. csc θ = √1 + 𝑥 4 /𝑥 2


47. In an isosceles right triangle, the hypotenuse is how much longer than its sides?
A. 2 times B. √2 times C. 1.5 times D. none of these
48. Find the angle in mils subtended by a line 10 yards long at a distance of 5000 yards.
A. 2.5 mils B. 2 mils C. 4 mils D. 1mil
49. The angle or inclination of ascend of a road having 8.25% grade is _____ degrees.
A. 5.12 degrees B. 4.72 degrees C. 1.86 degrees D. 4.27 degrees
50. The sides of a right triangle is an arithmetic progression whose common difference if 6 cm. Its
area is:
A. 216 cm2 B. 270 cm2 C. 360 cm2 D. 144 cm2

78
PART IX: HIGHER
ORDER DERIVATIVES

The second derivative, or second order derivative, is the derivative of the derivative of a
function. The derivative of the function may be denoted by ‘ and its double (or "second")
derivative is denoted by ‘’ This is read as " double prime of ", or "The second derivative of ".
Because the derivative of function is defined as a function representing the slope of function, the
double derivative is the function representing the slope of the first derivative function.
Furthermore, the third derivative is the derivative of the derivative of the derivative of a function,
which can be represented by ‘’’ This is read as " triple prime of ‘’ or "The third derivative of " .
This can continue as long as the resulting derivative itself is differentiable, with the fourth
derivative, the fifth derivative, and so on. Any derivative beyond the first derivative can be
referred to as a higher order derivative.

Examples:
1. Determine the fourth derivative of h(t) = 3t7 − 6t4 + 8t 3 − 12t + 18
The first derivative is then,
h′(t) = 21t 6 − 24t3 + 24t2 − 12
The second derivative is,
h″(t) = 126t5 − 72t2 + 48t
The third derivative is,
h‴(t) = 630t4 − 144t + 48

The fourth, and final derivative for this problem is,


ℎ(4)(𝑡) = 2520𝑡 3 − 144
2. Determine the fourth derivative of 𝑉(𝑥) = 𝑥 3 − 𝑥 2 + 𝑥 − 1
The first derivative is then,

79
𝑉′(𝑥) = 3𝑥 2 − 2𝑥 + 1

The second derivative is,

𝑉″(𝑥) = 6𝑥 − 2

The third derivative is,

𝑉‴(𝑥) = 6

The fourth, and final derivative for this problem, is,


𝑉(4)(𝑥 ) = 0

3. Determine the fourth derivative of 𝑦 = 𝑒 − 5𝑧 + 8𝑙𝑛(2𝑧 2 )𝑦 = 𝑒 − 5𝑧 + 8𝑙𝑛 (2𝑧 4 )


Not much to this problem other than to take four derivatives so each step will show each
successive derivative until we get to the fourth.
The first derivative is then,
𝑑𝑦𝑑𝑧 = −5𝑒 − 5𝑧 + 8(8𝑧 3 2𝑧 4 ) = −5𝑒 − 5𝑧 + 32𝑧 − 1

The second derivative is,

𝑑 2 𝑦𝑑𝑧 2 = 25𝑒 − 5𝑧 − 32𝑧 − 2

The third derivative is,

𝑑 3 𝑦𝑑𝑧 3 = −125𝑒 − 5𝑧 + 64𝑧 − 3

The fourth, and final derivative for this problem is,


𝑑 4 𝑦𝑑𝑧 4 = 625𝑒 − 5𝑧 − 192𝑧 − 4

4. Determine the second derivative of 𝑧 = 𝑙𝑛(7 − 𝑥3)


Not much to this problem other than to take two derivatives so each step will show each
successive derivative until we get to the second. The first derivative is then,
3𝑥 2
𝑑𝑧/𝑑𝑥 = − − 𝑥3
7
Do not forget that often we will end up needing to do a quotient rule in the second derivative
even though we did not need to do that in the first derivative. The second derivative is then,
𝑑 2 𝑧𝑑𝑥 2 = −6𝑥(7 − 𝑥 3 ) − (−3𝑥 2 )(−3𝑥 2 )(7 − 𝑥 3 )2 = −42𝑥 − 3𝑥4(7 − 𝑥 3 )2
Exercises:
1. Determine the second derivative of 𝑔(𝑥) = 𝑠𝑖𝑛(2𝑥 3 − 9𝑥)
2. Determine the second derivative of 𝐻(𝑡) = cos 2 (7𝑡)

80
3. Find the first four derivatives for each of the following. 𝑦 = 𝑐𝑜𝑠𝑥
4. Given the function y=(2x+1)3(x−1). Find all derivatives of the nth order from 𝑛 = 1 to 𝑛 = 5.

Answers:
1. Not much to this problem other than to take two derivatives so each step will show each
successive derivative until we get to the second. The first derivative is then,
𝑔′(𝑥) = (6𝑥 2 − 9)𝑐𝑜𝑠(2𝑥 3 − 9𝑥)
𝑔′′(𝑥) = 12𝑥𝑐𝑜𝑠(2𝑥 3 − 9𝑥) − (6𝑥 2 − 9)2𝑠𝑖𝑛(2𝑥 3 − 9𝑥)
2. Not much to this problem other than to take two derivatives so each step will show each
successive derivative until we get to the second. The first derivative is then,
𝐻′(𝑡) = −14𝑐𝑜𝑠(7𝑡)𝑠𝑖𝑛(7𝑡)
𝐻′′(𝑡) = 98𝑠𝑖𝑛(7𝑡)𝑠𝑖𝑛(7𝑡) − 98𝑐𝑜𝑠(7𝑡)𝑐𝑜𝑠(7𝑡)
= 98 sin2 (7𝑡) − 98 cos 2 (7𝑡)
3. Again, let’s just do some derivatives.
𝑦 = 𝑐𝑜𝑠𝑥
𝑦′ = −𝑠𝑖𝑛𝑥
𝑦′′ = −𝑐𝑜𝑠𝑥
𝑦′′′ = 𝑠𝑖𝑛𝑥
𝑦(4) = 𝑐𝑜𝑠𝑥
4. First we convert the given function into a polynomial:
𝑦 = (2𝑥 + 1)3 (𝑥 − 1)
= (8𝑥 3 + 12𝑥 2 + 6𝑥 + 1)(𝑥 − 1)
= 8𝑥 4 + 12𝑥 3 + 6𝑥 2 + 𝑥 − 8𝑥 3 − 12𝑥 2 − 6𝑥 − 1
= 8𝑥 4 + 4𝑥 3 − 6𝑥 2 − 5𝑥 − 1
Now we successively calculate the derivatives from 1st to 5th order:
𝑦 ′ = (8𝑥 4 + 4𝑥 3 − 6𝑥 2 − 5𝑥 − 1)′ = 32𝑥 3 + 12𝑥 2 − 12𝑥 − 5
𝑦′′ = (𝑦′)′ = (32𝑥 3 + 12𝑥 2 − 12𝑥 − 5)′ = 96𝑥 2 + 24𝑥 − 12
𝑦′′′ = (𝑦′′)′ = (96𝑥 2 + 24𝑥 − 12)′ = 192𝑥 + 24
𝑦(4) = (𝑦′′′)′ = (192𝑥 + 24)′ = 192,
𝑦(5) = (𝑦(4))′ = (192)′ = 0

81
Problem set 11: triangles, angle of
elevation & depression
1. The hypotenuse of a right triangle is 34 cm. Find the length of the shortest leg if it is 14 cm
shorter than the other leg.
A. 15 cm B. 16 cm C. 17 cm D. 18 cm
2. A truck travels from point M northward for 30 min, then eastward for one hour, the shifted N
30° W. If the constant speed is 40 kph, how far directly from M, in km will be it after 2 hours?
A. 43.5 B. 45.2 C. 47.9 D. 41.6
3. Two sides of a triangle measures 6 cm and 8 cm and their included angle is 40°. Find the third
side.
A. 5.144 cm B. 5.263 cm C. 4.256 cm D. 5.645 cm
4. Given a triangle: C = 100°, a = 15, b = 20. Find c:
A. 34 B. 27 C. 43 D. 35
5. Given angle A = 32°, angle B = 70°, and side c = 27 units. Solve for side a of the triangle.
A. 24 units B. 10 units C. 14.63 units D. 12 units
6. In a triangle, find the side c if angle C = 100°, side b = 20, and side a = 15.
A. 28 B. 27 C. 29 D. 26
7. In triangle ABC, A = 45 degrees and angle C = 70 degrees. The side opposite angle C is 40 m
long. What is the side opposite angle A?
A. 29.10 meters B. 32.25 meters C. 30.10 meters D. 31.25 meters
8. Two sides of triangle are 50 m and 60 m long. The angle included between these sides is 30
degrees, what is the interior angle (in degrees) opposite the longest side?
A. 92.74 B. 93.74 C. 94.74 D. 91.74
9. The sides of a triangle ABC are AB = 15 cm, BC = 18 cm, and CA = 24 cm. Determine the
distance from the point of intersection of the angular bisectors to side AB.
A. 5.21 cm B. 3.78 cm C. 4.73 cm D. 6.25 cm
10. If AB = 15 cm, BC = 18 m and CA = 24 m, find the point intersection of the angular bisectors
from the vertex C.
A. 11.3 B. 12.1 C. 13.4 D. 14.3
11. In triangle ABC, angle C = 70 degrees; angle A = 45 degrees; AB = 40 m. What is the length
of the median drawn from vertex A side BC?
A. 36.8 meters B. 37.1 meters C. 36.3 meters D. 37.4 meters

82
12. The are of the triangle whose angles are 61°9’32’’, 34°14’46’’, and 84°35’42’’ is 680.60. The
length of the longest side is:
A. 35.53 B. 54.32 C. 52.43 D. 62.54
13. Given a triangle ABC whose angles are A = 40°, B = 95° and side b = 30 cm. Find the length
of the bisector of angle C.
A. 21.74 cm B. 22.35 cm C. 20.45 cm D. 20.98 cm
14. The sides of a triangular lot are 130 m, 180 m, and 190 m. The lot is to be divided by a line
bisecting the longest side and drawn from opposite vertex. The length of the dividing line is:
A. 100 meters B. 130 meters C. 125 meters D. 115 meters
15. From a point outside of an equilateral triangle, the distance to the vertices are 10m, 10m, and
18m. Find the dimension of the triangle,
A. 25.63 B. 45.68 C. 19.94 D. 12.25
16. Points A and B 1000m apart are plotted on a straight highway running East and West. From A,
the bearing of a tower C is 32 degrees N of W and from B the bearing of C is 26 degrees N to E.
Approximate the shortest distance of tower C to the highway.
A. 254 meters B. 274 meters C. 284 meters D.294 meters
17. An airplane leaves an aircraft carrier and flies South at 350 mph. The carrier travels S 30° E at
25 mph. If the wireless communication range of the airplane is 700 miles, when will it lose contact
with the carrier?
A. after 4.36 hours B. after 5.57 hours C. after 2.13 hours D. after 4.54 hours
18. A statue 2 meters high stands on a column that is 3 meters high. An observer in level with the
top of the statue observed that the column and the statue subtend the same angle. How far is the
observer from the statue?

A. 5√2 meters B. 2√5 meters C. 20 meters D. √10 meters


19. From the top of a building 100 m high the angle of depression of a point A due East of its 30°.
From a point B due South of the building, the angle of elevation of the top is 60°. Find the distance
AB.
A. 100 + 3√30 B. 200√30 C. 100√30/3 D. 100√3/30
20. An observer found the angle of elevation of the top of the tree to be 27°. After moving 10m
closer (on the same vertical and horizontal plane as the tree), the angle of elevation becomes 54°.
Find the height of the tree.
A. 8.65 meters B. 7.53 meters C. 7.02 meters D. 8.09 meters

83
21. From a point A at the foot of the mountain, the angle of elevation of the top B is 60°. After
ascending the mountain one (1) mile at an inclination of 30° to the horizon, and reaching a point
C, an observer finds that the angle ACB is 135°. The height of the mountain in feet is:
A. 14386 B. 12493 C. 11672 D. 11225
22. A 50-meter vertical tower casts a 62.3-meter shadow when the angle of elevation of the sun is
41.6°. The inclination of the ground is:
A. 4.72° B. 4.33° C. 5.63° D. 5.17°
23. A vertical pole is 10 m from a building. When the angle of elevation of the sun is 45°, the pole
cast a shadow on the building 1 m high. Find the height of the pole.
A. 0-meter B. 11 meters C. 12 meters D. 13 meters
24. A pole cast a shadow of 15 meters long when the angle of elevation of the sun is 61°. If the
pole has leaned 15° from the vertical directly toward the sun, what is the length of the pole?
A. 52.43 meters B. 54.23 meters C. 53.25 meters D. 53.24 meters
25. An observer wishes to determine the height of a tower. He takes sights at the top of the tower
from A and B, which are 50 ft apart, at the same elevation on a direct line with the tower. The
vertical angle at point A is 30° and at point B is 40°. What is the height of the tower?
A. 85.6 feet B. 143.97 feet C. 110.29 feet D. 92.54 feet
26. From the top of tower A, the angle of elevation of the top of the tower B is 46°. From the foot
of tower B the angle of elevation of the top of tower A is 28°. Both towers are on a level ground.
If the height of tower B is 120m, how high is tower A?
A. 38.6 m B. 42.3 m C. 44.1 m D. 40.7 m
27. Point A and B are 100 m apart and are on the same elevation as the foot of a building. The
angles of elevation of the top of the building from points A and B are 21° and 32°, respectively.
How far is A from the building?
A. 217.6 m B. 265.4 m C. 259.2 m D. 277.9 m
28. A man finds the angle of elevation of the top of a tower to be 30 degrees. He walks 85 m nearer
the tower and finds its angle of elevation to be 60 degrees. What is the height of the tower?
A. 76.31 meters B. 73.61 meters C. 73.31 meters D. 73.16 meters
29. The angle of elevation of point C from a point B is 29°42; the angle of elevation of C from
another point A 31.2 m directly below B is 59°23’. How high is C from the horizontal line through
A?
A. 47.1 meters B. 52.3 meters C. 35.1 meters D. 66.9 meters

84
30. A rectangular piece of land 40m x 30 is to be crossed diagonally by a 10-m wide roadway as
shown. If the land cost P1,500,00 per square meter, the cost of the roadway is:
A. P401.10 B. P60,165.00 C. P601,650.00 D. P651,500.00
31. A man improvises a temporary shield from the sun using a triangular piece of wood with
dimensions of 1.4m, 1.5m, and 1.3m. With the longer side lying horizontally on the ground, he
props up the other corner of the triangle with a vertical pole 0.9m long. What would be the area of
the shadow on the ground when the sun is vertically overheard?
A. 0.5 m2 B. 0.75 m2 C. 0.84 m2 D. 0.95 m2
32. A rectangular piece of wood 4cm x 12 cm tall is tilted at an angle of 45°. Find the vertical
distance between the lower corner and the upper corner.
A. 4√2 cm B. 2√2 cm C. 8√2 cm D. 6√2 cm
33. A clock has dial face 12 inches in radius. The minute hand is 9 inches long while with hour
hand is 6 inches long. The plane of rotation of the minute hand is 2 inches above the plane of
rotation of the hour hand. Find the distance between the tips of the hands at 5:30 A.M.
A. 9.17 inches B. 8.23 inches C. 10.65 inches D. 11.25 inches
34. If the bearing of A from B is S 40° W, then the bearing of B from A is:
A. N 40° E B. N 40° W C. N 50° E D. N 50° W
35. A plane hillside is inclined at an angle of 28° with the horizontal. A man wearing skis can
climb this hillside by following a straight path inclined at an angle of 12° to the horizontal, but one
without skis must follow a path inclined at an angle 5° with the horizontal. Find the angle between
the direction of the two paths.
A. 13.21° B. 18.74° C. 15.56° D. 17.22°
36. Calculate the area of a spherical triangle whose radius is 5 m and whose angles are 40°, 65°,
and 110°.
A. 12.34 sq. m. B. 14.89 sq. m. C. 16.45 sq. m. D. 15.27 sq. m.
37. A right spherical triangle has an angle C = 90°, a = 50°, and c = 80°. Find the side b.
A. 45.33° B. 78.66° C. 74.33° D. 75.89°
38. If the time is 8:00 a.m. GMT, what is the time in the Philippines, which is located at 120° East
longitude?
A. 6 p.m. B. 4 a.m. C. 4 p.m. D. 6 a.m.
39. An airplane flew form Manila (14° 36’N, 121° 05’E) at a course of S 30° E maintaining a
certain altitude and following a great circle path. If its groundspeed is 350 knots, after how many
hours will it cross the equator?
A. 2.87 hours B. 2.27 hours C. 3.17 hours D. 3.97 hours

85
40. Find the distance in nautical miles between Manila and San Francisco. Manila is located at 14°
36’ N latitude and 121° 05’ E longitude. San Francisco is situated at 37° 48’ N latitude and 122°
24’ W longitude.
A. 7856.2 nautical miles B. 5896.2 nautical miles
C. 6326.2 nautical miles D. 6046.2 nautical miles
41. A at certain point on the ground, the tower at the top of a 20-m high building subtends an angle
of 45°. At another point on the ground 25 m closer the building, the tower subtends an angle of
45°. Find the height of the tower.
A. 124.75 m B. 87.45 m C. 154.32 m D. 101.85 m
42. A wooden flagpole is imbedded 3 m deep at corner A of a concrete horizontal slab ABCD,
square in form and measuring 20 ft on a side. A storm broke the flagpole at a point one meter above
slab and inclined toward corner C in the direction of the diagonal AC. The vertical angles observed
at the center of the slab and at corner C to the tip of the flagpole were 65° and 35°, respectively.
What is the total length of the flagpole above the slab in yards?
A. 5.61 B. 16.83 C. 4.32 D. 12.38
43. From the third-floor window of a building the angle of depression of an object on the ground
is 35° 58’, while from a sixth-floor window, 9.75 m above the first point of observation the angle
of depression is 58° 35’. How far is the object from the building?
A. 11.9 m B. 10.7 m C. 9.3 m D. 15.3 m
44. The sides of a triangle are 18 cm, 24 cm and 34 cm, respectively. Find the length of the median
to the 24-cm side in cm.
A. 24.4 B. 21.9 C. 23.4 D. 20.4
45. In the spherical triangle ABC, A = 116° 19’, B = 55° 30’, and C = 80° 37’. What is the value
of side a.
A. 115.57° B. 113.21° C. 119.64° D. 115.65°

86
PROBLEM SET 12: TRIANGLES,
QUADRILATERALS, POLYGONS
1. The sides of a right triangle have lengths (a – b), a, and (a + b). What is the ratio of a to b if a
is greater that b and b could not be equal to zero?
A. 1:4 B. 3:1 C. 1:4 D. 4:1
2. Two sides of a triangle measures 8 cm and 12 cm. Find its area if its perimeter is 26 cm.
A. 21.33 sq. m, B. 32.56 sq. m. C. 3.306 sq. m. D. 32.56 sq. m.
3. If three sides of an acute triangle is 3 cm, 4 cm and “x” cm what are the possible values of x?
A. 1 < x < 5 B. 0 < x > 5 C. 0 < x < 7 D. 1 < x > 7
4. In triangle ABC, AB = 8m and BC = 20m. One possible dimension of CA is:
A. 13 B. 7 C. 9 D. 11
5. In a triangle BCD, BC = 25 m and CD = 10 m. The perimeter of the triangle may be.
A. 72 m. B. 70 m. C. 69 m. D. 71 m.
6. The sides of a triangle ABC are AB = 25 cm, BC = 39 cm, and AC = 40 cm. Find its area.
A.486 sq. m. B. 846 sq. m. C. 648 sq. m. D. 468 sq. m.
7. The corresponding sides of two similar triangles are in the ratio 3:2. What is the ratio of their
areas?
A. 3 B. 2 C. 9/4 D. 3/2
8. Find the area of the triangle whose sides are 12, 16, and 21 units.
A. 95.45 sq. units B. 102.36 sq. units C. 87.45 sq. units D. 82.78 sq. units
9. The sides of a right triangle are 8, 15 and 17 units. If each side is doubled, how many square
units will be the area of the new triangle?
A. 240 B. 300 C. 320 D. 420
10. Two triangles have equal base. The altitude of one triangle is 3 units more than its base and
the altitude of the other is 3 units less that its base. Find the altitudes if the areas of the triangle
differ by 21 square units.
A. 5 & 11 B. 4 & 10 C. 6 & 12 D. 3 & 9
11. A triangular piece of wood having a dimension 130 cm, 180 cm and 190 cm is to be divided
by a line bisecting the longest side drawn from its opposite vertex. The area of the part adjacent to
the 180-cm side is:
A. 5126 sq. cm B. 5162 sq. cm C. 5612 sq. cm D. 5216 sq.cm

87
12. Find EB if the area of the inner triangle is 1/4 of the outer tringle as shown.
A. 32.5 B. 55.7 C. 56.2 D. 57.5
13. A piece of wire is shaped to enclose a square whose area is 169 cm2 . It is then reshaped to
enclose a rectangle whose length is 15 cm. The area of the rectangle is:
A. 165 cm2 B. 175 cm2 C. 170 cm2 D. 156 cm2
14. The diagonal of the floor of a rectangle room is 7.50 m. The shorter side of the room is 4.5 m.
What is the area of the room?
A. 36 sq. m. B. 27 sq. m. C. 58 sq. m. D. 24 sq. m.
15. A man measuring a rectangle “x” meters by “y” meters, makes each side 15% too small. By
how many percent will his estimate for the area be too small?
A. 23.55% B. 25.67% C. 27.75% D. 72.25%
16. The length of the side of a square is increased by 100%. Its perimeter is increased by:
A. 25% B. 100% C. 200% D. 300%
17. A piece if wire of length 52 cm is cut into two parts. Each part is then bent to form a square. It
is found that total area of the two squares is 97 sq. cm. The dimension of the bigger square is:
A. 4 B. 9 C. 3 D. 6
18. In the figure shown, ABCD is a square and PDC is an equilateral triangle. Find θ
A. 5° B. 15° C. 20° D. 25°
19. One side of a parallelogram is 10 m and its diagonals are 16 m and 24 m respectively. Its area
is:
A. 156.8 sq. m. B. 185.6 sq. m. C. 158.7 sq. m. D. 142.3 sq. m.
20. If the sides of the parallelogram and an included angle are 6, 10 and 100 degrees respectively,
find the length of the shorter diagonal.
A. 10.63 B. 10.37 C. 10.73 D. 10.23
21. The area of a rhombus is 132 square cm. If its shorter diagonal is 12 cm, the length of the
longer diagonal is:
A. 20-centimeter B. 21-centimeter C. 22-centimeter D. 23-centimeter
22. The diagonals of a rhombus are 10 cm, and 8 cm, respectively. Its area is:
A. 10 sq. m. B. 50 sq. m. C. 60 sq. m. D. 40 sq. m.
23. Given a cyclic quadrilateral whose sides are 4 cm, 5 cm, 8 cm and 11 cm. Its area is:
A. 40.25 sq. m. B. 48.65 sq. m. C. 50.25 sq. m. D. 60.25 sq. m.

88
24. A rectangle ABCD which measures 18 by 24 units is folded once, perpendicular to diagonal
AC, so that the opposite vertices A and C coincide. Find the length of the fold.
A. 2 B. 7/2 C. 54/2 D. 45/2
25. The sides of a quadrilateral are 10m, 8m, 16m and 20m, respectively. Two opposite interior
angles have a sum of 225°. Find the area of the quadrilateral in sq. m.
A. 140.33 sq. cm. B. 145.33 sq. cm. C. 150.33 sq. cm. D. 155.33 sq. cm.
26. A non-square rectangle is inscribed in a square so that each vertex of the rectangle is at the
trisection point of the different sides of the square. Find the ratio of the area of the rectangle to the
area of the square.
A. 5:9 B. 2:7 C. 7:72 D. 4:9
27. A trapezoid has an area of 36 m2 and altitude of 2 m. Its two bases in meters have ratio of 4:5
the bases are:
A. 12, 15 B. 7, 11 C. 16, 20 D. 8, 10
28. Determine the area of the quadrilateral ABCD shown if OB = 80 cm, OA = 120 cm, OD = 150
cm and θ = 25°
A. 2272 sq. cm B. 7222 sq. cm C. 2572 sq. cm D. 2722 sq. cm
29. A corner lot of land is 35 m on one street and 25 m on the other street. The angle between the
two lines of the street being 82°. The other to two lines of the lot are respectively perpendicular to
the lines of the streets. What is the worth of the lot if its unit price is P2500 per square meter?
A. P1.978,456 B. P1,588,045 C. P2,234,023 D. P1,884,050
30. Determine the area of the quadrilateral having (8, -2), (5, 6), and (-7,4) as consecutive vertices.
A. 22 sq. units B. 44 sq. units C. 32 sq. units D. 48 sq. units
31. Find the area of the shaded portion shown if AB is parallel to CD.
A. 16 sq. m. B. 18 sq. m. C. 20 sq. m. D. 22 sq. m.
32. The deflection angles of any polygon has a sum of:
A. 360° B. 720° C. 180°(n-3) D. 180°n
33. The sum of the interior angles of a dodecagon is:
A. 2160° B. 1980° C. 1800° D. 2520°
34. Each interior angle of a regular polygon is 165°. How many sides?
A. 23 B. 24 C. 25 D. 26
35. The sum of the interior angles of a polygon is 540°. Find the number of sides.
A. 4 B. 6 C. 7 D. 5

89
36. The sum of the interior angles of a polygon of n sides is 1080°. Find the value of n.
A. 5 B. 6 C. 7 D. 8
37. How many diagonals does a pentadecagon have:
A. 60 B. 70 C. 80 D. 90
38. A polygon has 170 diagonals. How many sides does it have?
A. 20 B. 18 C. 25 D. 26
39. A regular hexagon with an area of 93.53 square centimeters is inscribed in a circle. The are in
the circle not covered by the hexagon is:
A. 18.38 cm2 B. 16.72 cm2 C. 19.57 cm2 D. 15.68 cm2
40. The area of a regular decagon inscribed in a circle of 15 cm diameter is:
A. 156 sq. cm. B. 158 sq. cm. C. 165 sq. cm. D. 177 sq. cm.
41. The sum of the interior angle of a polygon is 2,520°. How many are the sides?
A. 14 B. 15 C. 16 D. 17
42. The area of a regular hexagon inscribed in a circle of radius 1 is:
A. 2.698 sq. units B. 2,598 sq. units C. 3.698 sq. units D. 3.598 sq. units
43. The corner of a 2-meter square are cut off to form a regular octagon. What is the length of the
sides of the resulting octagon?
A. 0.525 B. 0.626 C. 0.727 D. 0.828
44. If a regular polygon has 27 diagonals, then it is a:
A. hexagon B. nonagon C. pentagon D. heptagon
45. One side of a regular octagon is 2. Find the are of the region inside the octagon.
A. 19.3 sq. units B. 13.9 sq. units C. 21.4 sq. units D. 31 sq. units
46. A regular octagon is inscribed in a circle of radius 10. Find the area of the octagon.
A. 228.2 sq. units B. 288.2 sq. units C. 282.8 sq. units D. 238.2 sq. units
47. Lot ABCDEFA is a closed traverse in the form of a regular hexagon with each side equal to
100 m. The bearing of AB in N 25° E. What is the bearing of CD?
A. S 35° E B. S 45° E C. S 30° E D. S 40° E
48. Two sides of a parallelogram measure 68 cm and 83 cm and the shorter diagonal is 42 cm.
Determine the smallest interior angle of the parallelogram,
A. 23.87° B. 30.27° C. 49.45° D. 12.65°

90
49. The parallel sides of a trapezoid lot of measure 160 m and 240 m and are 40 m apart. Find the
length of the dividing line parallel to the two sides that will divide the lot into two equal areas.
A. 203.96 B. 214.25 C. 200 D. 186.54
50. What do you call a polygon with 35 diagonals?
A. decagon B. nonagon C. dodecagon D. undecagon

91
PART X: POLYNOMIAL
CURVES

A polynomial curve is a curve that can be parametrized by polynomial functions of 𝑅[𝑥], so it is


a special case of rational curve. A rational curve is a curve that can be parametrized with rational
function in R(x).
Therefore, any polynomial curve is an algebraic curve of degree equal to the higher degree of the
above polynomials 𝑃 and 𝑄 of a proper representation. A curve is said to be algebraic when it's
described by a polynomial Cartesian equation with real coefficients.

A polynomial curve cannot be bounded, nor have asymptotes, except if it is a line.

It is obtained by fitting polynomials to each ordinate of an ordered sequence of points.


The above plots show polynomial curves where the order of the fitting polynomial varies
from 𝑝 − 3 to 𝑝 − 1 where 𝑝 is the number of points.

GENERALITIES OF A STRAIGHT LINE

What is the definition of a straight line?


- a straight line is the set of all points between and extending beyond two points. In most
geometries, a line is a primitive object that does not have formal properties beyond length, its
single dimension.

92
How to Name a Straight Line?
- You can take any two points on that line and use those points as the name. You should draw a
line above those two points with arrows on either side, indicating that it is a line that extends on
either side into infinity.
- You can also use just one letter. Usually this letter is indicated in lower case. A commonly used
letter to name lines is y. This manner of naming a line is usually used when the line does not
have any known points on it to reference.

What are the properties of a straight line?


- One dimensional
- Can be horizontal, vertical, diagonal
- Both ends extend in two directions forever
- Makes a 180-degree angle when drawing an angle arc from one point to another

What are the different kinds of straight lines?


- Vertical straight lines go up and down.
- Horizontal straight lines go from left to right or vice versa.
- Parallel straight lines have the same slope and are the same distance apart, so they will never
intersect.
- Perpendicular straight lines cross each other and form four perfect right angles in the process. --
- Slanted or oblique straight lines are just as they sound: they are straight lines at an angle.

Tangents and Normal Curves


Tangents and Normals
- Have you ever sat on a merry-go-round? If yes, then you would understand from your
experience when I tell you that the force you experience is towards the Centre of the merry-go-
round but your velocity or the tendency of motion is in the way towards which your body is
pointing.
Applications in Real Life
- Tangent:
1. If we are traveling in a car around a corner and we drive over something slippery on the road
(like oil, ice, water or loose gravel) and our car starts to slide, it will continue in a direction
tangent to the curve.
2. Likewise, if we hold a ball and swing it around in a circular motion then let go, it will fly off
in a tangent to the circle of motion.

- Normal:
1. When you are going fast around a circular track in a car, the force that you feel pushing you
outwards is normal to the curve of the road. Interestingly, the force that is making you go around
that corner is actually directed towards the center of the circle, normal to the circle.

93
2. The spokes of a wheel are placed normal to the circular shape of the wheel at each point where
the spoke connects with the center.

- A tangent to a curve is a line that touches the curve at one point and has the same slope as the
curve at that point.
- A normal to a curve is a line perpendicular to a tangent to the curve.

Here’s a graph showing the tangent and the normal to a curve at a point:

- Consider a function 𝑓 (𝑥) such as that shown in Figure 1. When we calculate the derivative 𝑓’
of the function at a point 𝑥 = 𝑎. Example, we are finding the gradient of the tangent to the
graph of that function at that point, Figure 1 shows the tangent drawn at 𝑥 = 𝑎. The gradient of
this
tangent is 𝑓’(𝑎).

Figure 1

Figure 1. The tangent drawn at 𝑥 = 𝑎 has gradient 𝑓’(𝑎). We will use this information to calculate the
equation of the tangent to a curve at a particular point, and then the equation of the normal to a curve
at a point.

Equation of a Tangent Line:


To find the equation of the tangent line we need its slope and a point on the line.
Given the function 𝑓(𝑥) and the point (𝑎, 𝑓(𝑎)), we can find the equation of the tangent
line using the slope equation.

94
(𝑓(𝑥) − 𝑓(𝑎))
𝑚=
𝑥−𝑎

As such, the equation of the tangent line at 𝑥 = 𝑎 can be expressed as:

𝑓(𝑥) − 𝑓(𝑎) = 𝑓′(𝑎)(𝑥 − 𝑎)

Equation of a Normal Line:


To find the equation of the normal line at 𝑥 = 𝑎 by taking the negative inverse of the slope of the
tangent line equation. Thus, if 𝑓 ′ (𝑎) is the slope of the tangent line at 𝑥 = 𝑎.The negative inverse is
−1
As such, the equation of the normal line at x = a can be expressed as:
𝑓′(𝑎)
−1
𝑓 (𝑥 ) − 𝑓 ( 𝑎 ) = (𝑥 − 𝑎)
𝑓 ′ (𝑎 )

Examples:
1. Find the equation of the tangent and normal lines of the function 𝑓 (𝑥 ) = √2𝑥 − 5
at the point (7,3)
a. Equation of the Tangent Line:
Step 1: Find the slope of the function by solving for its first derivative.
𝑓(𝑥 ) = √2𝑥 − 5
1
𝑓(𝑥 ) = (2𝑥 − 5)2
1 1
𝑓 ′(𝑥 ) = (2𝑥 − 5)−2 (2)
2
1
𝑓 ′ (𝑥 ) =
√2𝑥 − 5
′( )
Step 2: Knowing 𝑓 𝑎 , solve for the slope of the tangent at a = 7
1
𝑓 ′(7) =
√2(7) − 5
1
𝑓 ′(7) =
3
Step 3: Solve for 𝑓 (𝑎)
𝑓(7) = √2(7) − 5
𝑓(7) = 3
Step 4: Substitute found values into the equation of a tangent line
𝑓 (𝑥 ) − 𝑓(𝑎) = 𝑓 ′ (𝑎)(𝑥 − 𝑎)
1
𝑓 (𝑥 ) − 3 = (𝑥 − 7)
3
b. Equation of the Normal Line:
−1
Step 1: Find the slope of the normal line 𝑓′ (𝑎)

95
1 −1
Since 𝑓 ′ (7) = 3 then 𝑓′ (𝑎) = −3
Step 2: Given the equation of a tangent line, swap slopes.
−1
𝑓 (𝑥 ) − 𝑓 ( 𝑎 ) = ′ (𝑥 − 𝑎 )
𝑓 (𝑎 )
𝑓 (𝑥 ) − 3 = −3 (𝑥 − 7)

2. Find the equation of the tangent and normal lines of the function 𝑓(𝑥) = (𝑥 2 − 1)3 at the point (2,7)
a. Equation of the Tangent Line:
Step 1: Find the slope of the function by solving for its first derivative.
𝑓(𝑥 ) = (𝑥2 − 1)3
𝑓(𝑥 ) = 3(𝑥2 − 1)2 (2x)
𝑓 ′(𝑥 ) = 6𝑥 (𝑥2 − 1)2
Step 2: Knowing 𝑓 ′(𝑎) , solve for the slope of the tangent at a = 2
𝑓 ′(𝑥 ) = 6(2)((2)2 − 1)2
𝑓 ′(2) = 108
Step 3: Solve for 𝑓 (𝑎)
3
𝑓(2) = ((2)2 − 1)
𝑓(2) = 27
Step 4: Substitute found values into the equation of a tangent line
𝑓 (𝑥 ) − 𝑓(𝑎) = 𝑓 ′ (𝑎)(𝑥 − 𝑎)
𝑓 (𝑥 ) − 27 = 108(𝑥 − 2)
b. Equation of the Normal Line:
−1
Step 1: Find the slope of the normal line 𝑓′ (𝑎)
−1 1
Since 𝑓 ′ (2) = 108 then 𝑓′ (𝑎) = − 108
Step 2: Given the equation of a tangent line, swap slopes.
−1
𝑓 (𝑥 ) − 𝑓 ( 𝑎 ) = ′ (𝑥 − 𝑎 )
𝑓 (𝑎 )
1
𝑓 (𝑥 ) − 27 = − (𝑥 − 2)
108
3. Find the equation of the tangent and normal lines of the function 𝑓 (𝑥 ) = (3𝑥 2 − 5)3 at the
point (3,5)
a. Equation of the Tangent Line:
Step 1: Find the slope of the function by solving for its first derivative.
𝑓(𝑥 ) = (3𝑥 2 − 5)3
𝑓(𝑥 ) = 3(3𝑥 2 − 5)2 (6x)
𝑓 ′(𝑥 ) = 18𝑥 (3𝑥 2 − 5)2
′( )
Step 2: Knowing 𝑓 𝑎 , solve for the slope of the tangent at a = 3
𝑓 ′(𝑥 ) = 18(3)(3(3)2 − 5)2
𝑓 ′(3) = 26136
Step 3: Solve for 𝑓 (𝑎)
𝑓 (3) = (3(3)2 − 5)3
96
𝑓 (3) = 10648
Step 4: Substitute found values into the equation of a tangent line
𝑓 (𝑥 ) − 𝑓(𝑎) = 𝑓 ′(𝑎)(𝑥 − 𝑎)
𝑓 (𝑥 ) − 10648 = 26136(𝑥 − 3)
b. Equation of the Normal Line:
−1
Step 1: Find the slope of the normal line 𝑓′ (𝑎)
−1 1
Since 𝑓 ′ (3) = 26136 then 𝑓′ (𝑎) = − 26136
Step 2: Given the equation of a tangent line, swap slopes.
−1
𝑓 (𝑥 ) − 𝑓 ( 𝑎 ) = ′ (𝑥 − 𝑎 )
𝑓 (𝑎 )
1
𝑓 (𝑥 )−= 10648 − (𝑥 − 3)
26136
4. Find the equation of the tangent and normal lines of the function 𝑓 (𝑥 ) = (𝑥 3 − 3)3 at the
point (2,3)
a. Equation of the Tangent Line:
Step 1: Find the slope of the function by solving for its first derivative.
𝑓 (𝑥 ) = (𝑥 3 − 3)3
𝑓 (𝑥 ) = 3(𝑥 3 − 3)2 (3x 2 )
𝑓 ′ (𝑥 ) = 9𝑥 2 (𝑥 3 − 3)2
′( )
Step 2: Knowing 𝑓 𝑎 , solve for the slope of the tangent at a = 2
𝑓 ′ (𝑥 ) = 9(2)2 ((2)3 − 3)2
𝑓 ′ (2) = 900
Step 3: Solve for 𝑓 (𝑎)
𝑓 (2) = (𝑥 3 − 3)3
𝑓 (2) = 125
Step 4: Substitute found values into the equation of a tangent line
𝑓 (𝑥 ) − 𝑓(𝑎) = 𝑓 ′ (𝑎)(𝑥 − 𝑎)
𝑓 (𝑥 ) − 125 = 900(𝑥 − 2)
b. Equation of the Normal Line:
−1
Step 1: Find the slope of the normal line 𝑓′ (𝑎)
−1 1
Since 𝑓 ′ (2) = 900 then =−
𝑓 ′ (𝑎) 900
Step 2: Given the equation of a tangent line, swap slopes.
−1
𝑓 (𝑥 ) − 𝑓 ( 𝑎 ) = ′ (𝑥 − 𝑎 )
𝑓 (𝑎 )
1
𝑓 (𝑥 )−= 125 − (𝑥 − 2)
900

97
Exercises:
1. Find the equation of the tangent and normal lines of the function 𝑓 (𝑥 ) = (𝑥 2 − 1)4 at the
point (3,6)
2. Find the equation of the tangent and normal lines of the function 𝑓 (𝑥 ) = (2𝑥 3 − 3)3 at the
point (2,4)
3. Find the equation of the tangent and normal lines of the function 𝑓 (𝑥 ) = (4𝑥 2 − 2)3 at the
point (1,2)
4. Find the equation of the tangent and normal lines of the function 𝑓 (𝑥 ) = (4𝑥 2 − 2)2 at the
point (4,6)

Answers:
1.
a. Equation of the Tangent Line:
Step 1: Find the slope of the function by solving for its first derivative.
𝑓 (𝑥 ) = (𝑥 2 − 1)4
𝑓 (𝑥 ) = 4(𝑥 2 − 1)3 (2x)
𝑓 ′(𝑥 ) = 8𝑥 (𝑥 2 − 1)3
′( )
Step 2: Knowing 𝑓 𝑎 , solve for the slope of the tangent at a = 3
𝑓 ′(𝑥 ) = 8(3)((3)2 − 1)3
𝑓 ′(3) = 12288
Step 3: Solve for 𝑓 (𝑎)
𝑓 (3) = (𝑥 2 − 1)4
𝑓 (3) = 4096
Step 4: Substitute found values into the equation of a tangent line
𝑓 (𝑥 ) − 𝑓 (𝑎) = 𝑓 ′(𝑎)(𝑥 − 𝑎)
𝑓 (𝑥 ) − 4096 = 12288(𝑥 − 3)
b. Equation of the Normal Line:
−1
Step 1: Find the slope of the normal line 𝑓′ (𝑎)
−1 1
Since 𝑓 ′ (3) = 12288 then 𝑓′ (𝑎) = − 12288
Step 2: Given the equation of a tangent line, swap slopes.
−1
𝑓 (𝑥 ) − 𝑓 (𝑎 ) = ′ (𝑥 − 𝑎 )
𝑓 (𝑎 )
1
𝑓 (𝑥 )−= 4096 − (𝑥 − 3)
12288
2.
a. Equation of the Tangent Line:
Step 1: Find the slope of the function by solving for its first derivative.
𝑓 (𝑥 ) = (2𝑥 3 − 3)3
𝑓 (𝑥 ) = 3(2𝑥 3 − 3)2 (6x 2 )
𝑓 ′(𝑥 ) = 18𝑥 2 (2𝑥 3 − 3)2
Step 2: Knowing 𝑓 ′(𝑎) , solve for the slope of the tangent at a = 2
𝑓 ′ (𝑥 ) = 18(2)2 (2(2)3 − 3)2
98
𝑓 ′ (2) = 12168
Step 3: Solve for 𝑓 (𝑎)
𝑓 (2) = (2𝑥 3 − 3)3
𝑓 (2) = 2197
Step 4: Substitute found values into the equation of a tangent line
𝑓 (𝑥 ) − 𝑓 (𝑎) = 𝑓 ′(𝑎)(𝑥 − 𝑎)
𝑓 (𝑥 ) − 2197 = 12168(𝑥 − 2)
b. Equation of the Normal Line:
−1
Step 1: Find the slope of the normal line 𝑓′ (𝑎)
−1 1
Since 𝑓 ′ (2) = 12168 then 𝑓′ (𝑎) = − 12168
Step 2: Given the equation of a tangent line, swap slopes.
−1
𝑓 (𝑥 ) − 𝑓 (𝑎 ) = ′ (𝑥 − 𝑎 )
𝑓 (𝑎 )
1
𝑓 (𝑥 )−= 2197 − (𝑥 − 2)
12168
3.
a. Equation of the Tangent Line:
Step 1: Find the slope of the function by solving for its first derivative.
𝑓(𝑥 ) = (4𝑥 2 − 2)3
𝑓(𝑥 ) = 3(4𝑥 2 − 2)2 (8x)
𝑓 ′(𝑥 ) = 24𝑥 (4𝑥 2 − 2)2
Step 2: Knowing 𝑓 ′(𝑎) , solve for the slope of the tangent at a = 1
𝑓 ′(𝑥 ) = 24(1)(4(1)2 − 2)2
𝑓 ′(1) = 96
Step 3: Solve for 𝑓 (𝑎)
𝑓 (1) = (4𝑥 2 − 2)3
𝑓 (1) = 8
Step 4: Substitute found values into the equation of a tangent line
𝑓 (𝑥 ) − 𝑓(𝑎) = 𝑓 ′ (𝑎)(𝑥 − 𝑎)
𝑓 (𝑥 ) − 8 = 96(𝑥 − 1)
b. Equation of the Normal Line:
−1
Step 1: Find the slope of the normal line ′ ( )
𝑓 𝑎
′( −1 1
Since 𝑓 1) = 96 then 𝑓′ (𝑎) = − 96
Step 2: Given the equation of a tangent line, swap slopes.
−1
𝑓 (𝑥 ) − 𝑓 ( 𝑎 ) = ′ (𝑥 − 𝑎 )
𝑓 (𝑎 )
1
𝑓 (𝑥 )−= 8 − (𝑥 − 1)
96

99
4.
a. Equation of the Tangent Line:
Step 1: Find the slope of the function by solving for its first derivative.
𝑓 (𝑥 ) = (4𝑥 2 − 2)2
𝑓 (𝑥 ) = 2(4𝑥 2 − 2) (8x)
𝑓 ′(𝑥 ) = 16𝑥 (4𝑥 2 − 2)
Step 2: Knowing 𝑓 ′(𝑎) , solve for the slope of the tangent at a = 4
𝑓 ′(𝑥 ) = 16(4)(4(4)2 − 2)
𝑓 ′(4) = 3968
Step 3: Solve for 𝑓 (𝑎)
𝑓 (4) = (4𝑥 2 − 2)2
𝑓 (4) = 3844
Step 4: Substitute found values into the equation of a tangent line
𝑓 (𝑥 ) − 𝑓 (𝑎) = 𝑓 ′ (𝑎)(𝑥 − 𝑎)
𝑓 (𝑥 ) − 3844 = 3968(𝑥 − 4)
b. Equation of the Normal Line:
−1
Step 1: Find the slope of the normal line 𝑓′ (𝑎)
−1 1
Since 𝑓 ′ (1) = 3968 then =−
𝑓 ′ (𝑎) 3968
Step 2: Given the equation of a tangent line, swap slopes.
−1
𝑓 (𝑥 ) − 𝑓 (𝑎 ) = ′ (𝑥 − 𝑎 )
𝑓 (𝑎 )
1
𝑓(𝑥 )−= 3844 − (𝑥 − 4)
3968

EXTREMA AND THE FIRST DERIVATIVE TEST

The derivative of a function changes sign around a critical point. At that point, the function is
said to have a local relative extremum at that point. If the derivative changes from positive (an
increasing function) to negative (a decreasing function) then the function has a local relative
maximum at the critical point and if the derivative changes from negative (a decreasing
function) to positive (an increasing function) then the function has a local relative minimum at
the critical point.

Consider a relative maximum, we have that on the left, the function is increasing and on the
right the function is decreasing. Similarly, for a relative minimum, on the right the function is
decreasing and on the left the function is increasing

100
When this technique is used to determine local maximum or minimum function values, it is
called the First Derivative Test for Local Extrema

However, note that there is no guarantee that the derivative will change signs, and therefore, it is
essential to test each interval around a critical point.

How to find relative extrema using the First Derivative Test?


1. Find the numbers 𝑥 = 𝑐 in the domain of 𝑓 where 𝑓’(𝑐 ) = 0 or 𝑓’(𝑐 ) does not exist, in
shorter term it means that you need to find the critical numbers of 𝑓
2. Plot these critical numbers on a number line
3. Determine the sign of 𝑓’𝑥) both to the left and right of these critical numbers by evaluating
𝑓’(𝑥 ) at test numbers
4. Identify if it’s a relative maximum, relative minimum or neither of the two by using the rules
of the First Derivative Test

Examples:
1 1
1. Use the First Derivative Test to find the relative extrema of 𝑓 (𝑥 ) = 3 𝑥 3 + 2 𝑥 2 − 12𝑥 + 3
Step 1: Find the critical numbers
𝑓 ′(𝑥 ) = 𝑥 2 + 𝑥 − 12 = 0
(𝑥 + 4)(𝑥 − 3) = 0
𝑥+4= 0 𝑥−3=0
𝑥 = −4 𝑥=3

101
Step 2: Plot these numbers on a number line

-4 3

Step 3: Determine the sign of 𝑓’𝑥) both to the left and right of these critical numbers by
evaluating 𝑓’(𝑥 ) at test numbers.

-5 -4 0 3 4

𝑓 ′(−5) = (−5)2 + (−5) − 12 𝑓 ′(0) = (0)2 + (0) − 12 𝑓 ′(4) = (4)2 + (4) − 12


𝑓 ′ (−5) = +8 𝑓 ′(0) = −12 𝑓 ′ (4) = +8

Step 4: Identify if it’s a relative maximum, relative minimum or neither of the two by using the
rules of the First Derivative Test

By using the rules of First Derivative Test and from the chart above, we determine that 𝑓 has a
relative maximum at 𝑥 = −4 and a relative minimum at 𝑥 = 3

2. Use the First Derivative Test to find the relative extrema of 𝑓 (𝑥 ) = 𝑥 4 − 2𝑥 2 + 12


Step 1: Find the critical numbers
𝑓 ′(𝑥 ) = 4𝑥 3 − 4𝑥
4𝑥 (𝑥 − 1)(𝑥 + 1) = 0
𝑥−1= 0 𝑥+1= 0
𝑥=1 𝑥 = −1

102
Step 2: Plot these numbers on a number line

1 -1

Step 3: Determine the sign of 𝑓’𝑥) both to the left and right of these critical numbers by
evaluating 𝑓’(𝑥 ) at test numbers.

-2 -1 0 1 2

𝑓 ′(−2) = 4(−2)3 − 4(−2) 𝑓 ′(0) = 4(0)3 − 4(0) 𝑓 ′(2) = 4(2)3 − 4(2)


𝑓 ′ (−2) = −24 𝑓 ′(0) = 0 𝑓 ′ (4) = +24

Step 4: Identify if it’s a relative maximum, relative minimum or neither of the two by using the
rules of the First Derivative Test

By using the rules of First Derivative Test and from the chart above, we determine that 𝑓 has a
relative maximum at 𝑥 = 1 and a relative minimum at 𝑥 = −1
3. Use the First Derivative Test to find the relative extrema of 𝑓 (𝑥 ) = 2𝑥 3 − 9𝑥 2 − 108𝑥 + 2
Step 1: Find the critical numbers
𝑓 ′(𝑥 ) = 6𝑥 2 − 18𝑥 − 108
6(𝑥 2 − 3𝑥 − 18) = 0
6(x − 6)(x + 3) = 0

103
𝑥−6= 0 𝑥+3=0
𝑥=6 𝑥 = −3
Step 2: Plot these numbers on a number line

-3 6

Step 3: Determine the sign of 𝑓’𝑥) both to the left and right of these critical numbers by
evaluating 𝑓’(𝑥 ) at test numbers.

-4 -3 2 6 7

𝑓 ′ (−4) = 6(−4)2 − 18(−4) − 108 𝑓 ′ (2) = 6(2)2 − 18(2) − 108 𝑓 ′ (7) = 6(7)2 − 18(7) − 108
′( ′(
𝑓 −4) = +60 𝑓 2) = −120 𝑓 ′ (7) = +60

Step 4: Identify if it’s a relative maximum, relative minimum or neither of the two by using the
rules of the First Derivative Test

By using the rules of First Derivative Test and from the chart above, we determine that 𝑓 has a
relative maximum at 𝑥 = −3 and a relative minimum at 𝑥 = 6
4. Use the First Derivative Test to find the relative extrema of 𝑓 (𝑥 ) = 𝑥 3 − 12𝑥 + 1
Step 1: Find the critical numbers
𝑓 ′(𝑥 ) = 3𝑥 2 − 12
3(𝑥 2 − 4) = 0
3(x − 2)(x + 2) = 0

104
𝑥−2= 0 𝑥+2=0
𝑥=2 𝑥 = −2
Step 2: Plot these numbers on a number line

-2 2

Step 3: Determine the sign of 𝑓’𝑥) both to the left and right of these critical numbers by
evaluating 𝑓’(𝑥 ) at test numbers.

-3 -2 1 2 3

𝑓 ′(−3) = 3(−3)2 − 12 𝑓 ′ (1) = 3(1)2 − 12 𝑓 ′(3) = 3(3)2 − 12


𝑓 ′ (−4) = +15 𝑓 ′ (2) = −9 𝑓 ′(7) = +15

Step 4: Identify if it’s a relative maximum, relative minimum or neither of the two by using the
rules of the First Derivative Test

By using the rules of First Derivative Test and from the chart above, we determine that 𝑓 has a
relative maximum at 𝑥 = −2 and a relative minimum at 𝑥 = 2

Exercises:
1. Use the First Derivative Test to find the relative extrema of 𝑓 (𝑥 ) = 𝑥 4 − 6𝑥 2 + 5
2. Use the First Derivative Test to find the relative extrema of 𝑓 (𝑥 ) = 6𝑥 5 − 40𝑥 3 + 1
3. Use the First Derivative Test to find the relative extrema of 𝑓 (𝑥 ) = 𝑥 5 − 15𝑥 3
4. Use the First Derivative Test to find the relative extrema of 𝑓 (𝑥 ) = 𝑥 3 − 3𝑥 2 − 24𝑥 + 2

105
Solutions:
1. Step 1: Find the critical numbers
𝑓 ′(𝑥 ) = 4𝑥 3 − 36𝑥
4𝑥 (𝑥 2 − 9) = 0
4𝑥 (x − 3)(x + 3) = 0
𝑥−3= 0 𝑥+3=0
𝑥=3 𝑥 = −3
Step 2: Plot these numbers on a number line

-3 3

Step 3: Determine the sign of 𝑓’𝑥) both to the left and right of these critical numbers by
evaluating 𝑓’(𝑥 ) at test numbers.

-4 -3 2 3 4

𝑓 ′(−4) = 4(−4)3 − 36(−4) 𝑓 ′ (2) = 4(2)3 − 36(2) 𝑓 ′ (4) = 4(4)3 − 36(4)


𝑓 ′ (−4) = −112 𝑓 ′ (2) = −40 𝑓 ′ (4) = +112

Step 4: Identify if it’s a relative maximum, relative minimum or neither of the two by using the
rules of the First Derivative Test

By using the rules of First Derivative Test and from the chart above, we determine that 𝑓 has a

106
relative minimum at 𝑥 = 3 but doesn’t have any relative maximum since 𝑓’(−3) stayed the
same.

2. Step 1: Find the critical numbers


𝑓 ′(𝑥 ) = 30𝑥 4 − 120𝑥 2 +
30𝑥 2 (𝑥 2 − 40) = 0
30𝑥 2 (x −)(x + 3) = 0
𝑥−3= 0 𝑥+3=0
𝑥=3 𝑥 = −3
Step 2: Plot these numbers on a number line

-3 3

Step 3: Determine the sign of 𝑓’𝑥) both to the left and right of these critical numbers by
evaluating 𝑓’(𝑥 ) at test numbers.

-4 -3 2 3 4

𝑓 ′(−4) = 4(−4)3 − 36(−4) 𝑓 ′ (2) = 4(2)3 − 36(2) 𝑓 ′ (4) = 4(4)3 − 36(4)


𝑓 ′ (−4) = −112 𝑓 ′ (2) = −40 𝑓 ′ (4) = +112

Step 4: Identify if it’s a relative maximum, relative minimum or neither of the two by using the
rules of the First Derivative Test

107
By using the rules of First Derivative Test and from the chart above, we determine that 𝑓 has a
relative minimum at 𝑥 = 3 but doesn’t have any relative maximum since 𝑓’(−3) stayed the
same.

3. Step 1: Find the critical numbers


𝑓 ′(𝑥 ) = 5𝑥 4 − 45𝑥 2
5𝑥 2 (𝑥 2 − 9) = 0
5𝑥 2 (x + 3)(x − 3) = 0
𝑥+3= 0 𝑥−3=0
𝑥 = −3 𝑥=3
Step 2: Plot these numbers on a number line

-3 3

Step 3: Determine the sign of 𝑓’𝑥) both to the left and right of these critical numbers by
evaluating 𝑓’(𝑥 ) at test numbers.

-4 -3 1 3 4

𝑓 ′(−4) = 5(−4)4 − 45(−4) 2 𝑓 ′ (1) = 5(1)4 − 45(1) 2 𝑓 ′(4) = 5(4)4 − 45(4) 2


𝑓 ′ (−4) = 560 𝑓 ′(1) = −40 𝑓 ′ (4) = 560

Step 4: Identify if it’s a relative maximum, relative minimum or neither of the two by using the
rules of the First Derivative Test

By using the rules of First Derivative Test and from the chart above, we determine that 𝑓 has a
relative maximum at 𝑥 = −3 and relative minimum at x = 3

108
4. Step 1: Find the critical numbers
𝑓 ′ (𝑥 ) = 3𝑥 2 − 6𝑥 − 24
3(𝑥 2 − 2𝑥 − 8) = 0
3(x + 2)(x − 4) = 0
𝑥+2= 0 𝑥−4= 0
𝑥 = −2 𝑥=4
Step 2: Plot these numbers on a number line

-2 4

Step 3: Determine the sign of 𝑓’𝑥) both to the left and right of these critical numbers by
evaluating 𝑓’(𝑥 ) at test numbers.

-3 -2 2 4 5

𝑓 ′(−3) = 3(−3)2 − 6(−3) − 24 𝑓 ′(2) = 3(2)2 − 6(2) − 24 𝑓 ′(5) = 3(5)2 − 6(5) − 24


𝑓 ′ (−3) = 21 𝑓 ′ (2) = −24 𝑓 ′(5) = 21

Step 4: Identify if it’s a relative maximum, relative minimum or neither of the two by using the
rules of the First Derivative Test

By using the rules of First Derivative Test and from the chart above, we determine that 𝑓 has a
relative maximum at 𝑥 = −2 and relative minimum at x = 4

109
CONCAVITY AND THE SECOND DERIVATIVE TEST

Previously, we used first derivative test where we determine whether a critical number 𝑐 of a
function 𝑓 indicated a local extreme value of 𝑓 by looking at the sign of 𝑓′ on either side
of 𝑐. If 𝑓′ changed sign at cc, there was a local extreme value there, since 𝑓′ changes sign
exactly where 𝑓 changes direction.

Here we will learn how to apply the Second Derivative Test, which tells us where a function is
concave upward or downward. Concavity is simply which way the graph is curving up or down.
It can also be thought of as whether the function has an increasing or decreasing slope over a
period. Over a specific interval, a function is concave upward if 𝑓 ′ is increasing, and concave
downward if 𝑓 ′ is decreasing.

From the above


figure we can conclude the
following:
1. Curve segment that lies above its tangent lines is concave upward.
2. Curve segment that lies below its tangent lines is concave downward.

To determine concavity without seeing the graph of the function, we need a test for finding
intervals on which the derivative is increasing or decreasing.
Obviously, the second derivative of function can be used to determine these intervals, in the

110
same way as we have used the first derivative to determine intervals in which function itself is
increasing or decreasing.

How to apply the Second Derivative test?


1. Calculate the first derivative of 𝑓 (𝑥 )
2. Find all critical points
3. Calculate the second derivative of 𝑓 (𝑥 )
4. Evaluate 𝑓’’ at the critical points
5. Apply the second derivative rule. Conclude
Examples:
1. Find all local extrema of the function 𝑓 (𝑥 ) = 2𝑥 3 − 3𝑥 2 − 12𝑥 + 4
Step 1: Calculate the first derivative of 𝑓(𝑥 )
𝑓 ′(𝑥 ) = 6𝑥 2 − 6𝑥 − 12
= 6(𝑥 2 − 𝑥 − 2)
Step 2: Find all critical points
6(𝑥 2 − 𝑥 − 2) = 0
6(𝑥 + 1)(𝑥 − 2) = 0
𝑥+1= 0 𝑥−2= 0
𝑥 = −1 𝑥 = 2
Step 3: Calculate the Second Derivate
𝑓 ′′(𝑥 ) = (6𝑥 2 − 6𝑥 − 12)′
= 12𝑥 − 6
Step 4: Evaluate 𝑓 ′′ (𝑥 ) at the critical points
𝑓 ′′(−1) = 12(−1) − 6
= −18
′′ ( )
𝑓 2 = 12(2) − 6
= 18
Step 5: Apply the second derivative rule, Conclude
- At the critical point 𝑥 = 1, the second derivative (𝑓 ′′(−1) = −18 < 0 )is negative. The
function is therefore concave at that point, indicating it is a local maximum.
- At the critical point x = 2, the second derivative (𝑓 ′′ (2) = 18 > 0) is positive. The function is
convex at that point indicating it is a local minimum.

2. Find all local extrema of the function 𝑓 (𝑥 ) = 𝑥 4 − 3𝑥 3 + 6𝑥 2 + 2


Step 1: Calculate the first derivative of 𝑓(𝑥 )
𝑓 ′ (𝑥 ) = 3𝑥 3 − 9𝑥 2 + 12𝑥
= 3𝑥 (𝑥 2 − 3𝑥 + 4)
Step 2: Find all critical points
= 3𝑥 (𝑥 2 − 3𝑥 + 4)
3𝑥 (𝑥 + 1)(𝑥 − 4) = 0
𝑥+1=0 𝑥−4=0
𝑥 = −1 𝑥 = 4
Step 3: Calculate the Second Derivate
𝑓 ′′(𝑥 ) = (3𝑥 3 − 9𝑥 2 + 12𝑥 )′
= (9𝑥 2 − 18𝑥 + 12)
′′ ( )
Step 4: Evaluate 𝑓 𝑥 at the critical points

111
𝑓 ′′ (−1) = 9(−1)2 − 18(−1) + 12
= 39
𝑓 ′′ (4) = 9(4)2 − 18(4) + 12
= 84
Step 5: Apply the second derivative rule, Conclude
- At the critical point 𝑥 = 1, the second derivative (𝑓 ′′(−1) = 39 > 0 )is positive. The function
is therefore concave at that point, indicating it is a local minimum.
- At the critical point x = 2, the second derivative (𝑓 ′′ (4) = 18 > 0) is positive. The function is
convex at that point indicating it is a local minimum.

3. Find all local extrema of the function 𝑓 (𝑥 ) = 𝑥 3 + 3𝑥 2 − 24𝑥 + 2


Step 1: Calculate the first derivative of 𝑓(𝑥 )
𝑓 ′(𝑥 ) = 3𝑥 2 + 6𝑥 − 24
= 3(𝑥 2 + 2𝑥 − 8)
Step 2: Find all critical points
= 3𝑥 (𝑥 2 + 2𝑥 − 8)
3𝑥 (𝑥 − 2)(𝑥 + 4) = 0
𝑥−2=0 𝑥+4=0
𝑥 = 2 𝑥 = −4
Step 3: Calculate the Second Derivate
𝑓 ′′ (𝑥 ) = (3𝑥 2 + 6𝑥 − 24)′
= 6x + 6

Step 4: Evaluate 𝑓 ′′ (𝑥 ) at the critical points


𝑓 ′′ (2) = 6(2) + 6
= 18
𝑓 ′′ (−4) = 6(−4) + 6
= −18
Step 5: Apply the second derivative rule, Conclude
- At the critical point 𝑥 = 2, the second derivative (𝑓 ′′(2) = 18 > 0 )is positive. The function
is therefore concave at that point, indicating it is a local minimum.
- At the critical point x = -4, the second derivative (𝑓 ′′(−4) = −18 > 0) is negative. The
function is convex at that point indicating it is a local maximum.

4. Find all local extrema of the function 𝑔(𝑥 ) = 𝑥 3 − 9𝑥 2 + 15𝑥 − 7


Step 1: Calculate the first derivative of 𝑓(𝑥 )
𝑔(𝑥 ) = 3𝑥 2 − 18𝑥 + 15
= 3(𝑥 2 − 6𝑥 + 5)
Step 2: Find all critical points
= 3(𝑥 2 − 6𝑥 + 5)
3(𝑥 − 1)(𝑥 − 5) = 0
𝑥−1= 0 𝑥−5= 0
𝑥= 1 𝑥= 5
Step 3: Calculate the Second Derivate
𝑓 ′′ (𝑥 ) = (3𝑥 2 − 18𝑥 + 15)′

112
= 6𝑥 − 18

Step 4: Evaluate 𝑓 ′′ (𝑥 ) at the critical points


𝑓 ′′(1) = 6(1) − 18
= −12
′′ ( )
𝑓 5 = 6(5) − 18
= 12
Step 5: Apply the second derivative rule, Conclude
- At the critical point 𝑥 = 1, the second derivative (𝑓 ′′(1) = −12 < 0 ) is negative. The
function is therefore concave at that point, indicating it is a local maximum.
- At the critical point x = 5, the second derivative (𝑓 ′′ (5) = 18 < 0) is positive. The function is
convex at that point indicating it is a local minimum

POINTS OF INFLECTION
Inflection points area points where the function changes concavity, i. e. from
being “concave up” to being “concave down” or vice versa. They can be found by
considering where the second derivative changes signs. In similar to critical
points in the first derivative, inflection points will occur when the second
derivative is either zero or undefined.

113
Examples:
1. 𝑦 = 5𝑥 3 + 2𝑥 2 − 3𝑥
2. Find the point(s) of inflection for the function y = x³
3. 𝑦 = 𝑥 3 − 6𝑥 2 + 12𝑥 − 5
4. Find all the points of inflection of 𝑓(𝑥) = −𝑥³ + 𝑥² − 𝑥 + 1

Answers:
1. Let’s work out the second derivative:
𝑦 ′ = 15𝑥 2 + 4𝑥 − 3
𝑦 ′′ = 30𝑥 + 4
And 30x + 4 is negative up to x = -4/30 =-2/15, positive from there onwards So,
f(x) is concave downward up to x = -2/15
f(x) is concave upward from x = -2/15 on
And the inflection point is at x = -2/15
2.
point of inflection is found where the graph of a function changes concavity. To find this
algebraically, we want to find where the second signs, from negative to positive, or vice-versa.
So, we find the second derivative of the given function y = x³.
The first derivative using the power rule
y = 𝑥𝑛 => y’ = 𝑛𝑥 𝑛−1 is,
y’ = 3x² and the second derivative is y” = 6x.
we then find where this second derivative equals 0, 6x = 0 when x = 0.

114
We then look to see if the second derivative changes signs at this point. Both graphically and
algebraically, we can see that the function y” = 6x does indeed change sign at, and only at, x = 0,
so this is our inflection point.
3. Let’s work out the second derivative
𝑦 ′ = 3𝑥 2 − 12𝑥 + 12
𝑦 ′′ = 6𝑥 − 12
And 6x – 12 is negative up to x = 2, positive from there onwards, So
f(x) is concave downward up to x = 2
f(x) is concave upward from x = 2 on

And the inflection point is at x = 2


4.
In order to find the points of inflection, we need to find 𝑓”(𝑥) using the power rule,
f(x) = 𝑥 𝑛 => 𝑓’(𝑥) = 𝑛𝑥 𝑛−1
f(x) = -x³ + x² - x + 1
f’(x) = -3x² + 2x – 1
f”(x) = -6x + 2
Now, we set 𝑓”(𝑥) = 0, and solve for x.
-6x + 2 = 0
-6x = -2
1
x=
3

To verify this is a true inflection point, we need to plug in a value that is less than it and a value
that is greater than it into the second derivative. If there is a sign change around the point then it
is a true inflection point.
Let x = 0
𝑓” (0) = -6(0) + 2 = 2
Now let x = 1
𝑓”(1) = -6(1) + 2 = -4
1
Since the sign changes from a positive to a negative around the point x = 3, we can conclude it is
an inflection point.
Exercises:
1. Find the points of inflection of f(x) = x² - 2x + 1.

115
2. Find the points of inflection of the following function:
f(x) = x³ + 2x² + x
3. Find all the points of inflection of f(x) = 𝑥 4 + 3𝑥³ − 4𝑥 + 1.
4. What are the x coordinates of the points of inflection for the graph
𝑥4
y= + x³ + 4x² - 8x + 5?
12

Answers
In order to find the points of inflection, we need to find f”(x)
f(x) = x² - 2x + 1
𝑓’(𝑥) = 2x – 2
𝑓”(𝑥) = 2
Now we set 𝑓”(𝑥) = 0.
0 = 2.
This last statement says that 𝑓”(𝑥) will never be 0. Thus there are no points of inflection.
2.
The points of inflection of a given function are the values at which the second derivative of the
function is equal to zero.
The first derivative of the function is
𝑓’(𝑥) = 3x² + 4x + 1, and the derivative of this function (the second derivative of the original
function) is
𝑓”(𝑥) = 6x + 4
Both derivatives were found using the power rule
𝑑
𝑥 𝑛 = 𝑛𝑥 𝑛−1
𝑑𝑥
2
Solving 0 = 6x + 4, x = − 3

To verify that this point is a true inflection point, we need to plug in a value that is less than the
point and one that is greater than the point into the second derivative. If there is a sign change
between the two numbers than the point in question is an inflection point.
Let’s plug in x = -1.
6(-1) + 4 = -2
Now plug in x = 0

116
6(0) + 4 = 4
2
Therefore, x = − 3 is the only point of inflection of the function.

3.
In order to find all the points of inflection, we first find f”(x) using the power rule twice,
f(x) =𝑥 𝑛 => f’(x) = 𝑛𝑥 𝑛−1
f(x) = 𝑥 4 + 3x³ - 4x + 1
𝑓’(𝑥) = 4x³ + 9x² - 4
𝑓”(𝑥) = 12x² + 18x
Now we set 𝑓”(𝑥) = 0
12x² + 18x = 0
Now we factor the left hand side
x (12x + 18) = 0
From this, we see that there is one point of inflection at x = 0.
For the point of inflection, let’s solve for x for the equation inside the parentheses.
12x + 18 = 0
12x = -18
18 3
x = − 12 = − 2

4.
Inflection points are the points of a graph where the concavity of the graph changes. The
inflection points of a graph are found by taking the double derivative of the graph equation,
setting is equal to zero, then solve for x.
To take the derivative of this equation, we must use the power rule,
𝑑
𝑥 𝑛 = 𝑛𝑥 𝑛−1
𝑑𝑥

We also must remember that the derivative of a constant is 0.


After taking the first derivative of the graph equation using the power rule, the equation becomes
𝑥³
y’ = + 3x² + 8x – 8.
3

In this problem, the double derivative of the graph equation comes out to 𝑦” = 𝑥² + 6𝑥 + 8,
factoring this equation out it becomes 𝑦” = (𝑥 + 4)(𝑥 + 2)

117
Solving for x when the equation is set equal to zero, the inflection points are located at x = -4, 2.

SKETCHING POLYNOMIAL CURVES


The following steps are taken in the process of curve sketching.

1. Domain
Find the domain of the function and determine the points of discontinuity (if
any).
2. Intercepts
To find the x- intercept, we set y = 0 and solve the equation for x. Similarly, we
set x = 0 to find the y- intercept. Find the intervals where the function has a
constant sign ( f (x) > 0 and f (x0 < 0).
3. Symmetry
Determine whether the function is even, odd, or neither, and check the
periodicity of the function. If f (-x) = f (x) for all x in the domain, then f (x) is even
and symmetric about the y-axis. If f (-x) = -f (x) for all x in the domain, then f (x)
is odd and symmetric about the origin.
4. Asymptotes
Find the vertical; horizontal and oblique (slant) asymptotes of the function.
5. Intervals of Increase and Decrease
Calculate the first derivative f’(x) and find the critical points of the function.
(Remember that critical points are the points where the first derivative is zero or
does not exist.) Determine the intervals where the function is increasing and
decreasing using the First Derivative Test.
6. Local Maximum and Minimum
Use the First or Second Derivative Test to classify the critical points as local
maximum or minimum. Calculate the y-values of the local extrema points.
7. Concavity/Convexity and Points of Inflection
Using the Second Derivative Test, find the points of inflection (at which f” (x) =
0). Determine the intervals where the function is convex upward (f” (x) < 0) and
convex downward (f” (x) > 0).
8. Graph of the Function
Sketch a graph of f (x) using all the information obtained above.

Sketch graphs of the following functions.

Example Problem #1: y = x³ - 3x² +2x.

The function is defined for all x  R.


Consequently, this function has no vertical asymptotes. Check for oblique (slant) asymptotes.
Calculate the slope of the asymptote:

118
This indicates that the function has also no oblique asymptotes.
Determine the points of intersection of the graph with the coordinate axis.
y (0) = 0
Next, solving the equation
x³ - 3x² + 2x = 0,
we find:
x (x² - 3x + 2) = 0,
𝑥1 = 0, 𝑥2 = 1, 𝑥3 = 2, that is the function that has three real roots.

The intervals where the function is positive or negative can be found solving the following
inequality
x³ - 3x² + 2x > 0,
=> x (x – 1)(x – 2) > 0.
The first derivative of the function is
y’ (x) = (x³ - 3x² + 2x)’
= 3x² - 6x + 2.
We find the stationary points by setting the first derivative equal to zero:

119
3
When passing through the point x = 1− 3 , the derivative changes sign from plus to minus.
3
Hence, this point is the maximum point. Similarly, we establish that x = 1+ is the minimum
3
point. Calculate the approximate value of the function at the points of maximum and minimum:

=>

Similarly, we find that


3 2 3
y (1 + 3 ) = x = 1−  -0, 38.
9

Thus, the function has a local maximum at the point


 3 2 3
(1 − 3 ,  (0, 42; 0,38)
9

Respectively, the local minimum is reached at the point


3 23
(1 + ,− )  (1, 58; -0, 38)
3 9

The intervals of increasing/decreasing


Consider the second derivative:
y” (x) = (3x² - 6x + 2)’ = 6x -6;
y” (x) = 0, => 6x – 6 = 0,
= > x = 1.
If x ≤ 1, the function is convex upward, and if x ≥ 1, it is convex downward. Hence, the point x =
1 is a point of inflection. At this point we have:
y (1) = 1³ - 3 (1²) + 2 (1) = 0
Given these results, we can draw a schematic graph of the function.

120
Example Problem #2: f(x) = x² (x + 3)
The function is defined for all x  R.. It has the following x-intercepts:
f (x) = 0, => x² (x + 3) = 0,
= > 𝑥1 = 0, 𝑥2 = -3.
The y-intercept is equal to f (0) = 0
The function is positive on the intervals (-3, 0) and (0, +) and negative on (-, -3).
The function is neither even nor odd, and it has no asymptotes.
Take the derivative:
f’(x) = (x² (x + 3))’ = (x³ + 3x²)’ = 3x² + 6x
Find the critical points:
f’ (x) = 0, => 3x² + 6x = 0,
= > 3x (x + 2) = 0,
= > 𝑥1 = 0, 𝑥2 = -2

We can see from the sign chart that x = -2 is a point maximum, and x = 0 is a point of minimum.
The y-values of these points are

121
f (-2) = (-2)² (-2 + 3) = 4
f (0) = 0
We differentiate once more to get the second derivative:
f”(x) = (3x² +6x)’ = 6x + 6
f”(x) = 0, => 6x + 6 = 0,
= > x = -1.
The graph of the function is concave downward on (-, -1) and concave upward on (-1, +).
Therefore, x = -1 is a point of inflection. The y-coordinate of this point is
F (-1) = (-1)² (-1 + 3) = 2
Given this result, we can draw a schematic graph of the function.

Example Problem #3: y = (x + 2)² (x – 1)


The function is defined for all real x. Therefore, it has no vertical asymptotes. Check for oblique
(slant) asymptotes.

Since the slope k is infinity, the function has no oblique asymptotes, too.
Find the points of intersection of the graph with the coordinate axis:

122
y (0) = 2² (-1) = -4;
y (x) = 0,
= > (x + 2)² (x – 1) = 0,
= > 𝑥1 = -2, 𝑥2 = 1
The function is positive for x > 1 and negative for x  (-, -2)  (-2, 1)
Calculate the first derivative:

The stationary points are


y’ (x) = 0, => 3x (x + 2) = 0
= > 𝑥1 = 0, 𝑥2 = -2
The derivative changes its sign.

Therefore, x = -2 is the maximum point, and x = 0 is the minimum point. The function has the
following values at these extrema points:
y (-2) = -4, y (0) = 0
Find the second derivative:
y” (x) = [3x (x + 2)]’ = 3 (x + 2) + 3x = 6x + 6
So the function is strictly convex upward for x < -1 and strictly convex downward for x > -1.
Hence, x = -1 is the inflection point, and y (-1) = (-1 +2)² (-1 -1) = -2.

123
As a result, we can draw a graph of the function. The shape of the graph resembles a cubic
parabola.

1
Example Problem #4: y = 1+𝑥²

The function is defined for all real values of x. Consequently, it has no vertical asymptotes. Since

then the graph of the function has horizontal


asymptotes y = 0, that is the x-axis is the horizontal asymptote.
This function is even. Indeed,
1 1
y (-x) = 1+(−1)² = = y (x)
1+𝑥²
It is obvious that the function has no roots and positive for all x. At the point x = 0, its value is
1
y (0) = 1+0² = 1

Find the first derivative:


1
y’ (x) = ( )’
1+𝑥²
1
= − (1+𝑥2)² ● (1 + x²)’

2𝑥
=
(1+𝑥 2 )²

This shows that x = 0 is a stationary point. When passing through this point, the derivative
changes sign from plus to minus.

124
Therefore, we have a maximum at x = 0. Its value is y (0) = 1
Calculate the second derivative:
2𝑥
y” (x) = (− (1+𝑥2 )2)′
2
2(1+𝑥 2 ) −2𝑥 ● 2 (1+𝑥 2 )
=− (1+𝑥 2 )^4

8𝑥²−2−2𝑥² 6𝑥²−2
= =
(1+𝑥 2 )³ (1+𝑥 2 )³

It is equal to zero at the following points:

When passing through these points, the second derivative changes its sign. Therefore, both points
are inflection points. The function is strictly convex downward in the intervals (-, -3) and (3,
+) and, accordingly, strictly convex upward in the interval (-3, 3). Since, the function is
even, the found inflection points have the same values of y:
1 1 1
y (±3) = 1+(±3)² = =
1+ 3 4

This figure presents a schematic graph of the function.

125
ANSWER AND SHOW THE GRAPH OF THE FOLLOWING EXERCISES:
𝑥²+1
1. f (x) = 𝑥−1
𝑥
2. f (x) = x³𝑒
𝑒𝑥
3. f (x) = 𝑥
4. f (x) = x ln x

𝑥²+1
Exercise Answer #1: f (x) =
𝑥−1

The function is defined for all x except the point x = 1 where it has a discontinuity. Find the y-
intercept:
0²+1
f (0) = = -1
0−1

The function is negative for x < 1 and positive for x > 1, but it has no x-intercepts.
Look for vertical asymptote near x = 1:

There is a vertical asymptote at x = 1.


Rewrite the function in the form
𝑥²+1
f (x) = 𝑥−1

𝑥²−𝑥+𝑥−1+2 𝑥(𝑥− 1)+𝑥−1+2 2


= = = x + 1+
𝑥−1 𝑥−1 𝑥−1
2
where 𝑥−1 => 0 as x → . Hence, the function has an oblique asymptote y = x + 1.

Take the first derivative:


𝑥²+1
f’ (x) = ( 𝑥−1 )’

2𝑥 ● (𝑥−1)−(𝑥 2 +1) ● (𝑥−1)


= (𝑥−1)²

2𝑥²−2𝑥−𝑥 2 −1
= (𝑥−1)²

𝑥²−2𝑥−1
=
(𝑥−1)²

126
Determine the critical points:

Solve the quadratic equation:

Thus, the function has two critical points: 𝑥1 = 1 - 2


 -0.41 and 𝑥2 = 1 + 2  2.41
The second derivative is written as
𝑥²−2𝑥− 1 4
f” (x) = ( )’ =
(𝑥−1)² (𝑥−1)³

We see that the function is concave downward at x < 1 and concave upward at x > 1, though it
has no inflection points.
Draw a sign chart for the function and its derivatives.

127
The point x = 1 - 2 is a local maximum, and the point x = 1 + 2 is a local minimum. Calculate
their y-coordinates:

Now, we can sketch a graph of the function.

128
Exercise Answer #2: f (x) = x³𝑒 𝑥
The function is defined and differentiable on the whole real line. In this case, it does not have a
vertical asymptote. Check for the existence of oblique asymptotes. Compute the following limits:

When calculating the second limit we made the change (-x) → z and used L’Hopital’s Rule. It
can be seen that the function approaches zero as x → -, that is the graph has the horizontal
asymptote y = 0.
Calculate the roots of the function:
y (x) = 0, => x³𝑒 𝑥 = 0,
= > x³ = 0, => x=0
It follows that the function is positive for x > 0 and negative for x < 0
Find the first derivative and determine the extreme points and intervals of monotonicity:
y’ (x) = (x³𝑒 𝑥 )’
= (x³)’ 𝑒 𝑥 + x³(𝑒 𝑥 )’
= 3 x²𝑒 𝑥 + x³𝑒 𝑥 = x²𝑒 𝑥 (3 + x)
Then the roots of the derivative have the following values:
y’ (x) = 0, => x²𝑒 𝑥 (3 + x) = 0,
= > 𝑥1 = 0, 𝑥2 = -3
The intervals of constant sign of the first derivative are indicated in the figure above. The
function is strictly decreasing in the interval (-, -3) and strictly increasing in the intervals (-3, 0)
and (0, +). Thus, the point x = -3 is a minimum. The other critical point x = 0 is not a local
extremum as when passing through this point the derivative does not change its sign. At the point
of minimum, we have
y (-3) = (-3)³ 𝑒 −3
 -27 ● 0, 0498  -1, 34

129
Now we investigate the second derivative:

Compute its roots:

So the second derivative has the following roots:


𝑥1 = -3 - 3, 𝑥2 = -3 + 3, 𝑥3 = 0
When passing through each of these points, the sign of the derivative is reversed.

Therefore, these points are points of inflection. The approximate values of the corresponding
coordinates y are given by
y (-3 - 3)

= (-3 - 3)³ 𝑒 −3−3  -0, 93;


y (-3 + 3)

= (-3 + 3)³ 𝑒 −3∓3  -0, 57;


y (0) = 0
Now, considering all the characteristic points, we can sketch a graph of the function.
130
𝑒𝑥
Exercise Answer #3: f (x) = 𝑥

The function is defined for all x except the point x = 0 where it has a discontinuity.
The function has no roots; it is positive for x > 0 and negative for x < 0. The function is neither
even, nor odd.

Make sure it has a vertical asymptote at x = 0:

Hence, x = 0 (y-axis) is a vertical asymptote.


Check for horizontal asymptotes. Using L’Hopital’s rule, we have:

We see that y = 0 (x-axis) is a horizontal asymptote of the function as x → -.


Calculate the derivative:
𝑒𝑥
f’ (x) = ( )′
𝑥

(𝑒 𝑥 )′ ● 𝑥−𝑒 𝑥 ● 𝑥′ 𝑥𝑒 𝑥 −𝑒 𝑥 𝑒 𝑥 (𝑥−1)
= = =
𝑥² 𝑥² 𝑥²

131
Determine the critical points:

It follows from the sign chart that x = 1 is a point of local minimum. Its y-coordinate is
𝑒1
f (1) = = e, so the local minimum is at (1, e).
1

Next, we are to investigate the concavity of the function by using the second derivative:
𝑒 𝑥 (𝑥−1) 𝑒 𝑥 (𝑥 2 −2𝑥+1)
f” (x) = ( )’ =
𝑥² 𝑥³

Note that the quadratic function x² - 2x + 2 in the numerator does not have roots and is always
positive. As the exponential function 𝑒 𝑥 is also positive everywhere, then the sign of the second
derivative depends only on the sign of the denominator. Hence, the function is concave
downward on (-, 0) and concave upward on (0, +).
Here’s a schematic graph of the function.

Exercise Answer #4: f (x) = x ln x


The function is defined for x > 0. Determine the x-intercepts:
f (x) = 0, => x ln x = 0, = > x = 1
The other root x = 0 does not belong to the domain of the function. Clearly, that the function is
negative for 0 < x < 1 and positive for x > 1.

132
Look for the vertical asymptote at x = 0: Using L’Hopital’s rule, we can write the right-sided
limit in the form

The function has neither vertical, nor horizontal asymptotes. Consider the possible oblique
asymptote as x → :

So there are no oblique asymptotes.


Take the derivative:

Find the critical points:


f’ (x) = 0, => ln x + 1 = 0,
1
=> ln x = -1, => x=𝑒
1
The derivative changes sign from negative to positive at x = 𝑒
1
Therefore, x = 𝑒 is a point of local minimum. The minimum value is
1 1 1 1 1
𝑓𝑚𝑖𝑛 = f ( 𝑒 ) = 𝑒ln 𝑒
= 𝑒
● (-1) = − 𝑒
1 1
Hence, the minimum point is at ( 𝑒 , − 𝑒 )

Calculate the second derivative:


1
f” (x) = (ln x + 1)’ = 𝑥

Since 𝑓’’ (x) > 0 for all x in the domain, the function is concave up
everywhere.

133
PROBLEM SET 13: CIRCLES, PARABOLA,
ELLIPSE & MISCELLANEOUS FIGURES
1.In the figure shown OA and OB are tangent to the circle. If <AOB is 50 degrees find the
<APB.
A. 45 degrees
B. 50 degrees
C. 60 degrees
D. 65 degrees
2. In the figure shown, arc BC is half the length of arc CD. Solve for degrees
A. 30 degrees
B. 40 degrees
C. 45 degrees
D. 50 degrees
3. In the figure shown, MAN is a tangent to the circle of center O. If <BAN = 70 degrees find
degrees.
A. 220 degrees
B. 210 degrees
C. 140 degrees
D. 250 degrees
4. In the figure shown, AB is the diameter of the circle. Find the angle degrees.
A. 100 degrees
B. 105 degrees
C. 110 degrees
D. 210 degrees
5. The area of a circle is 89.42 square inches. What is the circumference
A. 35.33 inches
B. 32.25 inches
C. 33.52 inches
D. 35.55 inches
6. Find the area of the circle shown.
A. 152.53 sq.units
B. 193.30 sq.units
C. 215.30 sq.units
D. 226.98 sq.units

134
7. A circle whose area is 452cm square is cut into two segment by a chord whose distance from
the center of the circle is 6 cm. Find the area of the larger segment in the cm square.
A. 372.5
B. 363.6
C. 368.4
D. 377.6
8. A circle is divided into two parts by a chord, 3cm away from the center. Find the area of the
smaller part, in cm square, if the circle has an area of 201 cm square.
A. 51.4
B. 57.8
C. 55.2
D. 53.7
9. A quadrilateral ABCD is inscribed in a semi-circle with a side AD coinciding with the
diameter of the circle. If sides AB, BC, and CD are 8cm, 10cm, and 12cm long, respectively,
find the area of the circle.
A. 317 sq. cm
B. 356 sq. cm
C. 456 sq. cm
D. 486 sq. cm
10. A semi-circle of radius 14cm is formed from a piece of wire. If it is bent into a rectangle
whose length is 1cm more than its width, find the area of the rectangle.
A. 256.25 sq. cm
B. 323.57 sq. cm
C. 386.54 sq. cm
D. 452.24 sq. cm
11. Find the area of the shaded circle shown.
A. 2/3 (pi)r^2
B. 2/9 (pi)r^2
C. 4/3 (pi)r^2
D. 4.9 (pi)r^2
12. The angle of a sector is 30 degrees and the radius is 15 cm. What is the area of the sector?
A. 89.5cm^2
B. 58.9 cm^2
C. 59.8 cm^2
D. 85.9 cm^2
13. A sector has a radius of 12cm. If the length of its arc is 12 cm, its area is;
A. 66 sq. cm
B. 82 sq. cm
C. 144 sq. cm

135
D. 72 sq. cm
14. The perimeter of a sector is 9cm and its radius is 3cm. What is the area of the sector?
A. 4cm^2
B. 9/24cm^2
C. 11/24cm^2
D. 27/24cm^2
15. Find the area of the shaded portion.
A. 50(pi)
B. 25(pi)
C. 20(pi)
D. 30(pi)
16. A swimming pool is to be constructed in the shape of the partially overlapping identical
circles. Each of the circles has a radius of 9m, and each passes through the center of the other,
Find the area of the swimming pool.
A. 302.33 m^2
B. 362.55 m^2
C. 389.99 m^2
D. 409.44 m^2
17. Given are two concentric circles with the outer circle having a radius of 10cm. If the area of
the inner circle is half of the outer circle, find the boarder between the two circles the two circles.
A. 2.930cm
B. 2.856cm
C. 3.265cm
D. 2.444cm
18. A circle of radius 5cm has a chord which is 6cm long. Find the area of the circle concentric
to this circle and the tangent to the given chord.
A. 14(pi)
B. 16(pi)
C. 9(pi)
D. 4(pi)
19. A reversed curved on a railroad track consist of two circular arcs. The central angle of one
side is 20 degrees with radius 2500ft, and the central angle of the other is 25 degrees with the
radius 3000ft. Find the total lengths of the two arcs.
A. 2812ft.
B. 2218ft.
C. 2821ft.
D. 2182ft.
20. Given a triangle whose sides are 24 cm, 30 cm, and 36 cm. Find the radius of a circle which
is tangent to the shortest and longest side of the triangle and whose center lies on the third side.

136
A. 9.111cm
B. 11.91cm
C. 12.31cm
D. 18cm
21. Find the area of the largest circle that can be cut from a triangle whose sides are 10cm, 18cm,
and 20cm
A. 11(pi)cm^2
B. 12(pi)cm^2
C. 14(pi)cm^2
D. 15(pi)cm^2
22. The diameter of the circle circumscribed about a triangle ABC with sides a, b, c is equal to:
A. a/sin A
B. b/sin B
C. c/sin C
D. all of the above
23. The sides of a triangle are 14cm, 15cm, and 13cm. Find the area of the circumscribing circle.
A. 207.4 sq. cm.
B. 209.6 sq. cm.
C. 215.4 sq. cm.
D. 220.5 sq. cm.
24. What is the radius of the circle circumscribing an isosceles right having an area of 162 sq.
cm?
A. 13.52
B. 14.18
C. 12.73
D. 1564
25. If the radius of the circle is decreased by 20%, by how much is its area decreased?
A. 36%
B. 26%
C. 46%
D. 56%
26. The distance between the centers of the three circles which are mutually tangent to each other
externally are 10, 12 and 14 units. The area of the largest circle is.
A. 72(pi)
B. 23(pi)
C. 64(pi)
D. 16(pi)
27. The sides of a cyclic quadrilateral measures 8cm, 9cm, 12cm, and 7cm, respectively, Find the
area of the circumscribing circle.

137
A. 8.65 cm^2
B. 186.23 cm^2
C. 6.54 cm^2
D. 134.37 cm^2
28. The wheel of a car revolves n times, while the car travels x km. The radius of the wheel in
meter is:
A. 10000 x/(pi*n)
B. 500 x/(pi*n)
C. 500000 x/(pi*n)
D. 5000 x/(pi*n)
29. If the inside wheels of a car running a circular track are going half as fast as the outside
wheel, determine the length of the track, described by the outer wheels, if the wheels are 1.5m
apart.
A. 4(pi)
B. 5(pi)
C. 6(pi)
D. 8(pi)
30. A goat is tied to a corner of a 30 ft by 35 ft building. If the rope is 40 ft long and the goat can
reach 1 ft farther than the rope length, what is the maximum area the goat can cover?
A. 5281 ft^2
B. 4084 ft^2
C. 3961 ft^2
D. 3970 ft^2
31. What is the area of the shaded portion shown?
A. 8-8(pi)
B. 8-2(pi)
C. 8-4(pi)
D. 8-(pi)
32. The interior angles of a triangle measures 2x, x+15, and 2x + 14. What is the value of x?
A. 30 degrees
B. 66 degrees
C. 42 degrees
D. 54 degrees
33. Two complementary angles are in the ration 2:1. Find the larger angle.
A. 30 degrees
B. 60 degrees
C. 75 degrees
D. 15 degrees

138
34. Two transmission towers 40ft. high is 200 feet apart. If the lowest point of the cable is 10 feet
above the ground, the vertical distance from the roadway to the cable is 50 ft from the center is:
A. 17.25 feet
B. 17.5 feet
C. 17.75 feet
D. 18 feet
35. What is the area bounded by the curves y^2=4x and x^2=4y?
A. 6.0
B. 7.333
C. 6.666
D. 5.333
36. What is the area between y = 0, y = 3x^2, x = 0, and x = 2?
A. 8
B. 12
C. 24
D. 6
37. A circle having an area pf 224 sq. m, is inscribed in an octagon. Find the area of the octagon.
A. 238.6 sq. m
B. 245.2 sq. m
C. 236.3 sq. m
D. 246.7 sq. m
38. A circle is circumscribed about a hexagon. Determine the area of the hexagon if the area
outside the hexagon but inside the circle is 15 sq. cm.
A. 73.3 sq. cm
B. 72.4 sq. cm
C. 71.7 sq. cm
D. 74.8 sq. cm
39. A circle of radius 8cm is inscribed in a sector having a central angle of 80 degrees. What is
the area of the sector?
A. 195.63 cm^2
B. 291.84 cm^2
C. 321.47 cm^2
D. 475.42 cm^2
40. A triangular piece of land has one side measuring 2k. The land is to be divided into two equal
areas by a dividing line parallel to the given side. What is the length of the dividing line?
A. 6
B. 8.485
C. 7.623
D. 8

139
41. A piece of wire having a total length of 72 cm was cut into two unequal segments and bent to
from two unequal squares. If the total area of the squares is 180 sq. cm, what is the difference in
the lengths of the two segments?
A. 24cm
B. 16cm
C. 32cm
D. 28cm
42. Determine the area of a regular hexagon inscribed in a circle having an area of 170 square
centimeters.
A. 169.8
B. 124.1
C. 148.2
D. 140.6
43. Three circles of radii 110, 140, and 220 are tangent to one another, What is the area of the
triangle formed by joining the centers of the circles?
A. 39904
B. 25476
C. 32804
D. 47124
44. A circle with area 254469 sq. cm is circumscribed about a triangle whose area is 48.23
square cm. If one side of the triangle measures 18 cm, determine length of the shorter leg of the
triangle in cm.
A. 3.625
B. 4.785
C. 8.652
D. 5.643
45. A road is tangent to a circular lake. Along the road and 12 miles from the point of tangency,
another road opens towards the lake. From the intersection of the two roads to the periphery of
the lake, the length of the new road is 11 miles. If the new road will be prolonged across the lake,
find the length of the bridge to be constructed.
A. 2.112 mi
B. 2.091 mi
C. 2.103 mi
D. 2.512 mi
46. The center of two circles with radii of 3m and 5 m, respectively are 4 m apart. Find the area
of the portion of smaller circle outside the larger circle.
A. 11.25 m^2
B. 12.15 m^2
C. 9.75 m^2
D. 10.05 m^2

140
47. Find the area in square centimeter of the largest square that can be cut from a sector of a
circle radius 8cm and central angle 120 degrees.
A. 21.9
B. 45.2
C. 33.5
D. 54.8
48. A circle is inscribed in a square and circumscribed about another. Determine the ratio of the
area larger square to the area of the smaller square.
A. 2:1
B. 1:1
C. 1:4
D. 4:1
49. One side of a rectangle, inscribed in a circle of diameter 17cm, is cm. Find the length of the
other side.
A. 16cm
B. 15cm
C. 14cm
D. 13cm
50. The sum of the sides of two polygons is 12 and the sum of their diagonals is 19. The
polygons are:
A. Pentagon and heptagon
B. Both hexagon
C. Quadrilateral and octagon
D. Triangle and nonagon

141
PROBLEM SET 14: PRISMS, PYRAMIDS,
CYLINDERS, CONES
1. If the edge of a cube is doubled, which of the following is incorrect?
A. The lateral area will be quadrupled
B. The volume is increased 8 times
C. The diagonal is doubled
D. The weight is doubled
2. The volume of a cube is reduced by how much it all side are halved?
A. 1/8
B. 5/8
C. 6/8
D. 7/8
3. Each side of a cube is increased by 1%. By what percent is the volume of the cube increased?
A. 23.4%
B. 33.1%
C. 3%
D. 34.56%
4. If the edge of a cube is increased by 30% by how much is the surface area increased?
A. 67
B. 69
C. 63
D. 65
5. Find the approximate change in the volume of a cube of side x inches caused by increasing its
side by 1%.
A. 0.3x^3 cu.in
B. 0.1x^3 cu.in
C. 0.02 cu.in
D. 0.003x^3 cu.in
6. A rectangular bin 4 feet long, 3 feet wide, and 2 feet high is solidly packed with bricks whose
dimensions are 8 in by 4 in. The number of bricks in the bins is:
A. 68
B. 368
C. 648
D. 956
7. Find the total surface area of a cube of side 6cm.
A. 214 sq.cm
B. 216 sq.cm

142
C. 226 sq.cm
D. 236 sq.cm
8. The space diagonal of a cube of 4 square root of 3m. Find its volume.
A. 16 cubic meters
B. 48 cubic meters
C. 64 cubic meters
D. 86 cubic meters
9. A reservoir is shaped like a square prism. If the area of its base is 225 sq.cm, how many liters
of water will it hold?
A. 3.375
B. 3375
C. 33.75
D. 337.5
10. Find the angle formed by the intersection of a face diagonal to the diagonal of a cube drawn
from the same vertex.
A. 35.26 degrees
B. 32.56 degrees
C. 33.69 degrees
D. 42.23 degrees
11. The space diagonal of a cube (the diagonal joining two non-coplanar vertices) is 6m. The
total surface area of a cube is:
A. 60
B. 66
C. 72
D. 78
12. The base edge of a regular hexagonal prism is 6cm and its base are 12cm apart. Find its
volume in cu .cm.
A. 1563.45 cm^3
B. 1058.45 cm^3
C. 1896.37 cm^3
D. 1122.37 cm^3
13. The base edge of a regular pentagonal prism is 6cm and its bases are 12cm apart. Find its
volume in cu.cm.
A. 743.22 cm^3
B. 786.89 cm^3
C. 587.45 cm^3
D. 842.12 cm^3
14. The base of a right prism is a hexagon with one side 6 cm long. If the volume of the prism is
450 cc, how far apart are the bases?

143
A. 5.74cm
B. 3.56cm
C. 4.11cm
D. 4.81cm
15. A trough has an open top 0.30m by 6m and closed vertical end which are equilateral triangles
30 cm on each side. It is filled with water to half its depth. Find the volume of the water in cubic
meters.
A. 0.058
B. 0.046
C. 0.037
D. 0.065
16. Determine the volume of a right truncated prism with the following dimensions: Let the
corner of the triangular base be defined by A, B and C. The length AB = 10 feet, BC = 9 feet,
and CA = 12 feet. The sides at A, B and C are perpendicular to the triangular base and have the
height of 8.6 feet, 7.1 feet, and 5.5 feet, respectively.
A. 413 ft^3
B. 311 ft^3
C. 313 ft^3
D. 391 ft^3
17. The volume of a regular tetrahedron of side 5cm is:
A. 13.72 cu.cm
B. 14.73 cu.cm
C. 15.63 cu.cm
D. 17.82 cu.cm
18. A regular hexagonal pyramid whose base perimeter is 60 cm has an altitude of 30 cm. The
volume of the pyramid is:
A. 2958 cu.cm.
B. 2598 cu.cm.
C. 2859 cu.cm.
D. 2589 cu.cm.
19. A frustum of a pyramid has an upper base 100m by 10m and a lower base of 80m by 8m. If
the altitude of the frustum is 5m, find its volume.
A. 4567.67 cu. m.
B. 3873.33 cu. m.
C. 4066.67 cu. m.
D. 2345.98 cu. m.
20. The altitude of the frustum of a regular rectangular pyramid is 5m the volume is 140 cu.m
and the upper base is 3m by 4m. What are the dimension of the lower base in m?
A. 9 x 10
B. 6 x 8
144
C. 4.5 x 6
D. 7.50 x 10
21. The frustum of a regular triangular pyramid has equilateral triangles for its bases. The lower
and upper base edges are 9m and 3m, respectively. If the volume is 118.2 cu.m.., how far apart
are the base?
A. 9m
B. 8m
C. 7m
D. 10m
22. A cylindrical gasoline tank, lying horizontally, 0.90m. in diameter and 3m long is filled to a
depth of 0.60m. How many gallons of gasoline does it contain?
A. 250
B. 360
C. 300
D. 270
23. A closed cylindrical tank is 8 feet long and 3 feet in diameter. When lying in a horizontal
position, the water is 2 feet deep. If the tank is in the vertical position, the depth of water in the
tank is:
A. 5.67m
B. 5.82m
C. 5.82ft
D. 5.67ft
24. A circular cylindrical is circumscribed about a right prism having a square base one meter on
an edge. The volume of the cylinder is 6.283 cu. m. Find its altitude in m.
A. 5
B. 4.5
C. 69.08
D. 4
25. If 23 cubic meters of water are poured into a conical vessel, it reaches a depth of 12cm. How
much water must be added so that the depth reaches 18cm?
A. 95 cubic meters
B. 100 cubic meters
C. 54.6 cubic meters
D. 76.4 cubic meters
26. The height of a right circular cone with circular base down is h. If it contains water to a depth
of 2h/3 the ratio of the volume of water to that of the cone is:
A. 1:27
B. 2:3
C. 8:27
D. 26:27
145
27. A right circular cone with an altitude of 9m is divided into two segments, one is a smaller
circular cone having the same vertex with an altitude of 6m. Find the ratio of the volume of the
two cones.
A. 19:27
B. 2:3
C. 1:3
D. 8:27
28. A conical vessel has a height of 24 cm, and a base diameter of 12 cm. It holds water to a
depth of 18 cm. above its vertex. Find the volume of its content in cc.
A. 387.4
B. 381.7
C. 383.5
D. 385.2
29. A right circular cone with an altitude of 8cm is divided into two segments. One is smaller
circular cone having the same vertex with volume equal to ¼ of the original cone. Find the
altitude of the smaller cone.
A. 4.52cm
B. 6.74cm
C. 5.04cm
D. 6.12cm
30. The slant height of a right circular cone is 5m long. The base diameter is 6m. What is the
lateral area in sq. m?
A. 37.7
B. 47
C. 44
D. 40.8
31. A right circular cone has a volume of 128 (pi)/3cm^3 and an altitude of 8cm.
The lateral area is:
A. 16 square root of 5*(pi)(cm^2)
B. 12 square root of 5*(pi)(cm^2)
C. 16(pi)cm^2
D. 15(pi)cm^2
32. The volume of a right circular cone is 36(pi). If its altitude is 3, find its radius.
A. 3
B. 4
C. 5
D. 6
33. A cone and hemisphere share base that is a semicircle with radius 3 and the cone is inscribed
inside the hemisphere. Find the volume of the region outside the cone and inside the hemisphere.

146
A. 24.874
B. 27.284
C. 28.274
D. 28.724
34. A cone was formed by rolling a thin sheet of metal in the form of a sector of a circle 72 cm in
diameter with a central angle of 210 degrees. What is the volume of the cone?
A. 13602cc
B. 13504cc
C. 13716cc
D. 13318cc
35. A cone was formed by rolling a thin sheet of metal in the form of a sector of a circle 72 cm in
diameter with a central angle of 150 degrees. Find the volume of the cone in cc.
A. 7733
B. 7722
C. 7744
D. 7711
36. A chemist’s measuring glass is conical in shape. If it is 8cm deep and 3cm across the mouth,
find the distance on the slant edge between the marking for 1 cc and 2 cc.
A. 0.82cm
B. 0.79cm
C. 0.74cm
D. 0.92cm
37. The base areas of a frustum of a cone are 25 sq. cm and 16 sq. cm, respectively. If its altitude
is 6cm, find its volume.
A. 120 cm^3
B. 122 cm^3
C. 129 cm^3
D. 133 cm^3
38. How far from a vertex is the opposite face of a tetrahedron if an edge is 50 cm long
A. 38.618 cm
B. 40.825 cm
C. 39.421 cm
D. 41.214 cm
39. A truncated prism has a horizontal triangular base ABC, AB=10cm, BC = 12cm and CA
=8cm. The vertical edges through A, B, and C are 20 cm, 12cm, and 18cm long respectively.
Determine the volume of the prism, in cc.
A. 661
B. 559
C. 685
D. 574
147
40. A lateral edge of the frustum of a regular pyramid is 1.8m long. The upper base is a square
1m by 1m and the lower base 2.4m square. Determine the volume of the frustum in cubic meters.
A. 4.6
B. 3.3
C. 5.7
D. 6.5
41. A solid spherical steel ball 20 cm in diameter is placed into a tall vertical cylinder containing
water, causing the water level to rise by 10 cm. What is the radius of the cylinder?
A. 12.14
B. 9.08
C. 10.28
D. 11.55
42. A conical vessel one meter diameter at the top and 60 cm high holds salt at a depth of 36cm
from the bottom. How many cc of salt does it contain?
A. 37214
B. 33929
C. 35896
D. 31574
43. An open-top cylindrical tank is made of a metal sheet having an area of 43.82 square meter.
If the diameter is 2/3 the height, what is the height of the tank?
A. 3.24m
B. 2.43m
C. 4.23m
D. 5.23m
44. The lateral area of a right circular cone is 386 square centimeters. If its diameter is one-half
its altitude, determine its altitude in centimeters.
A. 24.7
B. 17.4
C. 18.9
D. 22.5
45. The surface area of a regular tetrahedron is 173.2 square centimeters. What is its altitude?
A. 8.2cm
B. 9.6cm
C. 7.2
D. 6.5
46. A cube edge 25 cm is cut by a plane containing two diagonally opposite edges of the cube.
Find the area of the section thus formed in sq.cm
A. 812.3
B. 912.7

148
C. 841.2
D. 883.9
47. A swimming pool is rectangular in shape length 12 m and width 5.5m. It has a sloping
bottom and is 1m deep at one end and 3.6m deep at the other end. The water from a from a full
cylindrical reservoir 3.6m in diameter and 10 m deep is emptied to the pool. Find the depth of
water at the deep end.
A. 2.912m
B. 2.695m
C. 2.842m
D. 2.754m
48. Two vertical conical tanks (both inverted) have their vertices connected by a short horizontal
pipe. One tank initially full of water, has an altitude of 6m and a diameter of base 7m. The other
tank, initially empty, has an altitude of 9m and a diameter of base 8m. If the water is allowed to
flow through the connecting pipe, find the level to which the water will ultimately rise in the
empty tank. (Neglect the water in the pipe.)
A. 4.254m
B. 3.257m
C. 4.687m
D. 5.151m
49. A sphere of radius 5cm and a right circular cone of radius 5cm and height 10cm stand on a
plane. How far from the base of the cone must a cutting plane ( parallel to the base of the cone)
pass in order to cut the solids in

149
PART XI: OPTIMIZATION
PROBLEMS
(applications of the derivative)

Applied Minimum and Maximum Problems


One of the most common applications of calculus involves the determination of minimum and
maximum values. Consider how frequently you hear or read terms such as greatest profit, least
cost, least time, greatest voltage, optimum size, least size, greatest strength, and greatest distance.
Guidelines for solving applied minimum and maximum problems:
1. Identify all given quantities and all quantities to be determined. If possible, make a
sketch.
2. Write a primary equation for the quantity that is to be maximized or minimized. (A
review of several useful formulas from geometry is presented on the formula card inside
the back cover.)
3. Reduce the primary equation to one having a single independent variable. This may
involve the use of secondary equations relating the independent variables of the primary
equation.
4. Determine the feasible domain of the primary equation. That is, determine the values for
which the stated problem makes sense.
5. Determine the desired maximum or minimum value by the calculus techniques discussed.
Example Problem #1: We need to enclose a rectangular field with a fence. We have 500 feet of
fencing material and a building is on one side of the field and so won’t need fencing. Determine
the dimensions of the field that will enclose the largest area.
We will two functions. The first is that we are actually trying to optimize and the second will be
the constraint. Also in this problem, we want to maximize the area of the field and we know that
will use 500 ft of fencing material. So, the area will be the function we are trying to optimize and
the amount of fencing is the constraint. The two equations for these are,
Maximize: A = xy
Constraint: 500 = x + 2y

150
We know how to find the largest or smallest value of a function provided it’s only got a single
variable. The area function (as well as the constraint) has two variables in it and so what we
know about finding absolute extrema won’t work. However, if we solve the constraint for one of
the two variables we can substitute this into the area and we will then have a function of a single
variable.
So, let’s solve the constraint for x. Note that we could have just as easily solved for y but that
would have led to fractions and so, in this case, solving for x will probably be best.
x = 500 – 2y
Substituting this into the area function gives a function of y.
A (y) = (500 – 2y) y = 500y - 2𝑦 2
We want to find the largest value this will have on the interval [0, 250]. The limits in this interval
corresponds to taking y = 0 (i.e. no sides to the fence) and y = 250 (i.e. only two sides and no
width, also if there are two sides each must be 250 ft to use the whole 500ft).
Note that the endpoints of the interval won’t make any sense from a physical standpoint if we
actually want to enclose some area because they would both give zero area. They do, however,
give us a set of limits on y and so the Extreme Value Theorem tells us that we will have a
maximum value of the area somewhere between the two endpoints.
So, recall that the maximum value of a continuous function on a closed interval will occur at
critical points and/or end points. As we’ve already pointed out the end points in this case will
give zero area and so don’t make any sense. That means our only option will be the critical
points.
Let’s get the derivative and find the critical points.
A’ (y) = 500 – 4y
Setting this equal to zero and solving gives a lone critical point of y = 125. Plugging this into the
area gives an area of A (125) = 31250 𝑓𝑡 2 . So according to the method from Absolute Extrema
section, this must be the largest possible area, since the area at either endpoint is zero.
Finally, let’s not forget to get the value of x and then we’ll have the dimensions since this is what
the problem statement asked for. We can get the x by plugging in our y into the constraints.
x = 500 – 2 (125) = 250
The dimensions of the field that will give the largest area, the subject to the fact that we used
exactly 500 ft of fencing material, are 250 x 125.
Example Problem #2: We want to construct a box whose base length is 3 times the base width.
With the material used to build the top and bottom cost $10/𝑓𝑡 2 and the material used to build
the sides cost $6/𝑓𝑡 2 . If the box must have a volume of 50 ft³, determine the dimensions that
will minimize the cost to build the box.
We want to minimize the cost of the materials subject to the constraint that the volume must be
50 ft³. Note as well that the cost for each side is just the area of that sides times the appropriate
cost.

151
The two functions we’ll be working this time are,
Minimize: C = 10 (2lw) + 6 (2wh + 2lh) = 60𝑤 2 + 48wh
Constraint: 50 = lwh = 3𝑤 2 ℎ
Solve the constraint for.
50
h = 3𝑤 2

Plugging this into the cost gives,


120𝑤³−800
C (w) = 120w - 800𝑤 −2 = C” (w) = 120 + 1600𝑤 −3
𝑤2

Now we need the critical point(s) for the cost function. First, notice that w = 0 is not a critical
point. Clearly the derivative does not exist at w =0 but then neither does the function and
remember that values of w will only be critical points if the function also exists at that point.
Note that there is also a physical reason to avoid w = 0. We are constructing a box and it would
make no sense to have a zero width of the box.
So it looks like the only critical point will come from determining where the numerator is zero.
800 20
120𝑤 ³ - 800 = 0 => w = ³120 = ³ 3 = 1.8821

So, we’ve got a single critical point and we now have to verify that this is in fact the value that
will give the absolute minimum cost. First, we know that whatever the value of w that we get it
will have to be positive and we can see second derivative above that provided w > 0 we will have
C” (w) > 0 and so in the interval of possible optimal values the cost function will always be
concave up and so w = 1.8821 must give the absolute minimum cost.
All we need to do now is to find the remaining dimensions.
w = 1.8821
l = 3w = 3 (1.8821) = 5.6463
50 50
h = 3𝑤 2 = 3(1.8821)2 = 4.7050

Also, even though it was not asked for, the minimum cost is:
C (1.8821) = $637.60
Example Problem #3: We want to construct a box with a square base and we only have 10𝑚2
of material to use in construction of the box. Assuming that all the material is used in the
construction process determine the maximum volume that the box can have.
In this case, we want to optimize the volume and the constraint this time is the amount of
material used. We don’t have a cost here, but if you think about it, the cost is nothing more than
the amount of material used times a cost and so the amount of material and cost are pretty much
tied together. Note as well that the amount of material used is really just the surface area of the
box.

152
We want to maximize the volume and the amount of material is the constraint so here are the
equations we’ll need.
Maximize: V = lwh = 𝑤 2 ℎ
Constraint: 10 = 2lw +2wh + 2lh = 2𝑤 2 + 4wh
We’ll solve the constraint for h and plug this into the equation for the volume.
10−2𝑤2 5−𝑤 2 5−𝑤 2 1
h= = => V (w) = 𝑤 2 ( ) = 2 (5w - w³)
4w 2w 2w

Here are the first and second derivatives of the volume function.
1
V’ (w) = 2 (5 - 3𝑤 2 ) V” (w) = -3w

Note as well here that provided w > 0, which from a physical standpoint we know must true for
the width of the box, then the volume function will be concave down and so if we get a single
critical point, then we know that it will have to be the value that gives the absolute maximum.
Setting the first derivative equal to zero and solving gives us the two critical points,
5
w = ±3 = ± 1.2910

In this case, we can exclude the negative critical point since we are dealing with a length of a box
and we know that these must be positive. Do not however get into the habit of just excluding any
negative critical point. There are problems where negative critical points are perfectly valid
possible solutions.
Now, as noted above, we got a single critical point, 1.2910, and so this must be the value that
gives the maximum volume and since the maximum volume is all that was asked for in the
problem statement, the answer is then:
V (1.2910) = 2.1517m³
Note that we could also have noted here that if 0 < w <1.2910 then V’ (w) > 0 (using a test point
we have V’ (1) = 1>0) and likewise if w > 1.2910 then V’ (w) < 0 (using a test point we have V’
7
(2) = − 2 < 0) and so if we are to the left of the critical point, the volume is always increasing and
if we are to the right of the critical point, the volume is always decreasing. And so by the method
above, we can also see that the single critical point must give the absolute maximum of the
volume.
Finally, even though these weren’t asked for, here are the dimension of the box that gives the
maximum volume.
5 − 1.29102
l = w = 1.2910 h= = 1.2910
2 (1.2910)

So, it looks like in this case we actually have a perfect cube.

153
Example Problem #4: A rectangular page is to contain 24 square inches of print. The margins at
1
the top and bottom of the page are to be 12inches, and the margins on the left and right are to be
1 inch. What should the dimensions of the page be so that the least amount of paper is used?
The quantity to be minimized is area:
A = (x + 3)(y + 2)
Solution: Let A be the area to be minimized. Thus, it’s the primary equation.
A = (x + 3)(y + 2)
The secondary equation is the printed area inside the margins which is
24 = xy
24
Solving this equation for y produces y = . Substituting into the primary equation produces
x
24 72
A = (x + 3)( + 2) = 30 + 2x +
x x

Because x must be positive, you are interested only in values of A for x > 0. To find the critical
numbers, differentiate with respect to x
dA 72
= 2 − x²
dx

and note that the derivative is zero when x² = 36, or x = ±6. So, the critical numbers are x = ±6.
You do not have to consider x = -6 because it is outside the domain. The First Derivative Test
24
confirms that A is a minimum when x = 6. So, y = 6 = 4 and the dimensions of the page should
be x + 3 = 9 inches by y + 2 = 6 inches.

ANSWER THE FOLLOWING EXERCISES:


1. We have a piece of cardboard that is 14 inches by 10 inches and we’re going to cut out
the corners and fold up the sides to form a box. Determine the height of the box that will
give a maximum volume. Let the height of the box be h. So, the width/length of the
corners being cut out also h and so the vertical side will have a “new” height of 10 – 2h
and the horizontal side will have a “new” width of 14 – 2h.

2. A printer need to make a poster that will have a total area of 200 in² and will have 1 inch
margins on the sides, a 2inch margin on the top and a 1.5 inch margin on the bottom.
What dimensions will give the largest printed area?

3. We have 45 m² of material to build a box with a square base and no top. Determine the
dimensions of the box that will maximize the enclosed volume.

4. Find two positive numbers whose product is 750 and for which sum of one and 10 times
the other is a minimum.

154
Exercise Answer #1: We have a piece of cardboard that is 14 inches by 10 inches and we’re
going to cut out the corners and fold up the sides to form a box. Determine the height of the box
that will give a maximum volume. Let the height of the box be h. So, the width/length of the
corners being cut out also h and so the vertical side will have a “new” height of 10 – 2h and the
horizontal side will have a “new” width of 14 – 2h.
The constraint is simply the size of the piece of cardboard. In this case, we want to maximize the
volume. Here is the volume, in terms of h and its first derivative.
V (h) = h (14 – 2h)(10 – 2h) = 140h – 48h² + 4h³ V’ (h) = 140 – 96h + 12h²
Setting the first derivative equal to zero and solving gives the following two critical points,
12 ± 39
h= = 1.9183, 6.0817
3

We now have an apparent problem. We have two critical points and we’ll need to determine
which one is the value we need. The fact that we have two critical points means that neither the
first derivative test nor the second derivative test can be used here as they both require a single
critical point. The smallest h can be h = 0 even though this doesn’t make much sense as we won’t
get a box in this case. Also, from the 10 inch side, we can see that the largest h can be h = 5
although again, this doesn’t make much sense physically.
So, knowing that whatever h is it must be in the range 0 ≤ h ≤ 5. We can see that the second
critical point that we need to worry about is 1.9183.
Finally, since the volume is defined and continuous on 0 ≤ h ≤ 5, all we need to do is plug in the
critical points and endpoints into the volume to determine which gives the largest volume. Here
are those function evaluations.
V (0) = 0 V (1.9181) = 120.1644 V (5) = 0
So, if we take h = 1.9183, we get a maximum volume.

Exercise Answer #2: A printer need to make a poster that will have a total area of 200 in² and
will have 1 inch margins on the sides, a 2inch margin on the top and a 1.5 inch margin on the
bottom. What dimensions will give the largest printed area?
Both the constraint and the function we are going to optimize are areas. The constraint is that the
overall area of a poster must be 200 in² while we want to optimize the printed area (i.e. the area
of the poster with the margins taken out).
Let’s define the height of the poster to be h and the width of the poster to be w.
Here are the equations that we’ll be working with.
Maximize: A = (w – 2)(h – 3.5)
Constraint: 200 = wh
Solving the constraint for h and plugging into the equation for the printed area gives,

155
400 400−3.5w² 800
A’ (w) = -3.5 + = A” (w) = −
w² w² w³

From the first derivative, we have the following two critical points (w = 0 is not a critical point
because the area function does not exist there).
400
w =± = ±10.6904
3.5

However, since we’re dealing with the dimensions of a piece of paper, we know that we must
have w > 0 and so only 10.6904 will make sense.
Also notice that provided w > 0, the second derivative will always be negative and so in the
range of possible optimal values of the width, the area function is always concave down and so
we know that the maximum printed area will be at w = 10.6904 inches.
The height of the paper that gives the maximum printed area is then,
200
h = 10.6904 = 18.7084

Exercise Answer #3: We have 45 m² of material to build a box with a square base and no top.
Determine the dimensions of the box that will maximize the enclosed volume.
We need to set up the constraint and equation that we are being asked to optimize. We are told
that we have 45 m² of material to build the box and so that is the constraint. The amount of
material that we need to build the box is then,
45 = lw + 2 (lh) + 2 (wh) = w² + 2wh + 2wh = w² + 4wh
Note that because there is no top, the first term won’t have the 2 that the second and third term
have. We are being asked to maximize the volume so that the equation is,
V = lwh = w²h
Note as well that we went ahead and used fact that l = w in both of these equations to reduce the
three variables in the equation down to two variables. Now, let’s solve the constraint for h,
45−w²
h= 4w

Plugging this into the volume formula gives,


45−w² 1 1
V (w) = w² ( ) = 4w (45 - w²) = 4 (45w - w³)
4w

Finding the critical point(s) for this shouldn’t be too difficult at this point so here is that work.
1 1 45
V’ (w) = 4 (45 – 3w²) => (45 – 3w²) = 0 => w = ± 3 = ±15
4

Because we are dealing with the dimensions of a box, the negative width doesn’t make any sense
and so the only critical point that we can use here is:
w = 15

156
The second derivative of the volume function is,
3
V” (w) = − 2w

From this, we can see that the second derivative is always negative for positive w (which we will
always have for this case since w is the width of the box). Therefore, provided w is positive, V
(w) will always be concave down and so the single critical point we got must be a relative
maximum and hence must be that value that gives a maximum volume.
Now, let’s finish the problem by getting the remaining dimensions.
45−15
l = w =15 = 3.8730 h= = 1.9365
415

The final dimensions are then,


l = w = 3.8730 h= 1.9365

Exercise Answer #4: Find two positive numbers whose product is 750 and for which sum of one
and 10 times the other is a minimum.
First step, write down the equations describing this situation.
Let’s call the two numbers x and y and we are told that the product is 750 (this is the constraint
for the problem) or,
xy = 750
We are then being asked to minimize the sum of one and 10 times the other.
S = x + 10y
Note that it really doesn’t worry which is x and which is y in the sum so we simply chose the y to
be multiplied by 10.
We now need to solve the constraint for x and y (and it doesn’t really matter which variable we
solve for in this case) and plug this into the product equation.
750 750
x= => S (y) = + 10y
y y

The next step is to determine the critical points for this equation.
750 750
S’ (y) = − + 10 => − + 10 = 0 y = ±75 = 53
y2 y2

Because we are told that y must be positive , we can eliminate the negative value and so the only
value we really get out of this step is: y = 75 = 53
Just because we got a single value, we can’t just assume that this will give a minimum sum. We
need to do a quick check to see if it does give a minimum.
As discussed in notes, there are several methods for doing this, but in this case we can quickly
see that.

157
1500
S” (y) = y = y³

From this we can see that, provided we recall that y is positive, and then the second derivative
will always be positive. Therefore, S (y) will always be concave up and so single critical point
from the previous steps that we can use must be a relative minimum and hence must be the value
that gives a minimum sum.
Finally, let’s actually answer the question. We need to give both values. We already have y so
we need to determine x and that is easy to do from the constraint.
750
x = 53 = 503

The final answer is then,


x = 503 y = 53

158
PROBLEM SET 16: SPHERES,
PRISMATOID, SOLIDS OF REVOLUTION,
MISC
1. What is the surfaces area of a sphere whose volume is 36 cu.m?
e. 52.7𝑚2
f. 48.7 𝑚2
g. 46.6𝑚2
h. 54.6𝑚2
2. IF the surface area of the spheres is increased by 21%, its volume is increased by:
e. 13.31%
f. 33.1%
g. 21%
h. 30%
3. The surface area of a sphere is 4π𝑟 2 . Find the percentage increase in its diameter when the
surface area increases by 21%
e. 5%
f. 10%
g. 15%
h. 20%
4. Find the percentage increase in volume of a sphere if it’s the surface area is increased by
21%.
e. 30.2%
f. 33.1%
g. 34.5%
h. 30.9%
5. The volume of a sphere is increased by how much if its radius is increased by 20%
e. 32.6%
f. 33%
g. 44%
h. 72.8%
6. Given two spheres whose combined volume is known to be 819cu.m. If their radii are in the
ratio 3:4, what is the volume of the smaller spheres
a. 576cu.m
b. 243cu.m
c. 343cu.m
d. 476cu.m
7. How much will the surface area of a sphere be increased if its radius is increased by 5%

159
a. 25%
b. 15.5%
c. 12.5%
d. 10.25%
8. The volume of a sphere is 904.78.cu.m. Find the volume of the spherical segment of height is
4m.
a. 234.57cu.m
b. 256.58cu.m
c. 145.69cu.m
d. 124.58cu.m
9. A sphere of radius r just fits into a cylindrical container of radius r and altitude 2r. Find the
empty space in the cylinder.
a. (8/9) π𝑟 3
b. (20/27)π𝑟 3
c. (4/5)π𝑟 3
d. (2/3)π𝑟 3
10. If a solid steel ball is immersed in an eight cm. diameter cylinder, it displaces water to a
depth of 2.25cm. The radius of the ball is
a. 3cm
b. 6cm
c. 9cm
d. 12cm
11. The Diameter of two spheres are in the ratio 2.3. If the sum of their volumes is 1,260 cu.m,
the volume of the larger sphere is:
a. 972cu.m
b. 927cu.m
c. 856cu.m
d. 865cu.m
12. A hemispherical bowl of radius 10 cm is filled with water to such a depth that the water
surface area is equal to 75π sq.m
a. 625/3π𝑐𝑚3
b. 625/3π𝑐𝑚3
c. 625/3π𝑐𝑚3
d. 625/3π𝑐𝑚3
13. A water tank is in the form of a spherical segment whose base radii are 4m and 3m and
whose altitude is 6m. The Capacity of the tank in galloon is:
a. 91,011
b. 92,011
c. 95,011

160
d. 348.72
14. Find the volume of a spherical sector of altitude 3cm and radius 5cm.
a. 75π cu.cm
b. 100π cu.cm
c. 50π cu.cm
d. 25π cu.cm
15. A water tank is in the form of a spherical segment whose base radii are 4m and 3m and
whose altitude is 6m. The Capacity of the tank galloon is:
a. 91,011
b. 92,011
c. 95,011
d. 348.72
16. How far from the center of a sphere of a radius 10cm should a plane be passed so that the
ratio o the areas of two zone is 3:7
a. 3 cm
b. 4 cm
c. 5 cm
d. 6 cm
17. A 2m diameter spherical tank contains 1396 liters of water. How many liters of water must
be added for the water to reach a depth of 1.75m?
a. 2613
b. 2723
c. 2542
d. 2472
18. Find the volume of a spherical segment of a radius 10 m and altitude 5 m.
a. 654.5 cu.m
b. 659.8 cu.m
c. 675.2 cu.m
d. 680.5 cu.m
19. Find the volume of a spherical wedge of radius 10cm.
a. 425.66 sq. m.
b. 431.25 sq. m.
c. 581.78 sq. m.
d. 444.56 sq. m.
20. Determine the area of the zone of a sphere of radius 8 in and altitude 12 in.
a. 192π sq.in
b. 198 π sq.in
c. 185 π sq.in

161
d. 195 π sq.in
21. The corners of a cubical block touch the closed spherical shell that encloses it. The volume of
the box is 2744 cc. What volume in cc, inside the shell is not occupied by the block?
a. 1356𝑐𝑚3
b. 4721𝑐𝑚3
c. 3423𝑐𝑚3
d. 7623𝑐𝑚3
22. A cubical container that measures 2 inches on each side is tightly packed with 8 marbles and
is filled with water container and the adjacent marbles. All of the marbles are of the same size.
What is the volume of water in the container?
a. 0.38 cu.in
b. 2.5 cu.in
c. 3.8 cu.in
d. 4.2 cu.in
23. The volume of the water is spherical tank is 1470.265 𝑐𝑚3 . Determine the depth of water if
the tank has a diameter of 30cm.
a. 8
b. 6
c. 4
d. 10
24. The volume of water in a spherical tank having a diameter of 4m is 5.236𝑚3 . Determine the
depth of the water on the tank.
a. 1.0
b. 1.4
c. 1.2
d. 1.6
25. A mixture compound from equal parts of two liquids one white and the other black was
placed in a hemispherical bowl. The total depth of the two liquids is 6’’. After standing for a
short time the mixture separated the white liquid settling below the black. If the thickness of the
segment of the black liquid is 2’’, find the radius of the bowl in inches.
a. 7.53
b. 7.33
c. 7.73
d. 7.93
26. 20.5 cubic meters of water is inside a spherical tank whose radius is 2m. Find the height of
the water surface above the bottom tank in m.
a. 2.7
b. 2.5
c. 2.3

162
d. 2.1
27. The volume of the sphere is 36π cu. m. The surface area of this sphere in sq.m is:
a. 36 π
b. 24 π
c. 18 π
d. 12 π
28. Spherical balls 1.5cm in diameter area packed in a box measuring 6 cm by 3 cm. If as many
balls as possible are packed in the box how much free space remains in the box?
a. 28.41 cc
b. 20.47 cc
c. 29.87 cc
d. 25.73 cc
29. A solid has a circular base of radius r. Find the volume of the solid if every plane
perpendicular to a given diameter is a square.
a. 16𝑟 3 /3
b. 5𝑟 3
c. 6𝑟 3
d. 19𝑟 3 /3
30. A solid has circular base of diameter 20cm. Find the volume of the solid if every cutting
plane perpendicular to the base along a given diameter is an equilateral triangle.
a. 2514cc
b. 2107cc
c. 2309cc
d. 2847cc
31. The base of a certain solid is a triangle of base b and altitude h. If all the sections
perpendicular to the altitude of the triangle are regular hexagons, find the volume solid
a. 1/2√3 𝑏2 h
b. 2√3 𝑏2 h
c. √3 𝑏2 h/3
d. √3 𝑏2 h
32. The volume generated by the circle 𝑥 2 + 𝑦 2+4x-6y-12=0 revolved about the line in 2x-3y-
12=0 is:
a. 3242 cubic units
b. 3342 cubic units
c. 3452 cubic units
d. 3422 cubic units
33. The volume generated by the rotating the curve 9𝑥 2 + 4𝑦 2 = 36 about the line
4x+3y= 20 is:

163
a. 48π
b. 58π2
c. 42π
d. 48π2
34. Find the volume generated by revolving the area bounded by the ellipse 𝑦 2 /9+𝑥 2 /4=1
about the line x=3
a. 347.23 cu.units
b. 355.31 cu.units
c. 378.43 cu.units
d. 389.51 cu.units
35. The area in the second quadrant of the circle 𝑥 2 + 𝑦 2= 36 is revolved about the line y+10=0.
What is the volume generated?
a. 2218.6
b. 2228.8
c. 2233.4
d. 2208.5
36. A square area of edge “a” revolves about a line through one vertex making an angle 0 with an
edge and not crossing the square. Find the volume generated?
a. 3π𝑎3 (sin 0 + cos 0)
b. π𝑎3 (sin 0 + cos 0)/2
c. 2π𝑎3 (sin 0 + cos 0)
d. π𝑎3 (sin 0 + cos 0)
37. Given an ellipse whose semi-major axis is 6 ccm. And semi minor axis is 3cm. What is the
volume generated if it is revolved about the minor axis?
a. 35π cu.cm.
b. 72π cu.cm
c. 96π cu.cm
d. 144π cu.cm
38. A square hole 2” x 2” is cut through a 6-inch diameter log along its diameter and
perpendicular to its axis. Find the volume of wood that was removed.
a. 27.32 cu. in
b. 23.54 cu. in
c. 21.78 cu. in
d. 34.62 cu. in
39. Find the radius of the spherical wedge whose volume is 12cu. m. with a central angle of 1.8
radians
a. 2.36m
b. 2.73m
c. 2.52m

164
d. 2.15m
40. By using Pappus Theorem, determine the volume generated by revolving the area in the first
and second quadrants bounded by the ellipse 4𝑥 2 + 25𝑦 2 =100 and the x-axis about the x-axis.
a. 85.63
b. 93.41
c. 95.35
d. 83.78
41. Determine the volume of a spherical wedge of radius 2m and a central angle of 1.25 radians.
a. 6.67𝑚3
b. 8.64 𝑚3
c. 9.85𝑚3
d. 5.74𝑚3
42. Find the volume generated by revolving the triangle whose vertices are (2,2), (4,8), and (6,2)
about the line 3x-4y-24=0
a. 365.45
b. 498.12
c. 543.65
d. 422.23
43. A light bulb is placed at a certain distance from the surface of a spherical glove of radius
20cm, If it illuminates one third of the total surface of the globe. How far is it from the surface?
a. 30cm
b. 35cm
c. 60cm
d. 40cm
44. A conoid has a circular base of radius 25cm and an altitude of 30 cm. Find its volume in cc.
a. 32,457
b. 29452
c. 24,486
d. 18453
45. A sphere having a diameter of 30 cm is cut into 2 segments. The altitude of the first segment
is 8cm. What is the ratio of the volume of the second segment to that of the first?
a. 3.2
b. 4.7
c. 5.8
d. 2.5

165
Problem set 16: points, lines, circles
1. State the quadrant in which the coordinate (15, -2) lies.
a. I
b. IV
c. II
d. III
2. Of what quadrant is A if sec A is positive and csc A is negative?
a. III
b. I
c. IV
d. II
3. The segments from (-1,4) to (2. -2) is extended three times its own length. The terminal point
is.
a. (11,-18)
b. (11, -24)
c. (11,-20)
d. (-11,-20)
4. The midpoint of the line segment between 𝑃1 (x,y) and 𝑃2 (-2,4) is 𝑃𝑚 (2,-1). Find the
coordinate of 𝑃1.
a. (6,-5)
b. (5,-6)
c. (6,-6)
d. (-6,6)
5. Find the coordinates of the point P(2,4) with respect to the translated axis with the origin at
(1,3)
a. (1,-1)
b. (1,1)
c. (-1,-1)
d. (-1,1)
6. Find the median through 9-2,-5) of the triangle whose vertices are (-6,2), (2,-2) and (-2,-5).
a. 3
b. 4
c. 5
d. 6
7. Find the centroid of a triangle whose vertices are (2,3), (-4,6) and (2,-6).
a. (0,1)
b. (0,-1)
c. (1,0)

166
d. (-1,0)
8. Find the area of triangle whose vertices are A (-3,-1), B(5,3) and C(2,-8)
a. 34
b. 36
c. 389
d. 32
9. Find the distance between the points (4,-2) and (-5,1)
a. 4.897
b. 8.947
c. 7.149
d. 9.487
10. Find the distance between A(4,-3) and B(-2,5).
a. 11
b. 8
c. 9
d. 10
11. If the distance between the points (8,7) and (3,y) is 13, what is the value of Y?
a. 5
b. -19
c. 19 or -5
d. 5 or -19
12. The distance between the points (sinx, cosx) amd (cosx, -sinx) is:
a. 1
b. √2
c. 2 sinx cosx
d. 4 sinx cosx
13. Find the distance from the points (2,3) to the line 3x+4y+9=0
a. 5
b. 5.4
c. 5.8
d. 6.2
14. Find the distance from the point (5,-3) to the line 7x-4y-28=0
a. 2.62
b. 2.36
c. 2.48
d. 2.54
15. How far is the line 3x-4y+15=0 from the origin

167
a. 1
b. 2
c. 3
d. 4
16. Determine the distance from (5,10) to the line x-y=0
a. 3.86
b. 3.54
c. 3.68
d. 3.72
17. The two points on the lines 2x+3y+4=0 which are at distance 2 from the line 3x+4y+6=0 are:
a. (-8,-8) and (-16,16)
b. (-44,64) and (-5,2)
c. (-5.5,1) and (-5,2)
d. (64,-44) and (4,-4)
18. The intercepts form for algebraic straight line equation is:
a. a/x + y/b=1
b. y=mx+b
c. Ax+By+C=0
d. x/a + y/b =1
19. Find the slope of the line defined by y-x=5
a. 1
b. -1/2
c. ¼
d. 5+x
20. The slope of the line 3x+2y+5=0 is:
a. -2/3
b. -3/2
c. 3/2
d. 2/3
21. Find the slope of the line whose parametric equation is y=5-3t and x=2+t
a. 3
b. -3
c. 2
d. -2
22. Find the slope of the curve whose parametric equation are x=1+t and y=2t
a. 2
b. 3
c. 1

168
d. 4
23. Find the angle that the line 2y-9x-18=0 makes with the x-axis
a. 74.77°
b. 4.5°
c. 47.77°
d. 77.47°
24. Which of the following is perpendicular to the line x/3 +y/4=1?
a. x-4y-8=0
b. 4x-3y-6=0
c. 3x-4y-5=0
d. 4x+3y-11=0
25. Find the equation of the bisector of the obtuse angle between the lines 2x+y=4 and 4x+2y=7.
a. 4y=1
b. 8x=15
c. 2y=3
d. 8x+4y=6
26. The equation of the line through (1,2) and parallel to the line 3x-2y+4=0 is:
a. 3x-2y+1=0
b. 3x-2y-1=0
c. 3x+2y+1=0
d. 3x+2y-1=0
27. If the points (-3,-5), (x,y) and (3,4) lie on a straight line, which of the following is correct?
a. 3x-2y-1=0
b. 2x+3y+1=0
c. 2x+3y-1=0
d. 3x-2y-1=0
28. One line passes through the points (1,9) and (2,6), another line passes through (3,3) and (-
1,5). The acute angle between the two lines is:
a. 30°
b. 45°
c. 60°
d. 135°
29. The two straight lines 4x-y+3=0 and 8x-2y+6=0
a. Intersects at the origin
b. Are coincident
c. Are parallel
d. Are perpendicular
30. A line which passes through (5,6) and (-3,-4) has an equation of
169
a. 5x+4y+1=0
b. 5x-4y-1=0
c. 5x-4y+1=0
d. 5x+y-4=0
31. Find the equation of the line with the slope of 2 and y-intercept of -3.
a. y= -3x+2
b. y=2x-3
c. y=2/3x + 1
d. y=3x-2
32. What is the equation of the line that passes through (4,0) and is parallel to the line x-y-2=0?
a. y+x+4=0
b. y-x+4=0
c. y-x-4=0
d. y+x=4=0
33. Determine B such that 3x+2y-7=0 is perpendicular to 2x-By+2=0
a. 2
b. 3
c. 4
d. 5
34. The equation of a line that intercepts the x-axis at x=4 and the y-axis at y=6 is:
a. 2x-3y=12
b. 3x+2y=12
c. 3x-2y=12
d. 2x-37=12
35. How far from the y-axis is the center of the curve 2𝑥 2 + 2𝑦 2 +10x-6y-55=0?
a. -3.0
b. 2.75
c. -3.25
d. 2.5
36. Find the area of the circle whose center is at (2,-5) and tangent to the line 4x+3y-8=0
a. 6π
b. 9π
c. 3π
d. 12π
37. Determine the area enclose by the curve 𝑥 2 -10x+4y+𝑦 2 =196
a. 15 π
b. 225 π
c. 12 π

170
d. 144 π
38. Find the shortest distance from the point (1,2) to a point on the circumference of the circle
defined by the equation 𝑥 2 +𝑦 2 +10x+6y+30=0
a. 5.61
b. 5.71
c. 5.81
d. 5.91
39. Determine the length of the chord common to the circles 𝑥 2 +𝑦 2 =64 and 𝑥 2 +𝑦 2 -16x=0
a. 13.86
b. 12.82
c. 13.25
d. 12.28
40. If (3,-2) lies on a circle with center (-1,1) then the area of the circle is:
a. 5π
b. 25 π
c. 4π
d. 3π
41. The radius of the circle 2𝑥 2 +2𝑦 2 +-3x+4y-1=0 is:
a. √33 /4
b. 33/16
c. √33 /3
d. 17
42. What is the radius of a circle with the following equation? 𝑥 2 -6x+𝑦 2 -4y-12=0
a. 3.46
b. 5
c. 7
d. 6
43. The diameter of a circle described by 9𝑥 2 +9𝑦 2 =16 is:
a. 16/9
b. 4/3
c. 4
d. 8/3
44. Find the center of the circle 𝑥 2 + 𝑦 2 -6x+4y-23=0
a. (3,-2)
b. (3,2)
c. (-3,2)
d. (-3,-2)

171
45. Determine the equation of the circle whose center is at (4,5) and tangent to the circle whose
equation is 𝑥 2 +𝑦 2 +4x+6y-23=0.
a. 𝑥 2 +𝑦 2 -8x+10y-25=0
b. 𝑥 2 +𝑦 2 +8x-10y+25=0
c. 𝑥 2 +𝑦 2 -8x-10y+25=0
d. 𝑥 2 +𝑦 2 -8x-10y-25=0
46. The equation of the circle with center at (-2,3) and which is tangent to the line 20x-21y-42=0
is:
a. 𝑥 2 +𝑦 2 +4x-6y-12=0
b. 𝑥 2 +𝑦 2 +4x-6y+12=0
c. 𝑥 2 +𝑦 2 +4x+6y-12=0
d. 𝑥 2 +𝑦 2 -4x-6y-12=0
47. A circle has a diameter whose ends are at (-3,2) and (12,-6). Its equation is:
a. 4𝑥 2 +4𝑦 2-36x+16y+192=0
b. 4𝑥 2 +4𝑦 2-36x-+16y-192=0
c. 4𝑥 2 +4𝑦 2-36x-16y-192=0
d. 4𝑥 2 +4𝑦 2+36x+16y-192=0
48. Find the equation of the circles with center on x+y=4 and 5x+2y+1=0 and having a radius of
3.
a. 𝑥 2 +𝑦 2 +6x-16y+64=0
b. 𝑥 2 +𝑦 2 +8x-14y+25=0
c. 𝑥 2 +𝑦 2 +8x-14y+49=0
d. 𝑥 2 +𝑦 2 +6x-14y+36=0
49. If (3,-2) lies on the circle with center (-1,1) then the equation of the circle is
a. 𝑥 2 +𝑦 2 +2x-2y-23=0
b. 𝑥 2 +𝑦 2 +4x-2y-21=0
c. 𝑥 2 +𝑦 2 +2x-y-33=0
d. 𝑥 2 +𝑦 2 +4x-2y-27=0
50. Find the equation of K for which the equation 𝑥 2 +𝑦 2 + 4𝑥 − 2𝑦 − 𝑘 =
0 represents a point circle
a. 5
b. -5
c. 6
d. -6

172
part xii: related
rates
(application of the derivative)

Guidelines for solving related rate problems


1. Identify all given quantities and quantities to be determined. Make a
sketch and label the quantities.
2. Write an equation involving the variables whose rates of change either
are given or are to be determined.
3. Using the Chain Rule, implicitly differentiate both sides of the equation
with respect to time t.
4. After completing Step 3, substitute into the resulting equation all known
values for the variables and their rates of change. Then solve for the
required rate of change.

Example Problem #1: Air is being pumped into a spherical balloon at a rate of
5cm³/min. Determine the rate at which the radius of the balloon is increasing when
the diameter of the balloon is 20 cm.
Identify the information given and what to find.
V = volume t = time r = rate
V (t) and r (t)
Recall that rates of change are nothing more than derivatives and so we know that,
V’ (t) = 5
Determine the rate at which the radius is changing.
𝑑
r’ (t) = ? when r(t) = = 10 cm
2

Convert the diameter to radius.

173
Now that we’ve identified what we have been given and what to find, we need to relate
these two quantities to each other. Relate the volume and the radius with the formula
for the volume of the sphere.
4
V (t) = [r (t)] ³
3

Differentiate both sides with respect to t. We will need to do implicit differentiation on


the above formula. Doing this gives,
V’ = 4𝜋𝑟 2 r’
Plug in what we know and solve for what we want to find.
1
5 = 4𝜋 (102 ) r’ => r’ = cm/min
80

Example Problem #2: Two people are 50 feet apart. One of them starts walking North
at a rate so that the angle is changing at a constant rate of 0.01 rad/min. At what rate
is distance between the two people changing when  = 0.5 radians?
Let’s call the distance between them at any point in time x as noted above.
50 𝑥
Cos  = sec  =
𝑥 50

We want to determine x’ and we know that  = 0.5 and ’ = 0.01. So, just plug in and
solve.
(50) (0.01) sec (0.5) tan (0.5) = x’ => x’ = 0.311254ft/min

Example Problem #3: A spot light is on the ground 20ft away from a wall and a 6ft
tall person is walking towards the wall at a rate of 2.5ft/sec. How fast is the height of
the shadow changing when the person is 8 feet from the wall? Is the shadow
increasing or decreasing in height at this time?
We want to determine y’ when the person is 8ft from wall or x = 12ft. Also, if the
person is moving towards the wall at 2.5ft/sec then the person must be moving away
from the spotlight at 2.5ft/sec and so we also know that x’ = 2.5.
In this case, the equation that we are going to work with is,
𝑦 20 120
= => y=
6 𝑥 𝑥

All that we need to do is differentiate and plug values into solve to get y’.
120 120
y’ = - x’ => y’ = - (2.5) = -2.0833ft/sec
𝑋 𝑋

The height of the shadow is then decreasing at a rate of 2.0833ft/sec.

Example Problem #4: Suppose that we have two resistors connected in parallel with
resistance 𝑅1 and 𝑅2 measured in ohms (). The total resistance, R, is then given by,
1 1 1
= +
𝑅 𝑅1 𝑅2

174
Suppose that 𝑅1 is increasing at a rate of 0.4 /min and 𝑅2 is decreasing at a rate of
0.7 /min. At what rate is R changing when 𝑅1 = 80  and 𝑅2 = 105?
First, let’s note that we’re looking for R’ and that we know 𝑅′1 = 0.4 and 𝑅′2 = -0.7. Be
careful with the signs here.

Let’s determine R at the time we’re interested in.


1 1 1 37 1680
= + = => R= = 45.4054 
𝑅 80 150 1680 37

Next, we need to differentiate the equation given in the problem statement.


1 1 1
- R’ = - 𝑅′1 – 𝑅′ 2
𝑅2 (𝑅1 )2 (𝑅2 )2

1 1
R’ = 𝑅 2 ( 𝑅′ + 𝑅′2 )
(𝑅1 )2 1 (𝑅2 )2

Finally, all we need to do is plug into this and do some quick computations.
1 1
R’ = ( 45.4054)2 ( (0.4) + (−0.7) = −0.002045
802 1502

So, R is decreasing at a rate of 0.002045 /min.

ANSWER THE FOLLOWING EXERCISES:


1. In the following, assume that x, y and z are all functions of t. Given x = 4, y = -
2, x’ = 9 and y’ = -3. Determine z’ for the following equation.

x (1-y) + 5z³ = 𝑦 2 𝑧 2 + 𝑥 2 – 3
2. : A thin ice is in the form of a circle. If the ice is melting in such a way that the
area of the sheet is decreasing at a rate of 0.5 𝑚2 /sec. At what rate is the radius
decreasing when the area of the sheet is 12 𝑚2 ?

3. A pebble is dropped into a calm pond, causing ripples in the form of concentric
circles. The radius r of the outer ripple is increasing at a constant rate of 1
foot/sec. When the radius is 4 feet, at what rate is the total area A of the
disturbed water changing?

4. A plane is 750 meters in the air flying parallel to the ground at a speed of
100m/s and is initially 2.5 kilometers away from a radar station. At what rate
is the distance between the plane and the radar station changing (a) initially
and (b) 30 seconds after it passes over the radar station?

175
Exercise Answer #1: In the following, assume that x, y and z are all functions of t.
Given x = 4, y = -2, x’ = 9 and y’ = -3. Determine z’ for the following equation:
x (1-y) + 5z³ = 𝑦 2 𝑧 2 + 𝑥 2 – 3
Use implicit differentiation to differentiate the equation with respect to t.
x’ (1-y) – xy’ + 15 𝑧 2 z’ = 2y y’𝑧 2 + 2𝑦 2 z z’ + 2x x’
All we need to do now is plug in the given information and solve for y’.
45
27 + 12 + 15z’ = 12 + 8z’ + 72 => z’ =
7

Exercise Answer #2: A thin ice is in the form of a circle. If the ice is melting in such a
way that the area of the sheet is decreasing at a rate of 0.5 𝑚2 /sec. At what rate is the
radius decreasing when the area of the sheet is 12 𝑚2 ?
We’ll call the area of the sheet A and the radius r and we know that the area of a circle
is given by,
𝐴 = 𝜋𝑟 2
We know that A’ = -0.5 and want to determine r’ when A = 12.
Next step is to simply differentiate the equation with respect to t.
𝐴′ = 2𝜋rr′
To finish the problem off, we need to go back to the equation of the area and use the
fact that we know the area at the point we are interested in and determine the radius
at that time.
12
12 = 𝜋𝑟 2 => r= = 1.9544
𝜋

The rate of change of the radius is then,


-0.5 = 2 𝜋(1.9544) r’ => r’ = -0.040717

Exercise Answer #3: A pebble is dropped into a calm pond, causing ripples in the
form of concentric circles. The radius r of the outer ripple is increasing at a constant
rate of 1 foot/sec. When the radius is 4 feet, at what rate is the total area A of the
disturbed water changing?
The variables r and A are related by 𝐴 = 𝜋𝑟 2 . The rate of change of the radius r is dr/dt
= 1.
Equation: 𝐴 = 𝜋𝑟 2
𝑑𝑟
Given rate: = 1 foot per second
𝑑𝑡

176
𝑑𝐴
Find: when r = 4 feet
𝑑𝑡

Differentiate with respect to t and then do Chain Rule


𝑑 𝑑
[A] = [𝜋𝑟 2 ]
𝑑𝑡 𝑑𝑡
𝑑𝐴 𝑑𝑟
= 2 𝜋r
𝑑𝑡 𝑑𝑡
𝑑𝑟
Substitute 4 for r and 1 for and then simplify.
𝑑𝑡

= 2 𝜋(4)(1)
= 8𝜋 square feet per second
When the radius is 4 feet, the area is changing at a rate of 8 𝜋 square feet/sec.

Exercise Answer #4: A plane is 750 meters in the air flying parallel to the ground at a
speed of 100m/s and is initially 2.5 kilometers away from a radar station. At what rate
is the distance between the plane and the radar station changing (a) initially and (b)
30 seconds after it passes over the radar station?
(a) At what rate is the distance between the plane and the radar station changing
initially?

We know that x’ = -100 when x = 2500. In this case note that x’ must be negative
because x will be decreasing in this part. Also note that we converted x to meters since
all the other quantities are in meter.
Determine z’ using the Pythagorean Theorem we get the following equation to relate x
and z.
𝑧 2 = 𝑥 2 + 7502 = 𝑥 2 + 562500
Let’s differentiate this with respect to t and solve it for z’ so that the actual solution
will be quick and simple to find.
𝑥 𝑥′
2z z’ = 2x x’ => z’ =
𝑧

Reuse Pythagorean Theorem and then plug into the equation above.
z = 25002 + 7502 = 6812500 = 250 109 = 2610.0766
The rate of change of the distance between the two for this part is,
(2500)(−100)
z’ = = -95.7826
2610.0766

(b) At what rate is the distance between the plane and the radar station changing
30 seconds after it passes over the radar station?

177
Know that x’ = 100 and it will be positive in this case because x will now be increasing.
We want to determine z’ and equation we’ll need is identical to the previous part so
we’ll rewrite both and its derivative.
𝑧 2 = 𝑥 2 + 7502 = 𝑥 2 + 562500
𝑥 𝑥′
2z z’ = 2x x’ => z’ =
𝑧

We need to determine both x and z. For x, we know the speed of the plane and the fact
that it has flown for 30 seconds after passing over the radar station. So x is,
x = (100)(30) = 3000
For z, we need to reuse the Pythagorean Theorem.
z = 30002 + 7502 = 9562500 = 750
The rate of change of the distance between the two for this part is then,
(3000)(100)
z’ = = 97.0143
3092.3292

178
PROBLEM SET 17: PARABOLA, ELLIPSE,
HYPERBOLA, POLAR, SPACE
1. The vertex of the parabola 𝑦 2 -2x+6y=3=0 is at:
a. (-3,3)
b. (3,3)
c. (-3,3)
d. (-3,-3)
2. The length of the latus rectum of the parabola 𝑦 2 =4px is:
a. 4p
b. 2p
c. P
d. -4p
3. Given the equation of the parabola: 𝑦 2 − 8𝑥 − 20 = 0. The length of its latus rectum is:
a. 2
b. 4
c. 6
d. 8
4. What is the length of the latus rectum of the curve 𝑥 2 = 16x.
a. 12
b. -3
c. 3
d. -12
5. Find the equation of the directrix of the parabola 𝑦 2 = 16x:
a. X=8
b. X=4
c. X=-8
d. X=-4
6. The curve y= -𝑥 2 +x+1 opens:
a. Upward
b. To the left
c. To the right
d. Downward
7. The parabola y=−𝑥 2 +x+1 opens
a. To the right
b. To the left
c. Upward
d. Downward

179
8. Find the equation of the axis of symmetry of the function y=2𝑥 2 -7x+5.
a. 4x+7=0
b. x-2=0
c. 4x-7=0
d. 7x+4=0
9. Find the equation of the locus of the center of the circle which moves so that it is tangent to
the y-axis and to the circle of radius one(1) with center at (2,0).
a. 𝑥 2 +𝑦 2 -6x+3=0
b. 𝑥 2 − 6𝑥 + 3 = 0
c. 2𝑥 2 +𝑦 2 -6x+3=0
d. 𝑦 2 -6x+3=0
10. Find the equation of the parabola with vertex at (4,3) and focus at (4,-1).
a. 𝑦 2 -8x+16y-32=0
b. 𝑦 2 +8x-16y-32=0
c. 𝑥 2 +8x-16+32=0
d. 𝑥 2 -8x+16y-32=0
11. Find the area bounded by the curves 𝑥 2 +8y+16=0, x-4=0, the a-axis and the y-axis.
a. 10.67sq. units
b. 10.33sq. units
c. 9.67sq. units
d. 8sq. units
12. Find the area (in sq.units) bounded by the parabolas 𝑥 2 -2y=0 and 𝑥 2 +2y-8=0
a. 11.7
b. 10.7
c. 9.7
d. 4.7
13. The length of the latus rectum of the curve (𝑥 − 2)2 /4 + (𝑦 + 4)2 / 25=1 is:
a. 1.6
b. 2.3
c. 0.80
d. 1.52
14. Find the length of the latus rectum of the following ellipse:
25𝑥 2 +9𝑦 2 -300x-144y+1251=0
a. 3.4
b. 3.2
c. 3.6
d. 3.0

180
15. If the length of the major and minor axes of an ellipse is 10cm and 8cm, respectively, what is
the eccentricity of the ellipse.
a. 0.50
b. 0.60
c. 0.70
d. 0.80
16. The eccentricity of the ellipse 𝑥 2 /4 + 𝑦 2 /16 =1 is:
a. 0.725
b. 0.256
c. 0.689
d. 0.866
17. An ellipse has the equation 16𝑥 2 + 9𝑦 2 +32x-128=0. Its eccentricity is :
a. 0.531
b. 0.66
c. 0.824
d. 0.93
18. The center of the ellipse is 4𝑥 2 +𝑦 2 -16x-6y-43=0 is at:
a. (2,3)
b. (4,-6)
c. (1,9)
d. (-2,-5)
19. Find the ratio of the major axis to the minor axis of the ellipse:
9𝑥 2 +4𝑦 2 -24y-72x-144=0
a. 0.67
b. 1.8
c. 1.5
d. 0.75
20. The area of the ellipse 9𝑥 2 + 25𝑦 2 -36x-189=0 is equal to:
a. 15π sq. units
b. 20 π sq. units
c. 25 π sq. units
d. 30 π sq. units
21. The area of the ellipse is given as A= 3.1216 a b. Find the area of the ellipse 25𝑥 2 +16𝑦 2 -
100x+32y=284.
a. 86.2 square units
b. 62.8 square units
c. 68.2 square units
d. 82.6 square units

181
22. The semi major axis of an ellipse is 4 and its semi minor axis is 3. The distance from the
center to the directrix is:
a. 6.532
b. 6.047
c. 0.6614
d. 6.222
23. Given ellipse 𝑥 2 / 36 + 𝑦 2 /32=1. Determine the distance between foci.
a. 2
b. 3
c. 4
d. 8
24. How far apart are the directrices of the curve 25𝑥 2 +9𝑦 2 -300x-144y+1251=0?
a. 12.5
b. 14.2
c. 13.2
d. 15.2
25. The major axis of the elliptical path in which the earth moves around the sun is
approximately 186,000,000 miles and the eccentricity of the ellipse is 1/60. Determine the
apogee of the earth.
a. 94,550,000 miles
b. 94,335,100 miles
c. 91,450,000 miles
d. 93,000,000 miles
26. Find the equation of the ellipse whose center is at (-3, -1), vertex at (2,-1) and focus at (1,-1).
a. 9𝑥 2 + 36𝑦 2 -54x+50y-116=0
b. 4𝑥 2 + 25𝑦 2 +54x-50y-122=0
c. 9𝑥 2 +25𝑦 2 +50x+50y+109=0
d. 9𝑥 2 +25𝑦 2 +54x+50y-119=0
27. Point P(x,y) moves with a distance from point (0,1) one half of its distance from line y=4, the
equation of its locus is:
a. 4𝑥 2 +3𝑦 2 =12
b. 2𝑥 2 -4𝑦 2 =5
c. 𝑥 2 + 2𝑦 2 =4
d. 2𝑥 2 + 5𝑦 2 =3
28. The chords of the ellipse 64𝑥 2 +25𝑦 2 =1600 having equal slopes of 1/5 are bisected by its
diameter. Determine the equation of the diameter of the ellipse.
a. 5x-64y=0
b. 64x-5y=0
c. 5x+64y=0
182
d. 64x+5y=0
29. Find the equation of the upward asymptote of the hyperbolawhose equation is (𝑥 − 2)2 /9 –
(𝑦 + 4)2 /4=1 is:
a. 3x+4y-20=0
b. 4x-3y-20=0
c. 4x+3y-20=0
d. 3x-4y-20=0
30. The semi conjugate axis of the hyperbola is 𝑥 2 /9 - 𝑦 2 /4 = 1 is:
a. 2
b. -2
c. 3
d. -3
31. What is the equation of the asymptote of the hyperbola 𝑥 2 /9- 𝑦 2 /4=1?
a. 2x.3y=0
b. 3x-2y=0
c. 2x-y=0
d. 2x+y=0
32. The graph y=(x-1) ? (x+2) is not defined at:
a. 0
b. 2
c. -2
d. 1
33. The equation 𝑥 2 +Bx+𝑦 2 +Cy+D=) is:
a. Hyperbola
b. Parabola
c. Ellipse
d. Circle
34. The general second degree equation has the form A𝑥 2 + Bxy+C𝑦 2 + Dx+Ey+F=0 and
describes an ellipse if:
a. 𝐵2 -4AC=0
b. 𝐵2 − 4𝐴𝐶 > 0
c. 𝐵2 -4AC=1
d. 𝐵2 - 4AC<0
35. Find the equation of the tangent to the circle 𝑥 2 +𝑦 2 -34=0 through point (3,5).
a. 3x+5y—34=0
b. 3x-5y-34=0
c. 3x+ 5y+34=0
d. 3x-5y+34=0

183
36. Find the equation of the tangent to the curve 𝑥 2 + 𝑦 2 + 4x + 16y-32=0 through (4,0)
a. 3x-4y+12=0
b. 3x-4y-12=0
c. 3x+4y+12=0
d. 3x+4y-12=0
37. Find the equation of the normal curve 𝑦 2 +2x+3y=0 through (-5,2)
a. 7x+2y+39=0
b. 7x-2y+39=0
c. 2x-7y-39=0
d. 2x+7y-39-0
38. Determine the equation of the line tangent to the graph y=2𝑥 2 +1 at the point (1,3).
a. y=4x+1
b. y=4x-1
c. y=2x-1
d. y=2x+1
39. Find the equation of the tangent to the curve 𝑥 2 +𝑦 2 =41 through (5,4)
a. 5x+4y=41
b. 4x-5y=41
c. 4x+5y=41
d. 5x-4y=41
40. Find the equation of a line normal to the curve 𝑥 2 =16y at (4,1)
a. 2x-y-9=0
b. 2x-y+9=0
c. 2x+y-9=0
d. 2x+y+9=0
41. What is the equation of the tangent to the curve 9𝑥 2 + 25𝑦 2 -225=0 at (0,3)?
a. y+3=0
b. x+3=0
c. x-3=0
d. y-3=0
42. What is the equation of the normal curve to the 𝑥 2 + 𝑦 2 = 25 at (4,3)?
a. 3x-4y=0
b. 5x+3y=0
c. 5x-3y=0
d. 3x-+4y=0
43. The polar form of the equation 3x+4y-2=0 is:
a. 3r sin0 + 4r cos0=2
b. 3r cos0 + 4r sin0=-2
184
c. 3r cos0 + 4r in0=2
d. 3r sin- + 4r tan0= -2
44. The polar form of the equation 3𝑥 2 + 2𝑦 2 = 8 is:
a. 𝑟 2 =8
8
b. R= 𝑐𝑜𝑠2 0+2
c. R=8
d. 𝑟 2 =8/𝑐𝑜𝑠 2 0+2
45. The distance between points (5,30°) and (-8,-50°) is:
a. 9.84
b. 10.14
c. 6.13
d. 12.14
46. Convert 0=π/3 to cartesian equation
a. x=√3 x
b. y=x
c. 3y= √3 x
d. Y= √3 x
47. The point of intersection of the planes x+5y-2z=9, 3x-2y =z=3 amd x+y +z=2 is:
a. (2,1,-1)
b. (2,0,-1)
c. (-1,1,-1)
d. (-1,2,1)
48. A warehouse roof needs a rectangular skylight with vertices (3,0,0), (3,3,0), (0,3,4) and
(0,0,4). If the units are in meter, the area of the light is:
a. 12sq. m
b. 20sq. m
c. 15sq. m
d. 9sq. m
49. The distance between points in space whose coordinates are (3,4,5) and (4,6,7) is:
a. 1
b. 2
c. 3
d. 4
50. What is the radius of the sphere with center at origin and which passes through the point
(8,1,6)?
a. 10
b. 9
c. √101

185
d. 10.5
51. Points C (5,7) and D (4,1,6) are 7.28 cm apart. Find the value of z.
a. 3cm
b. 4cm
c. 2cm
d. 1cm
52. What is the total length of the curve r=4 sin0?
a. 8π
b. π
c. 2π
d. 4π
53. A triangle have vertices at A(-3,-2) , B(2,6) and C (4,2). What is the abscissa of the centroid
of the triangle?
a. 3/4
b. 5/4
c. 3/2
d. 1
54. What is the distance between the vertices of the following ellipse 64𝑥 2 + 25𝑦 2 + 16x-16y-
648=0?
a. 6.324
b. 12.54
c. 10.21
d. 5.105
55. Determine the equation of the curve such that the sum of the distances of any point of the
curve from two points whose coordinates are (-3,0) and (3,0) is always equal to 8
a. 4𝑥 2 + 49𝑦 2 -343=0
b. 7𝑥 2 +16𝑦 2 -122=0
c. 7𝑥 2 +16𝑦 2 -112=0
d. 7𝑥 2 +16𝑦 2 -112=0
56. Find the volume of the tetrahedron bounded by the coordinate planes and the plane
8x+12y+4z-24=0
a. 5
b. 9
c. 6
d. 12
57. The distances from the focus to the vertices of an ellipse are 4 and 6 units. Determine the
ellipse flatness.
a. 0.0202

186
b. 0.206
c. 0.0312
d. 0.0187
58. If the length of the latus rectum of an ellipse is three fourth of the length of the minor axis
determine its eccentricity.
a. 0.775
b. 0.332
c. 0.661
d. 0.553
59. Transform r= 3/3+3cos0 into cartesian coordinates.
a. 5𝑥 2 -9𝑦 2 +12x+9=0
b. 5𝑥 2 +9𝑦 2 -12x-9=0
c. 5𝑥 2 + 9𝑦 2 +12x+9=0
d. 5𝑥 2 + 9𝑦 2 +12x-9=0
60. Find the polar coordinates for the point whose rectangular coordinate of (-6,-8).
a. (10, -233.23°)
b. (10,233.23°)
c. (10,126.87°)
d. 10,-53.13°)

187
PROBLEM SET 18: RECENT BOARD
EXAMS
1. Solve the system:xy = 24 y - 2x + 2 = 0
2. Solve the equation √𝑥 2 - √3𝑥 = 2x – 6
3. √8 + √18 − 7 √2 =
4. If -9 < x < -4 and -12 < y <-6, then
5. What is the value of E in the following equation?
2𝑥 4 +3𝑥 3+ 7𝑥 2 +10𝑥 𝐴 𝑏𝑥+𝑐 𝐷𝑥+𝑐
= 𝑥−1 + + (𝑥2 +3)2
(𝑥−1)(𝑥 2 +3)2 𝑥 2 +3

a) 2
b) 3
c) 0
d) -1
6. If 3𝑥 = 54, then 3𝑥.2 is equal to
a) 6
b) 5
c) 8
d) 9
7. Which of the followeing is the value of 𝑥 2 + 1/𝑥 2 , when x+1/x = 5?
a) 28
b) 23
c) 24
d) 25
8. Evaluate the following Logarithm log √2/8
a) 2.5
b) 5.2
c) -2.5
d) -5.2
9. Given log10 2 = 0.3010, find log10 32.
a) 1.863
b) 0.256
c) 1.365
d) 1.505
1
10. Given: log(2𝑥 − 3) = 2. What is the value of x if the base of the algorithm is 9?
a) 1
b) 2
c) 3
d) 4
11. If log 8 𝑁 = 2/3, the N is equal to

188
a) 4
b) 2
c) 3
d) 5
12. What is the value of x if log(base x) 1296 = 4?
a) 5
b) 4
c) 3
d) 6
13. Solve for x if 2 log x – log(30 – 2x)= -1.
a) 12
b) 10
c) -30
d) 15
14. If log 6 2 = 0.69 and log 6 3 = 1.10, find log 6 6.
a) 1.7836
b) 1.5698
c) 1.1236
d) 2.1475
15. If log 𝑥 6 = 1.2925, 𝑤ℎ𝑎𝑡 𝑖𝑓 𝑡ℎ𝑒 𝑣𝑎𝑙𝑢𝑒 𝑜𝑓 log 𝑥 11?
a) 1.65
b) 1.73
c) 1.42
d) 1.51
16. What is the product (AB) of the following complex numbers? Express your answer in
polar form A = 3 + j4 B = 8 + j6
a) 50∠90 ͦ
b) 50∠30 ͦ
c) 50∠45 ͦ
d) 50∠60 ͦ
17. Add the following complex numbers; 3+4i and 2-5i.
a) 12 + 2i
b) 24 - 5i
c) 6 - i
d) 5 -i
18. Find the function defined as f(x) = 1/ 1 + x. For what value of x is f(f(x)) undefined?
a) (-1, -1/2)
b) (-1, -2)
c) (-1, 0)
d) (0)
19. If f(x)= 2x² + 2,find the value of f(x + 4)
a) 2x^2 + 16x + 24
b) 2x^2 + 16x

189
c) 2x^2+ 16x + 4
d) 2x^2 + 16x + 34
20. If f(x)= (3√𝑥 − 4) ², then how much f(x) increase as x goes from 2 to 3?
a) 1.372
b) 1.732
c) 1.273
d) 1.723
21. If f(x)=x^3 –x -1, what is the best set of all c if f(c) = f(-c)?
a) All real numbers
b) (-1, 0, 1)
c) (0)
d) (0, 1)
22. If f(x) = 3x -1, then f^-1(x) is equal to:
a) x-1/3
b) 3x + 1
c) 1/3x – 1/3
d) 1/3x - 1
23. If f(x) = x-2x² - 2, what is f(a-2)?
a) 9a - 2a² - 8
b) 9a – 2a² + 8
c) 2a – 9a² - 8
d) 2a – 9a² - 8
24. If f(x) = 3x + 2 and g(f(x)) = x, then g(x) is equal to
a) x – 2/3
b) 4x + 9
c) 1/3(x - 2)
d) 3x - 2
25. Solve for x in the following equation: x + 4x + 7x + 10x... + 64x = 1430
a) 3
b) 4
c) 5
d) 2
26. Four positive integers form arithmetic progression. If the product of the first and lasr
term is 70 and the second and third term is 88, what is the first term?
a) 3
b) 14
c) 5
d) 8
27. What is the value of x if 1, ¼, 1/x, 1/10,... form a harmonic progression
a) 6
b) 7
c) 8
d) 5

190
28. Fin dthe sum of the infinite progression 2^-1, 2^-3, 2^-5,...
a) ¾
b) ½
c) 2/3
d) 1/3

29. The sides of a right triangle are arithmetic progression whose common differenc is 6.
Find the hypotenuse.
a) 6
b) 30
c) 18
d) 24
30. If 3, 5, 8.333, and 13.889 are the first four terms of a sequence, then which of the
following could define a sequence?
a) 𝐴𝑞 = 3; 𝐴𝑛 = 𝐴𝑛−1 + 40/9
b) 𝐴𝑞 = 3; 𝐴𝑛+1 = 𝐴𝑛+2
c) 𝐴𝑞 = 3; 𝐴𝑛 = 5/3 𝐴𝑛−1
d) 𝐴𝑞 = 3; 𝐴𝑛+1 = 2𝐴𝑛+2
31. Two numbers differ by 40 and their arithmetic mean exceeds their geometric mean by 2.
What is the smaller number?
a) 81
b) 96
c) 64
d) 45
32. If kx³ - (k + 3)x² + 13 is divided by x – 4, the remainder is 157. What is the value of k?
a) 5
b) 4
c) 3
d) 7
33. The average rate of production of PCB is one (1) unit for every 2 hours work by two
workers. How many PCBs can be produced in 1 month by 80 workers working 200 hours
during the month?
a) 2000
b) 5000
c) 3000
d) 4000
34. A can do a job 50% faster than B and 20% faster than C. working altogether, they can
finish the job in 4 days. How many days will it take A to finish th job if he wrk alone?
a) 16
b) 18
c) 10
d) 12
191
35. If john can paint the room in 30 minutes and tom can paint tit in 1 hour, how many
minutes will takethem to paint the room if thery work together.
a) 15
b) 20
c) 12
d) 10
36. A sales man started walking from office A at 9:30 AM at the rate of 2.5 kph. He arrived
office B 12seconds late. Had he started at A at 9:00 am and walked at 1.5 kph, he would
have arrived at B one minutebefore the required time. At what time was he supposed to
be at B?
a) 10:13 am
b) 10:16 am
c) 10:22 am
d) 1:18 am
37. A man walk at a certain distance at 5 kph and returns at a rate of 4 kph. If the total time
that is takes him is 3 hours and 36 minutes, what is the total distance hat he walked?
a) 8
b) 9
c) 18
d) 16
38. John can shovel adriveway in 50 mins. If mary can shovel the driveway in 20 mins, how
long will it take them, to the nearest minute, to c=shovel the driveway if they work
together?
a) 12
b) 13
c) 14
d) 16
39. Carpenter pedro can make a bookshelf in 5 working days alone. Carpenter juan can make
the same bookshelf in 10 days working alone. How long will it take them to make a
bookshelf if they work together?
a) 4 – 1/3 days
b) 3 – 2/3 days
c) 3 – 1/3 days
d) 2 – 2/3 days
40. At exactly what time after 2 oclock will the hour hand and the minute hand of a
contiuously driven clock extend in the opposite direction for the next time?
a) 2:43;6.8
b) 2:43;58.1
c) 2:43;12.2
d) 2:43;38.2
41. A speedboat can make a trip of 100km in 1 hour and 30 mins if it travels upstream. If it
travels downstream, it will take 1 hour and 15 mins to travel the same distance. What is
the speed of the boat in calm water?

192
a) 80 kph
b) 123.33 kph
c) 66.67 kph
d) 73.33 kph
42. Tha train can pass an average of 3 stations every 10 mins, at this rate how many station
will it pass in one hour?
a) 30
b) 26
c) 18
d) 15
43. Going against the wind, a domestic plane can travel 5/8 of the distance it can travel in 1
hour if it is going withthe wind. If the plane can travel 300 kilometers in aclm air, what is
the velocity of the wind?
a) 69.23 kph
b) 62.93 kph
c) 63.92 kph
d) 96.23 kph
44. John is four times as olf5 old as Harry, in 6 years, john will be as old as Harry. What is
the of f of harry now?
a) 2
b) 3
c) 4
d) 5
45. What is the value of the following determinant
a) 1
b) 3
c) 4
d) 0
46. Find the number of ways two balls, four dolls, six toy guns can be given to 12 children, if
each child gets a toy.
a) 12606
b) 10620
c) 13860
d) 11640
47. If three coins are tossed, how many possible ways are there for at least one coin showing
tails?
a) 5
b) 6
c) 8
d) 7
3
48. What is the indicated root √−125?
a) -25
b) 5

193
c) -5
d) 25
49. Given that w vaires as direclty as the product of x and y and inversely as the square of z
and that w=4 when x=2, y=6, z=3. Whan it the value of w shen x=1, y=4 and z=2?
a) 3
b) 4
c) 5
d) 2
50. If 16 is 4 more than 3x, then x² + 5 is equal to:
a) 18
b) 21
c)
d) 10
51. A professional organization is composed of x ECEs and 2x Ces. If 6 ECEs are replaced
by 6 Ces, 1/6 of the members will be ECEs. What is the value of x?
a) 24
b) 36
c) 12
d) 18
52. An hour long test has 60 problems. If a student completes 30 problems in 20 minutes,
how many seconds does he have on average for completing each of the remaining
problems?
a) 60
b) 90
c) 120
d) 80
53. The sum of two numbers is 10 and the sum of the squares of the numbers is 52. Find the
product of the two numbers.
a) 26
b) 24
c) 32
d) 16
54. After 8:00 pm, a ride in a cab costs P25.00 plus P3.00 for every fifth of a kilometer
traveled. If a passenger travel x kilometers, what is the cost of the trip in pesos, as a
function of x?
a) 28x
b) 25 + 3x
c) 25 + 15x
d) 25 + 0.6x
55. On a scaled map, a distance of 10 cm represent 5 km. I a street is 750 meters long, what
is the lenght on the map, in centimeters?
a) 15
b) 1.5

194
c) 150
d) 0,15
56. Between 1950 and 1960, the population of singapore increased by 3,5 million. If the
amount of increase from 1960 to 1970 was 1.75 million more than the increase from 1950
to 1960, compute the total amount of increase in the population of singapore from 1950
to 1970.
a) 5.75 million
b) 4.25 million
c) 5,25 million
d) 8.75 million
57. In a typical month ½ of the UFO sightings in the california state are attributable to
airplanes in the 1/3 of the remaining sightings are attributable to weather balloons. If
there are 108 sightings in one typical month, how many would be attributable to weather
ballons?
a) 36
b) 54
c) 24
d) 18
58. If x is an integer, which of the following must be an odd integer?
a) 4(x - 1)
b) 4x – 1
c) 3x² - 2
d) x² - 3
59. Ernies average i 6 subjects is 83. If his lowest grade is disregarded, the average of his
ramainign subject if 84. What is his lowest grade?
a) 76
b) 77
c) 79
d) 78
60. A product has a current selling price of P325. If its selling price is expected to decline at
the rate of 10% per annum because of obscelence, what will be its selling price four years
hance?
a) P302.75
b) P202.75
c) P213.23
d) P156.00
61. Instead of multiplying the number by 17, karla divided it by 17. If the answer he
obtained was 1, what should have been the correct answer?
a) 285
b) 276
c) 289
d) 295

195
62. Kim was sent to the store to buy 11 boxes of sardines. Kim could only carry 2 boxes at a
time. How many trip would kim have to make?
a) 4
b) 6
c) 7
d) 5
63. Simplify cos(30 ͦ - A) -–cos (30 ͦ + A) as a function of angle A only.
a) sin A
b) tan A
c) cos A
d) sec A
sin 𝐴 7 sin 𝐵 5
64. in triangle ABC,sin 𝐵 = and sin 𝐶 = . if angle A, B, C are opposite side a, b, c
10 2
respectively, and the triangle had a perimeter of 16, what is the value of a?
a) 12.33
b) 8
c) 4.67
d) 5.33
65. At one side of the road is a pole 25ft high fixed on top of a wall a 5ft high. On the other
side of the roadn at a point on the ground directly opposite, the flagstaff and the wall
subtend equal angles. Find the width of the road.
a) 30 ft
b) 20 ft
c) 45 ft
d) 50 ft
66. If sec 2A = 1 / sin 13A, determine the value of A.
a) 3 ͦ
b) 6 ͦ
c) 7 ͦ
d) 5 ͦ
67. The sides of the triangle are 8, 15 and 17 units. If each side is doubled, by how many
square units of the area of the triangle be increased?
a) 120
b) 60
c) 180
d) 240
68. Find the area of the spherical triangle ABC having the following parts:
Angle A = 140 ͦ angle B = 75 ͦ angle C = 86 ͦ radius of sphere = 4m
a) 32.78m²
b) 41.41m²
c) 33.79m²
d) 34.56m²
69. What is the area of a rhombus whose diagonals are 12 adnd 24?
196
a) 144
b) 164
c) 108
d) 132
70. A wire with length of 54 cm is cut into two unequal lengths. Each pair is bent to form a
square. If the sum of the area of the two squares is 97 sq.cm, what is the area of the
smaller square?
a) 16
b) 81
c) 64
d) 49
71. The perimeter of a circular sector, whose central angle is 60 ͦ is 14ft. Find tha radius of
the circle.
a) 3.68 ft
b) 4.59 ft
c) 6.32 ft
d) 8.74 ft
72. An equilateral triangle has an altitude of 5√3 cm long. Find the area of the triangle.
a) 25√3
b) 3√5
c) 15√3
d) 10√3
73. A circle with radius r has a circumference equal to the perimeter of a square whose side
is S. Which of the following is correct?
a) r = S/2∏
b) r = S
c) r = 2∏/S
d) r = 2S/∏
74. how long will each side of a regular hexagon with perimeter 108 cm.
a) 18
b) 24
c) 32
d) 12
75. How many sides have a polygon in the sum of the measures of the angles is 900 ͦ?
a) 9
b) 6
c) 8
d) 7
76. A rectangular room in to contain 125 square meters of flooring. If its length is to be 5
times is width, what should it dimansions be?
a) 5m x 75m
b) 25m x 125m

197
c) 3m x 15m
d) 5m x 25m
77. If the total number of diagonals in and N-gon is 77, then what is the value of N?
a) 14
b) 13
c) 12
d) 15
78. A circle has a circumference that is numerically equal to its area. If a certain square has
the same area as the circle, what should be the length of the side?
a) 3√∏
b) ∏√3
c) ∏√2
d) 3√∏
79. A rectangular solid has dimensions3, 4 and 5. What is its diagonal?
a) 7.071
b) 7.253
c) 6.325
d) 9.125
80. Find tha volume of the pyramid formed in the first octant by the plane 6x + 10y + 5z –
30 = 0.
a) 15
b) 13
c) 12
d) 14
81. The upper and lower bases of the frutum of a rectangular pyramid are 3m by 4m and 6m
by 8m, respectively. If the volume of the solid is 140m³, how far apart are the bases?
a) 6.5m
b) 4.5m
c) 5m.
d) 3.5m
82. By how many percent will the volume of the cube increase if its edge is increased by
20%.
a) 44
b) 72.8
c) 1.728
d) 7.28
83. Close conical vessel with base radius of 1 m and altitude 2.5m has its axis vertical. In
upright position(vertex uppermost), the depth of water in it is 50 cm. If the vessels was
inverted(vertex lower most), how deep is the water in it?
a) 196.8 cm
b) 186.5 cm
c) 204.6 cm

198
d) 174.8 cm
84. When an irregular-shaped object is placed in a cylindrical vesselof radius 8 cm,
containing water, the water surface rises 6 cm. What is the volume of the object if it is
completely submerged in water?
a) 384∏ cc
b) 525∏ cc
c) 632∏ cc
d) 245∏ cc
85. A right prism has a base in the shape of regullar octagon inscribed in a 10cm by 10cm. If
it altitude of 15cm, fins its volume in cc.
a) 1242.6
b) 1163.4
c) 1359.7
d) 1421.6
86. If two points in the number line have coordinates 1 and 7, find the coordinateof a point
on the line which is twice as far from 1 as from 7.
a) 3
b) 4
c) 6
d) 5
87. Find the equation of the line with slope of 2/3 and which passes through point of
intersection in the lines 4x – 2y + 1 = 0 and x – 2y + 4 = 0
a) 4x – 6y + 9 = 0
b) 6x – 4y + 9 = 4
c) 6x – 4y + 11 =0
d) 4x - 6y + 11 = o)
88. Find x if the point (x, 2) is equidistant from (3, 2) and (7, 2)
a) 5
b) 4
c) 8
d) 6
89. A rectangle with sides parallel to the axes has vertices at (-3, 2), (2, 5), and (-3, 5). What
is the coordinate of the fourth vortex?
a) (-5, 2)
b) (2, 2)
c) (5, -2)
d) (-3, -2)
90. The triangle definde by the points A(6, 1), B(2, 4) and C(-2, 1) is what?
a) Right
b) Obtuse
c) Isosceles
d) scalene
91. given the curve 36x² + 9y² -36 = 0. What is the length of the latus rectum?

199
a) 0.6
b) 0.75
c) 1.5
d) 1
92. Which of the following points (1,0), (4,4), and (9,7) belong to the graph of the equation y
= x² - x²?
a) (-1,0)
b) (9,7)
c) (4,4) and (1,0)
d) (1,0)
93. A circle is described by the equation x² + y² -16x = 0. What is the lenght of the chord
the is 4 units from the center of the circle?
a) 12,536
b) 13,856
c) 8.523
d) 9.632
94. What is the eqaution of the line that passes through (-3, 5) and is parallel to the line 4x -
2y _2 =0?
a) 4y – 2x + 22 = 0
b) 2x + y + 20 = 0
c) 4x – 2y + 11 = 0
d) 2x – y + 11 = 0
95. A circle with its center in the first quadrant is tangent to both x and y- axes. If the radius
is 4, what is the equation of the circle?
a) (x + 4) ² + (y + 4 ) ² = 16
b) (x - 4) ² + (y – 4) ² = 16
c) (x-4) ² + (y + 4) ² = 16
d) (x + 2) ² + (y - 2) ² = 16
96. What is the deistance between the line x + 2y + 8 = 0 and the point (5, 2)?
a) 4.025
b) 4.502
c) 4.205
d) 4.052
97. Find the area enclosed by the followig curve: x²+y²-10y + 25 = 0
a) 85.47 square units
b) 95.61 square units
c) 78.55 square units
d) 68.53 square units
98. What is the diameter of a circle wiht the following equation: x²+y²- 6x + 4y -12 = 0
a) 10
b) 5
c) 16
d) 25

200
99. What conic section is described by the equation x² + y² - 4x + 2y -20 = 0
a) Cirlce
b) Parabola
c) Hyperbola
d) Elipse
100. The directrix of the parabola is y -5 = 0 and its focus is (4,-3). Find the length of
its lactus rectum.
a) 16
b) 12
c) 2
d) 4

101. Find the area bounded by the parabola x² = 16(y-1) and its lactus rectum
a) 56.33
b) 46.67
c) 36.67
d) 42.67
102. 4x² - y² = 16 is the equation of a:
a) Circle
b) Parabola
c) Hyperbola
d) ellipse
103. find the area bounded by the parabola 4y = x²- 2x + 1 and its lactus rectum.
a) 8/3
b) 4/3
c) 2/3
d) 10/3
104. Point (3, 4) is the center of a circle that is tangent to the x – axis. What is the
point of tangency?
a) (4, 0)
b) (0, 4)
c) (3, 0)
d) (0, 3)
105. An arc 18 m high has a form of a parabola with a vertical axis. The length of the
horizontal beam placed across the arch is 8m from the top is 64m. find the width at the
bottom.
a) 81
b) 96
c) 64
d) 74
106. Find k such that the line y = 4x + 3 is tangent to the curve y = x² + k.
a) 7

201
b) 6
c) 5
d) 4
107. A triangle have vertices at (0, 0), (6, 30) and (9, 70). What is the perimeter of the
triangle?
a) 29.45 units
b) 26.74 units
c) 20.85 units
d) 23.74 units
108. The square root of the variance is referred to as:
a) Dispersion
b) Central tendency
c) Median
d) Standard deviation
109. To find the angle of a triangle, given only the length of the sides, one would use:
a) Law of sines
b) Law on tangents
c) Orthogonal functions
d) Law of cosines
110. A curve generated by a point which moves in uniform circular motion about an
axis while traveling with a constant speed parallel to the axis.
a) Epicyclod
b) Cycloid
c) Spiral of Archimedes
d) helix

202
PART XIII: THE
DIFFERENTIAL

DEFINITION
-is the differential equations, derivatives, and applications of derivatives. For any given value,
the derivative of the function is defined as the rate of change of functions with respect to the
given values. Differentiation is a process where we find the derivative of a function. Let us
discuss the important terms involved in the differential calculus basics.
FUNCTIONS
- is defined as a relation from a set of inputs to the set of outputs in which each input is exactly
associated with one output. The function is represented by "𝑓(𝑥)".
DEPENDENT VARIABLE
- is a variable whose value always depends and determined by using the other variable called an
independent variable. The dependent variable is also called the outcome variable. The result is
being evaluated from the mathematical expression using an independent variable is called a
dependent variable.
INDEPENDENT VARIABLE
Independent variables are the inputs to the functions that define the quantity which is being
manipulated in an experiment. Let us consider an example 𝑦 = 3. Here, 𝑥 is known as
independent variables and y is known as the dependent variable as the value of y is completely
dependent on the value of 𝑥.
DOMAIN AND RANGE

The domain of a function is simply defined as the input values of a function and range is defined
as the output value of a function. Take an example, if 𝑓(𝑥 ) = 3 be a function, the domain values
or the input values are {1, 2, 3} then the range of a function is given as
𝑓 (1) = 3(1) = 3

203
𝑓 (2) = 3(2) = 6
𝑓 (3) = 3(3) = 9
Therefore, the range of a function will be {3, 6, 9}.
LIMITS
The limit is an important thing in calculus. Limits are used to define the continuity, integrals, and
derivatives in the calculus.
INTERVAL
An interval is defined as the range of numbers that are present between the two given numbers.
intervals can be classified into two types namely:

• Open Interval – The open interval is defined as the set of all real numbers 𝑥 such that
𝑎 < 𝑥 < 𝑏. It is represented as (𝑎, 𝑏).
• Closed Interval – The closed interval is defined as the set of all real numbers 𝑥 such that
𝑎 ≤ 𝑥 and 𝑥 ≤ 𝑏, or more concisely, 𝑎 ≤ 𝑥 ≤ 𝑏, and it is represented by [𝑎, 𝑏].

DERIVATIVES
The fundamental tool of differential calculus is derivative. The derivative is used to show the rate
of change. It helps to show the amount by which the function is changing for a given point. The
derivative is simply called a slope. It measures the steepness of the graph of a function. It defines
the ratio of the change in the value of a function to the change in the independent variable. The
𝑑𝑦
derivative is expressed by 𝑑𝑥 .

APPLICATION OF DIFFERENTIAL
In mathematics, differential calculus is used,

• To find the rate of change of a quantity with respect to other


• In case of finding a function is increasing or decreasing functions in a graph
• To find the maximum and minimum value of a curve
• To find the approximate value of small change in a quantity
Real-life applications of differential calculus are:

• Calculation of profit and loss with respect to business using graphs.


• Calculation of the rate of change of the temperature
• Calculation of speed or distance covered such as miles per hour, kilometres per hour, etc.
• To derive many Physics equations.

204
ERROR PROPAGATION
-We can also use differentials in Physics to estimate errors, say in physical measuring devices.
In these problems, we’ll typically take a derivative, and use the "𝑑𝑥" or "𝑑𝑦" part of the
derivative as the error. Then, to get percent error, we’ll divide the error by the total amount and
multiply by 100.
-The other thing to remember is that when we are solving for an error, it can go either way, so we
typically express our answers with a " ± ".
Example 1.
The volume of a cube is 125 in3. If the volume measurement is known to be correct to
within 2.5 in3, estimate the error in the measurement of a side of the cube.
Solution:
• We can attack this like a related rates problem: write down what we know, what we need,
and how we relate the variables.
• We have 𝑉 = 125, and 𝑑𝑉 = 2.5 (remember that the error is the"𝑑𝑥" part of the
equation). We want the error in the side of the cube, so we want 𝑑𝑠.
• Now let’s relate the variables and differentiate with respect to 𝑠:

𝑑𝑉
𝑉 = 𝑠 3; = 3𝑠 2 ; 𝑑𝑉 = 3𝑠 2 𝑑𝑠
𝑑𝑠

• Substitute and solve for 𝑑𝑠. Note that since we know the volume (𝑉) is 125, we know a
3
side (𝑠) is √125 = 5:
2.5
𝑑𝑉 = 3𝑠 2 𝑑𝑠; 2.5 = 3(5)2 𝑑𝑠; 𝑑𝑠 = = ±.0333
3(5)2

The error in the measurement of a side of the cube is ±. 𝟎𝟑𝟑𝟑 in.

Example 2.
The radius of a sphere is measured to be 5 mm. If this measurement is correct to within .05 mm,
a) Estimate the propagated error in the surface area of the sphere.
b) Estimate the propagated error in the volume of the sphere.
c) Estimate the percent error of the volume of the sphere.
Solution:
• We have 𝑟 = 5, and 𝑑𝑟 = .05 (remember that the error is the "𝑑𝑥" part of the
equation). We want the both the error in the surface area (𝑑𝐴) and the error in the volume
(𝑑𝑉). We have to remember the equations from Geometry.

205
a) Let’s relate surface area and radius, and differentiate with respect to 𝑟:
𝑑𝐴
𝐴 = 4𝜋𝑟 2 ; = 8𝜋𝑟; 𝑑𝐴 = (8𝜋𝑟) 𝑑𝑟
𝑑𝑟

We want 𝑑𝐴 (error in the surface area):


𝑑𝐴 = (8𝜋𝑟) 𝑑𝑟; 𝑑𝐴 = (8𝜋 ∙ 5) .05 ≈ ±𝟔. 𝟐𝟖𝟑
The error in the measurement of the surface area of the sphere is ±6.283 mm2.
b) Let’s relate volume and radius, and differentiate with respect to 𝑟.
4 3 𝑑𝑉
𝑉= 𝜋𝑟 ; = 4𝜋𝑟 2 ; 𝑑𝑉 = (4𝜋𝑟 2 )𝑑𝑟
3 𝑑𝑟
We want 𝑑𝑉 (error in the volume):
𝑑𝑉 = (4𝜋𝑟 2 ) 𝑑𝑟; 𝑑𝑉 = (4𝜋 ∙ 52 ). 05 ≈ ±𝟏𝟓. 𝟕𝟎𝟖
The error in the measurement of the volume of the sphere is ±15.708 mm3.
c) To get percent error:
𝐸𝑟𝑟𝑜𝑟 𝑑𝑉
𝑃𝑒𝑟𝑐𝑒𝑛𝑡 𝐸𝑟𝑟𝑜𝑟 = ∙ 100 = ∙ 100
𝑉𝑜𝑙𝑢𝑚𝑒 𝑉
15.708 15.708
= 4 = 4 ≈ .03 ∙ 100 = 3
𝜋𝑟 3 𝜋(5)3
3 3

• The percent error in the measurement of the volume of the sphere is 3%.

Example 3.
The measurement of the base and altitude of a triangle are 20 cm and 30 cm, respectively. The
possible error in each measurement is .3 cm.
a) Estimate the possible propagated error in computing the area of the triangle.
b) Approximate the percent error in computing this area.

Solution:
• This one’s a little trickier since we have two variables with error. We have
𝑏 = 20, ℎ = 30, 𝑑𝑏 = .3, 𝑑ℎ = .3 (remember that the error is the "𝑑𝑥" part of the equation).
• We want the error in the area of the triangle (𝑑𝐴), so we have to relate the two
1
perpendicular sides of the triangle to the area: 𝐴 = 𝑏ℎ.
2

We’ll be using the differential product rule, 𝑑[𝑢𝑣 ] = 𝑢𝑑𝑣 + 𝑣𝑑𝑢. We can also notice that

206
1 𝑑𝐴 1 𝑑ℎ 𝑑𝑏 1 𝑑ℎ 𝑑𝑏
𝐴= 𝑏ℎ; = (𝑏 ∙ +ℎ ∙ ) ; 𝑑𝐴 = (𝑏 ∙ +ℎ ∙ ) 𝑑𝑥
2 𝑑𝑥 2 𝑑𝑥 𝑑𝑥 2 𝑑𝑥 𝑑𝑥

1
• So we have 𝑑𝐴 = (𝑏 ∙ 𝑑ℎ + ℎ ∙ 𝑑𝑏).
2

Now substitute and solve for 𝑑𝐴:


1 1
𝑑𝐴 = (𝑏 ∙ 𝑑ℎ + ℎ ∙ 𝑑𝑏) = (20 × .3 + 30 × .3) = 7.5
2 2

a) The error in the measurement of the area of the triangle is ±7.5 cm2.
b) The percent error in the area is
𝑒𝑟𝑟𝑜𝑟 𝑖𝑛 𝑎𝑟𝑒𝑎 𝑑𝐴 7.5
∙ 100 = ∙ 100 = × 100 = 𝟐. 𝟓%.
𝑡𝑜𝑡𝑎𝑙 𝑎𝑟𝑒𝑎 𝐴 1
∙ 20 ∙ 30
2
Example 4
How much variation 𝑑𝑟 in the radius of a coin can be tolerated if the volume of the coin is to be
1
within 1000 of their ideal volume?

Solution:
• It’s difficult to know how to start this problem, but let’s start out by relating radius with
volume by using the volume of a cylinder equation.
1
We want 𝑑𝑟, and have 𝑑𝑉 (1000), so this will fit into using this equation and differentiating:
𝑑𝑉
𝑉 = 𝜋𝑟 2 ℎ; = 2𝜋𝑟ℎ; 𝑑𝑉 = 2𝜋𝑟ℎ 𝑑𝑟
𝑑𝑟
1
Since we know the volume of the coin is to be within of volume of its total volume, we can
1000
interpret 𝑑𝑉 to be this error, so |𝑑𝑉 | ≤ .001𝑉.
Now let’s solve for 𝑑𝑟: |𝑑𝑉 | = |2𝜋𝑟ℎ 𝑑𝑟| ≤ .001𝑉, 𝑜𝑟 |2𝜋𝑟ℎ 𝑑𝑟| ≤ .001 (𝜋𝑟 2ℎ).
.001(𝜋𝑟 2ℎ)
Therefore, |𝑑𝑟| ≤ , or |𝑑𝑟| ≤ .0005𝑟.
2𝜋𝑟ℎ

The variation of the radius should not exceed .05% of the ideal radius.

207
Exercises:
1. Suppose the radius of a circle is measured to be 𝑟 = 6 cm but the exact value of the radius (if it
could be measured with infinite precision) is 6.15 cm. Then ∆𝑟 = 0.15 cm. Computing the area of
the circle using the formula 𝐴 = 𝜋𝑟 2 based on the measurement of 𝑟 = 6 cm gives 𝐴 = 𝜋62 ≈
113.097 cm2, whereas the exact area should be 𝐴 = 𝜋6.152 ≈ 118.823 cm2. The difference,
2. The edge of a cube is measured to be 12 inches with a possible error of 0.03 inches. What is
the maximum propagated error of the volume and surface area?
3. The radius of a ball bearing is measured to be .7 inch. If the measurement is correct to
within .01 inch, find the above errors in the volume of the ball bearing.

4. The radius of a sphere was measured and found to be 16 cm, with a possible error of at most
0.05 cm. Find the maximum error and relative error in using value of radius?

Solution #1:
∆𝐴 = 𝐴(6.15) − 𝐴(6) ≈ 5.726 cm2,
is the error in the computed area due to the fact that an approximation of the exact radius was
used.
• Differentials can be used to approximate propagated errors. If 𝑦 = 𝑓 (𝑥 ), then
∆𝑦 ≈ 𝑑𝑦 = 𝑓 ′(𝑥 )𝑑𝑥,
Where 𝑑𝑥 = ∆x.
In the previous example
∆𝐴 ≈ 𝑑𝐴 = 2𝜋𝑟𝑑𝑟 = 2𝜋(6 cm) (0.15 cm) ≈ 5.655 cm2.
Therefore, the propagated error is approximately 5.655 cm2 . Meanwhile the relative error, which
gives a more meaningful measure of the relative size (how large or how small) the error is, is
∆𝐴 𝑑𝐴 2𝜋𝑟𝑑𝑟 2𝑑𝑟 2(0.15)
≈ = = = = 0.05, or approximately 5%.
𝐴 𝐴 𝜋𝑟 2 𝑟 6

Solution #2:
𝑉 = 𝑥3
𝑆𝐴 = 6𝑥 2
𝑑𝑉
𝑑𝑥 ( = 3𝑥 2 ) 𝑑𝑥
𝑑𝑥
𝑑𝑉 = 3𝑥 2 𝑑𝑥

208
𝑑𝑉 = 3(12)2 (±.03)
𝒅𝑽 = ±𝟏𝟐. 𝟗𝟔 in3

𝑑(𝑆𝐴)
𝑑𝑥 ( = 12𝑥) 𝑑𝑥
𝑑𝑥
𝑑 (𝑆𝐴) = 12𝑥𝑑𝑥
𝑑 (𝑆𝐴) = 12(12)(±.03)
𝒅(𝑺𝑨) = ±𝟒. 𝟑𝟐 in2

Solution #3:

4
𝑉 = 𝜋𝑟 3
3
𝑑𝑉 4
= 𝜋 ∙ 3𝑟 2
𝑑𝑟 3
𝑑𝑉
= 4𝜋𝑟 2
𝑑𝑟
𝑑𝑉 = 4𝜋𝑟 2 𝑑𝑟
∆𝑉 = 4𝜋𝑟 2 ∆𝑟
= 4𝜋 (. 7)2 (±.01)
= .062 in3 – Propagated error

∆𝑉 4𝜋𝑟 2 ∆𝑟
=
𝑉 4 3
3 𝜋𝑟
3∆𝑟
= 𝑟
3(±.01)
= .7

= ±.043 in3 – Relative error


Percent error – 4.3%

209
Solution #4:
𝑑
𝑦 = 𝑓 ′ (𝑥 )
𝑑𝑥
𝑑𝑦 = 𝑓 ′(𝑥 ) ∙ 𝑑𝑥
Linear Relation
𝑓 ′ (𝑥 ) = 𝑘
𝑑𝑦 ∝ 𝑑𝑥
𝑟 = 16 cm
𝑑𝑟 = 0.05 cm

For calculation of volume


4 3
𝑉= 𝜋𝑟
3
𝑑𝑉 4
= 𝜋 ∙ 3𝑟 2
𝑑𝑟 3
𝑑𝑉 = 4𝜋𝑟 2 ∙ 𝑑𝑟
𝑑𝑉 = 4𝜋(16)2 ∙ (0.05)
= 160.77 cm3
= 161 cm3 – Max error

Relative error
∆𝑉 160.77
= = 9.375 × 10−3
𝑉 17148

= 0.0093

Percent Error:
∆𝑉
× 100 = 0.0093 × 100
𝑉
= 𝟎. 𝟗𝟑%

210
APPROXIMATE FORMULA
-A method for approximating the value of a function near known value. The method uses the
tangent line at the known value of the function to approximate the function's graph. In this
method ∆𝑥and ∆𝑦 represent the changes in 𝑥 and 𝑦 for the function, and 𝑑𝑥 and 𝑑𝑦 represent the
changes in 𝑥 and 𝑦 for the tangent line.
-A differentiable function 𝑦 = 𝑓 (𝑥 ) can be approximated at 𝑎 by the linear function
𝐿(𝑥 ) = 𝑓 (𝑎) + 𝑓 ′(𝑎)(𝑥 − 𝑎).
Example 1.
Approximate √10 by differentials.

Solution:
√10 is near √9, so we will use 𝑓 (𝑥 ) = √𝑥 with 𝑥 = 9 and ∆𝑥 = 1.
1
Note that 𝑓 ′(𝑥 ) = .
2√𝑥

√10 = 𝑓 (𝑥 + ∆𝑥 )
≈ 𝑓 (𝑥 ) + 𝑓 ′(𝑥)∆𝑥
1
= √𝑥 + ∆𝑥
2√𝑥
1
= √9 + (1)
2√9
1
=3
6
1
Thus we see that √10 ≈ 3 = 𝟑. 𝟏𝟔𝟔𝟔. This is very close to the correct value of √10 ≈
6
𝟑. 𝟏𝟔𝟐𝟑.

Example 2.
Approximate ln(𝑥 + 2) ‘by differentials’, in terms of ln 𝑥 and 𝑥: This non-numerical question is
1
somewhat more sensible. Take 𝑓 (𝑥 ) = ln 𝑥, so that 𝑓 ′(𝑥 ) = 𝑥. Then

Solution:
∆𝑥 = (𝑥 + 2) − 𝑥 = 2
and by the formulas above
𝟐
ln(𝑥 + 2) = 𝑓 (𝑥 + 2) ≈ 𝑓(𝑥 ) + 𝑓 ′ (𝑥 ) ∙ 2 = ln 𝒙 + 𝒙.

211
Example 3.
Find the linear approximation of 𝑓 (𝑥 ) = √𝑥 at 𝑥 = 9 and use the approximation to
estimate √9.1 .
Solution:
Since we are looking for the linear approximation at 𝑥 = 9, using (Figure) we know the linear
approximation is given by
𝐿(𝑥 ) = 𝑓 (9) + 𝑓 ′(9)(𝑥 − 9)
We need to find 𝑓 (9) and 𝑓 ′ (9).
𝑓 (𝑥 ) = √𝑥 → 𝑓 (9) = √9 = 3
1 1 1
𝑓 ′ (𝑥 ) = → 𝑓 ′(9) = =
2√𝑥 2√9 6
Therefore, the linear approximation is given by (Figure).
1
𝐿 (𝑥 ) = 3 + (𝑥 − 9)
6
Using the linear approximation, we can estimate √9.1 by writing
1
√9.1 = 𝑓 (9.1) ≈ 𝐿(9.1) = 3 + 6
(9.1 − 9) ≈ 3.0167.

Example 4.
Another numerical example: Approximate sin 31° ‘by differentials’. Again, the point is not to hit
3, 1, sin on your calculator (after switching to degrees), but rather to imagine that you have no
calculator. And we are supposed to remember from pre-calculator days the ‘special angles’ and

212
1 √3
the values of trig functions at them: sin 30° = 2 and cos 30° = 2 . So we'd use the function
𝑓 (𝑥 ) = sin 𝑥, and we'd imagine that we can evaluate 𝑓 and 𝑓′ easily by hand at 30°. Then
Solution:
2𝜋 radians 2𝜋
∆𝑥 = 31° − 30° = 1° = 1° ∙ = radians
360° 360

We have to rewrite things in radians since we really only can compute derivatives of trig
functions in radians. Yes, this is a complication in our supposed ‘computation by hand’.
Anyway, we have
2𝜋
Sin 31° = 𝑓(31°) = 𝑓 (30°) + 𝑓′(30°)∆𝑥 = sin 30° + cos 30° ∙ 360
1 √3 2𝜋
= + ∙
2 2 360

= 𝟎. 𝟓𝟏𝟓𝟏

Exercises:
1. For example let's approximate √17 by differentials. For this problem to make sense at all
imagine that you have no calculator. We take
1
𝑓 (𝑥 ) = √𝑥 = 𝑥 2 . The idea here is that we can easily evaluate ‘by hand’ both 𝑓 and 𝑓′ at the
1
1
point 𝑥 = 16 which is ‘near’ 17. (Here 𝑓 ′(𝑥 ) = 𝑥 −2 ). Thus, here
2

1
2. Approximate ln(𝑒 + 2)in terms of differentials: Use 𝑓(𝑥 ) = ln 𝑥 again, so 𝑓 ′(𝑥 ) = 𝑥. We
1
probably have to imagine that we can ‘easily evaluate’ both ln 𝑥 and 𝑥 at 𝑥 = 𝑒. (Do we know a
numerical approximation to 𝑒?). Now

3. Find a good approximation for √9.2 without using a calculator.

4. Similarly, if we wanted to approximate √18 ‘by differentials’, we'd again take (𝑥 ) = √𝑥 =


1
𝑥 2 . Still we imagine that we are doing this ‘by hand’, and then of course we can ‘easily
evaluate’ the function 𝑓 and its derivative 𝑓′ at the point 𝑥 = 16 which is ‘near’ 18. Thus, here

213
Solution #1:
∆𝑥 = 17 − 16 = 1
and
1 1 1
√17 = 𝑓 (17) ≈ 𝑓 (16)∆𝑥 = √16 + ∙ ∙ 1 = 4 + = 4.125
2 2√16 8

Solution #2:
∆𝑥 = (𝑒 + 2) − 𝑒 = 2
so we have
2 𝟐
ln(𝑒 + 2) = 𝑓 (𝑒 + 2) ≈ 𝑓 (𝑒) + 𝑓 ′ (𝑒) ∙ 2 = ln 𝑒 + 𝑒
=𝟏+ 𝒆

since ln 𝑒 = 1.

Solution #3:
𝐿𝑒𝑡 𝑓 (𝑥 ) = √𝑥 , 𝑥 = 9 , ∆𝑥 = 𝑑𝑥 = 0.2
𝑓 (𝑥 + ∆𝑥 ) ≈ 𝑓 (𝑥 ) + 𝑓 ′(𝑥 )∆𝑥
𝑓 (9.2) ≈ 𝑓 (9) + 𝑓 ′ (9)(0.2)
1
≈ √9 + (0.2)
2√9
1
= 3 + 6 (0.2)
0.1
=3+ 3

= 3 + 0.0333 = 𝟑. 𝟎𝟑𝟑𝟑
Solution #4:
∆𝑥 = 18 − 16 = 2 and
1 1 1
√18 = 𝑓 (18) ≈ 𝑓 (16) + 𝑓 ′(16)∆𝑥 = √16 + ∙ ∙2=4+ .
2 √16 4
= 4.25

214
Problem set 20: recent board exams 2
1. Which of the following mathematically represents a two-digit number with x as the unit’s digit
and y as its ten’s digit.

A. 10y + 10x C. 10y + x


B. 10y – x D. 10x + y

2. What is the equation form of the statement; the amount by which 100 exceeds four times a
given number?
A. 4x(100) C. 100 - 4x
B. 100 + 4x D. 4x – 100

3. Solve for x if 8x + 2y+2 and 16 3x – y + 4 y.


A. 4 C. 3
B. 2 D. 1

4. From the equation 7x2 + (2k - 1)x - 3k + 2 = 0, determine the value of k so that the sum and
product of the roots are equal.
A. 4 C. 2
B. 3 D. 1

5. The arithmetic mean of six numbers is 17. If two numbers are added to the progression, the
new arithmetic mean is 21. What are the two numbers if their difference is 4?
A. 28 and 32 C. 26 and 30
B. 30 and 34 D. 34 and 38

6 .What is the sum of all the odd integers between 10 and 500?
A. 65,955 C. 87,950
B. 62,475 D. 124,950

7. How many terms of the progression 3, 5, 7 … should there be so that their sum will be 2600?
A. 54 C. 50
B. 52 D. 55

8 .If the first term of the geometric progression is 27 and the fourth term is -1, the third term is?
A. 3 C. -2
B. 2 D. -3

9. The fourth term of a geometric progression is 6 and the 10th term is 384.
What is the common ratio of the G.P.?
A. 1.5 C. 2.5
B. 3 D. 2

10. What is the first term?

215
A. 0.75 C. 3
B. 1.5 D. 0.5

11.What is the seventh term?


A. 24 C. 48
B. 32 D. 96

12. Candle A and candle B of equal length are to be lighted at the same time and burning until
candle A is twice as long as candle B. candle A is designed to fully burn in 8 hours and candle B
for 4 hours. How long will they be lighted?
A. 2 hrs and 40 mins C. 3 hrs and 40 mins
B. 3 hrs and 30 mins D. 2 hrs and 30 mins

13. At approximately what time between 6 and 7 o’clock will the minute and hour hands
coincide?
A. 27 min and 41 sec after 6 o’clock
B. 32 min and 72 sec after 6 o’clock
C. 25 min and 38 sec after 6 o’clock
D. 32 min and 43 sec after 6 o’clock

14. In how many minutes after 2 p.m. will the hands of the clock extend in opposite directions
for the first time?
A. 40.522 C. 45.575
B. 43.636 D. 41.725

15. How many ounces of pure nickel must be added to 150 ounces of alloy 70% pure to make an
alloy 85% pure?
A. 150 C. 125
B. 225 D. 175

16. If there are nine distinct items taken three at a time, how many permutations will there be?
A. 252 C. 504
B. 720 D. 336

17 .A bag contains 3 white and 5 red balls. If two balls are drawn in succession without returning
the first ball drawn, what is the probability that the balls drawn are both red?
A. 0.299 C. 0.357
B. 0.237 D. 0.107

18. A janitor with a bunch of nine keys is to open a door but only one key can open. What is the
probability that he will succeed in three trials?
A. 0.375 C. 0.333
B. 0.425 D. 0.255

19. Evaluate the expression: (1+ i2)10, where i is an imaginary number.

216
A. 0 C. i
B. -1 D. 1

Given the following matrix:


2 5 1 1 0 0
A = {3 1 0} and B = {0 1 0}
6 7 4 0 0 1
20 .Determine the minor corresponding to the 5 entry in matrix A.
A. 15 C. 12
B. 8 D. 4

21. Determine the cofactor corresponding to the 3 entry in matrix A.


A. 13 C. 4
B. -13 D. -4

22. What is the product A x B?


1 2 5 5 2 1
A. {0 3 1} C. {1 3 0}
4 6 7 7 6 4

2 1 5 2 5 1
B. {3 0 1} D. {3 1 0}
6 4 7 6 7 4

23. If sin A = 4/5 and sin B = 7/25, what is sin (A+B) if A is in the 3rd quadrant and B is in the 2nd
quadrant?
A. 3/5 C. -3/5
B. 2/5 D. 4/5

24. If the sides of a right triangle are 3, 4, 5 m, the area of the inscribed circle is _______.
A. 2π square m C. 3/4π square m
3π/2 square m D. π square m
25. A flagpole stands on top of a pedestal. From a point O at the same elevation as the base of
the pedestal, the vertical angle of the top of the angle is 60° and the vertical angle of the bottom
of the angle is 52°. Point O is 14.8 meters away from the base of the pedestal.
How high is the pedestal in meters?
A. 18.94 C. 15.24
B. 25.36 D. 14.28

26. How long is the flagpole in meters?


A. 3.68 C. 8.12
B. 5.88 D. 6.69

27. What is the distance from O to the top of the flagpole?


A. 33.2 m C. 15.4 m
B. 29.6 m D. 22.8 m

217
28.One of the diagonals of a rhombus measure 12 inches. If the area of the rhombus is 132
square inches, determine the following:

The length of the other diagonal in inches.


A. 24 C. 22
B. 8 D. 14

29. The measure of the side of the rhombus in inches.


A. 13.85 C. 8.52
B. 10.96 D. 12.53

30. The measure of the acute angle between the sides of the rhombus in degrees.
A. 28.61 C. 57.22
B. 45.26 D. 32.78

31. A triangular lot ABC have side AB = 400 and angle B = 50°. The lot is to be segregated by a
dividing line DE parallel to BC and 150 m long. The area of segment BCDE is 50,977.4 m2.
Calculate the area of lot ABC, in square meter.
A. 62,365 C. 57,254
B. 59,319 D. 76,325

32. Calculate the area of lot ADE, in square meter.


A. 8,342 C. 6,569
B. 14,475 D. 11,546

33. Calculate the value of angle C, in degrees.


A. 57 C. 63
B. 42 D. 68

34. A railroad curve is to be laid in a circular path. What should be the radius if the track is to
change direction by 30 degrees at a distance of 300 m?
A. 352 m C. 287 m
B. 573 m D 452 m

35. A swimming pool is shaped from two intersecting circles 9 m in radius with their centers 9 m
apart.
What is the area common to the two circles, in square meter?
A. 85.2 C. 128.7
B. 63.7 D. 99.5

36. What is the total water surface area, in square meter?


A. 409.4 C. 387.3
B. 524.3 D. 427.5

37. What is the perimeter of the pool, in meters?

218
A. 63.5 C. 82.4
B. 75.4 D. 96.3

38. Spheres of the same size are piled in the form of a pyramid with an equilateral triangle as its
base. Compute the total number of spheres in the pile if each side contains 4 spheres.
A. 20 C. 16
B. 64 D. 28

39. What is the volume in (cm3) of a right hexagonal prism 15 cm high and with one of its sides
equal to 6 cm?
A. 985 C. 818
B. 929 D. 1185

40. Eight balls are tightly packed in a cubical container that measures 8 cm on each side. The
balls are arranged with 4 balls per layer and in contact with the walls of the container and the
adjacent balls. If the container is filled with water, what is the volume of the water (in cubic cm)?
A. 355 C. 268
B. 335 D. 244

41. A closed conical vessel has a base radius of 2 m and is 6 m high. When in upright position,
the depth of water in the vessel is 3 m.
What is the volume of water in cubic meter?
A. 22 C. 28
B. 25 D. 32

42. If the vessel is held in inverted position, how deep is the water in meters?
A. 8.56 C. 4.12
B. 5.74 D. 6.87

43. What is the weight of water in quintals? Unit weight of water is 9,800 N/ m 3.
A. 263.4 C. 219.7
B. 195.4 D. 247.2

44. A total volume of two spheres is 100π cubic m. The ratio of their areas is 4:9. What is the
volume of the smaller sphere in cubic cm?
A. 75.85 C. 71.79
B. 314.16 D. 242.36

45. What is the distance between points (3,7) and (-4,-7)?


A. 18.65 C. 5.25
B. 6.54 D. 15.65

46. What is the area bounded by the curve defined by the equation
x2-8y = 0 and its latus rectum?
A. 5.33 C. 10.67
B. 7.33 D. 3.66

219
47. Given the ellipse 9x2+16y2 – 144 = 0.

Determine the length of arc in the first quadrant.


A. 5.55 C. 7.58
B. 6.32 D. 9.97

48. Determine the equation of its diameter bisecting all chords having equal slope of 3.
A. 4x + 25y C. 6x + 32y = 0
B. 9x + 16y = 0 D. 9x + 32y = 0

49. What is the volume generated if the area on the first and second quadrants is revolved about
the X-axis.
A. 201.1 C. 175.4
B. 150.8 D. 165.7

50. Give the equilateral hyperbola xy= 8:

What is the length of the conjugate axis of the hyperbola?


A. 12 C. 8
B. 16 D. 4

51. How far apart are the vertices of the hyperbola?


A. 4 C. 16
B. 8 D. 12

52. Determine the eccentricity of the hyperbola.


A. 1.1414 C. 1.368
B. 1.732 D. 1.521

Given the ellipse 16x2 + 25y2 = 400.

53. Compute its perimeter.


A. 32.653 C. 24.785
B. 28.448 D. 36.896

54. Determine its second eccentricity.


A. 0.75 C. 0.67
B. 0.82 D. 0.6

55. What is the equation of its diameter bisecting the chords having equal slope of 1/5?
A. 5x – 16y = 0 C. 16x + 5y = 0
B. 5x + 16y = 0 D. 16x – 5y = 0

220
PROBLEM SET 20: LIMITS,
DIFFERENTIATION, RATE OF CHANGE,
SLOPE
1. Evaluate lim (1 + 𝑠𝑖𝑛²𝜃 )½.
𝜃→∞

A. 0 C. 2
B. 1 D. 3
𝑥 2 −16
2. Simplify the expression lim .
𝑥→4 𝑥−4

A. 1 C. 0
B. 8 D. 16
𝑥 2 −1
3. Evaluate the following limit lim .
𝑥→2 𝑥 2 +3𝑥−4

/A. 2/5 C. 0
B. infinity D. 5/2
4. Evaluate the limit(x-4)/(x²-x-12) as x approaches 4.
A.0 C. e
B. undefined D. infinity
5. Evaluate the limit (In x)/x as x approaches positive infinity.
A.1 C. e
B.0 D. infinity..
𝑥+1
6. Evaluate the following limit lim .
𝑥→∞ 𝑥−4

A. 1 C.0
B. Indifinite D.2
1−𝑐𝑜𝑠𝑥
7. Evaluate lim .
𝑥→0 𝑥2

A.0 C.2
B.½ D.-½

221
𝜋𝑥
8. Evaluate the following limit lim (2 − 𝑥 )𝑡𝑎𝑛 2 .
𝑥→1

A.infinty C.0
2
B.𝑒 𝑥 D. 𝑒 𝜋
9 .Find dy/dx if y=52𝑥+1 .
A. 52𝑥+1 𝑙𝑛 5 C. 52𝑥+1 𝑙𝑛10
B. 52𝑥+1 𝑙𝑛 25 D. 52𝑥+1 𝑙𝑛2

10. Find dy/dx if =𝑒 √2 .

A. 𝑒 √𝑥 /2√𝑥 C. 𝑒 𝑥 /√𝑥

B. 𝑒 √𝑥 /√𝑥 D. 𝑒 √𝑥−2√𝑥
11. Find dy/dt if y=x²+3x +1 and t²+2.
A. 4t³+4t² C. 4t³+14t
B. t³+4t D. 4t³+t
12. Evaluate the first derivative of the implicit function:
4x²+2xy+y²=0
4𝑥+𝑦 4𝑥−𝑦
A. 𝑥+𝑦 C. 𝑥+𝑦

4𝑥+𝑦 4𝑥+𝑦
B. − D.−
𝑥+𝑦 𝑥−𝑦

13. Find the derivative of (x+5)/(x²-1) with respect to x.


A.DF(x)=(-x²-10x-1)/(x²-1)² C. DF(x)=(x²-10x-1)/(x²-1)²
B. DF(x)=(-x²+10x-1)/(x²-1)² D. DF(x)=(-x²-10x-1)/(x²+1)²
14. If a simple constant, what is the derivative of y=𝑥 𝑎 .
A. 𝑎 𝑥 𝑎−1 C. 𝑥 𝑎−1
B.( 𝑎 − 1)𝑥 D. 𝑎𝑥
15. Find the derivative of the function 2x²+8x+9 with respect to x.
A. 𝐷𝑓(𝑥 ) = 4𝑥 − 8 C. 𝐷𝑓 (𝑥 ) = 2𝑥 + 8
B 𝐷𝑓 (𝑥 ) = 2𝑥 + 9 D 𝐷𝑓(𝑥 ) = 4𝑥 + 8
16. What is the first derivative dy/dx of the expression.

222
(𝑥𝑦)𝑥 = 𝑒?
A.−𝑦(1 + ln 𝑥𝑦)/𝑥 C. .−𝑦(1 − ln 𝑥𝑦)/𝑥
B. 0 D.y/x
17. Find the derivative of (x+1)³/x.
3(𝑥+1)² (𝑥+1)² 2(𝑥+1)² (𝑥+1)³
A. + C. −
𝑥 𝑥² 𝑥 𝑥²

3(𝑥+1)² (𝑥+1)² 3(𝑥+1)² 2(𝑥+1)³


B. − D. +
𝑥 𝑥² 𝑥 𝑥²

18. Given the equation: 𝑦 = (𝑒 ln 𝑥 )², find y’.

A. 𝑙𝑛 𝑥 C. 2x
B. 2 (ln 𝑥)/𝑥 D. 2𝑒 ln 𝑥
19. Find the derivatives with respect to x of the function √2 − 3𝑥².

A. −2𝑥 2 /√2 − 3𝑥² C. −2𝑥 2 /√2 + 3𝑥²

B. −3𝑥/√2 − 3𝑥² D. −3𝑥 2 /√2 − 3𝑥²


20. Differentiate 𝑎𝑥 2 + 𝑏 𝑡𝑜 𝑡ℎ𝑒 ½ power.
A. −2𝑎𝑥 C. 2𝑎𝑥 + 𝑏
B. 2𝑎𝑥 D. 𝑎𝑥 + 2𝑏

21. Find dy/dx if y=ln √𝑥.

A.√𝑥/ ln 𝑥 C. 1/ 2 𝑥
B./ ln 𝑥 D. 2/𝑥
22. Evaluate the differential of tanθ.
A.ln sec 𝜃𝑑𝜃 C. 𝑠𝑒𝑐𝜃 𝑡𝑎𝑛𝜃 𝑑𝜃
B. ln cos 𝜃𝑑𝜃 D. 𝑠𝑒𝑐²θdθ
23. If y=cos 𝑥, what is dy/dx?
A. sec x C.sin 𝑥
B. -sec x D.− sin 𝑥
24. Find dy/dx: y=sin(ln x²).
A.2 cos(ln 𝑥²) C. 2𝑥 cos(ln 𝑥²)
B. 2 cos(ln 𝑥²) /x D 2 cos(ln 𝑥²)/𝑥²

223
25. The derivative of ln (cos x) is:
A.sec 𝑥 C. −tan 𝑥
B. −sec 𝑥 D. tan 𝑥
26. Find the derivative of 4x with respect to x.
A.−4/[1 − (4𝑥)⌃2] ⌃2 C .4/[1 − (4𝑥)⌃2] ⌃2]0.5
B . −4/[1 − (4𝑥 )⌃2]0.5 D . −4/[1 − (4𝑥)⌃2] ⌃0.5
27. What is the first derivative of y=arcsin 3x
3 3
A.-1=9𝑥² C.−
√1−9𝑥²

3 3
B. 1=9𝑥² D.
√1−9𝑥²

28. If y=x(In x), find d²y/dx².


A.1/𝑥² C. 1/𝑥
B. −1/𝑥 D. −1/𝑥²
29. Find the second derivative of y=x² at x=2
A. 96 C. −0.25
B. 0.375 D. −0.875
30. Given the function f(x)=x³-5x+2, find the value of the first derivative at x=2, f(2)
A.7 C. 2
B.3𝑥 2 − 5 D. 8
31. Given the function f(x)=x to the 3rd power -6x+2,find the value of the first derivative at x=2,
f(2).
A.6 C. 7
B.3x²-5 D. 8
32. Find the partial derivatives with respect to x of the function: xy²-5y+6.
A. y²-5 C.y²
B. xy-5y D.2xy
33. Find the point in the parabola y²=4x at which the rate of change of the ordinate and abscissa
are equal.
A.(1,2) C.(4,4)

224
B. (2,1) D (−1,4)
34. Find the slope of the line tangent to the curve y=x³-2x+1 at x=1
A.1 C. 1/3
B.½ D. 1/4
35. Determine the slope of the curve x²+y²-6x -4y-21=0 at (0,7).
A3/5 C . −3/5
B . −2/5 D .2/5
36. Find the slope of the tangent to a parabola y=x² at a point on the curve where x= ½.
A.0 C.1/4
B. 1 D.−1/2
37. Find the slope of the ellipse x²+4y²-10x+16y+5=0 at the point where y=-2 +80.5 and x=7.
A.−0.1654 C.−0.1768
B. −0.1538 D. −0.1463
38. Find the slope of the tangent to the curve y=𝑥 4 − 2𝑥 2 + 8 through point (2,16).
A. 20 C. 24
B. 1/24 D. 1/20
39. Find the slope of the tangent to the curve y²=3x²+4 through point
(-2,4)
A.−3/2 C. 2/3
B.3/2 D.−2/3
40. Find the slope of the line whose parametic equations are x=4t+6 and y=t-1
A.−4 C. 4
B.1/4 D.−1/4
41. What is the slope of the curve x²+y²-6x+10y+5=0 at (1,0).
A. 2/5 C.-2/5
B. 5/2 D.-5/2
42. Find the slope of the curve y=6(4 + 𝑥)½ at (0,12)
A.0.67 C.1.33

225
B. 1.5 D. 0.75
43. Find the acute angle that the curve y=1-3x³ cut the x-axis.
A.77° C. 79°
B.75° D. 120°
44. Find the angle that the line 2y-9x-18=0 makes with the x-axis
A.74.77° C .47.77°
B. 4.5° D .77.47°

226
PROBLEM SET 21: Maxima & Minima, Time
Rates
1.A function is given below, what x value maximizes y?
y²+y+x²-2x=5
A.2.23 C.5
B.−1 D.1
2. The number of newspaper copies distributed is given by C=50 t²-200t+10000, where t is in
years. Find the minimum number of copies distributed from 1995 to 2002.
A.9850 C.10200
B. 9800 D. 7500
3. Given the following profit-versus-production function for a certain commodity:
1.1 8
P=200000-x-( )
1+𝑥

3. Where P is the profit and x is the unit of production. Determine the maximum profit.
A. 190000 C. 250000
B. 200000 D. 550000
4. The cost C of a product is a function of the quantity x of the product is given by the relation:
C(x)=x²-4000x+50. Find the quantity for which the cost is minimum.
A.3000 C. 1000
B.2000 D.1500
5. If y=x to the 3rd power-3x. Find the maximum value of y.
A.0 C. 1
B.-1 D.2
6. Divide 120 into two parts so that the product of one and the square of the other is maximum.
Find the numbers.
A.60&60 C.70&50
B. 100&20 D.80&40
7. If the sum of two numbers is C,find the minimum where value of the sum of their squares
A.C²/2 C.C²/6

227
B.C²/4. D.C²/8
8. A certain travel agency offered a tour that will cost each person P1500.00 if not more than 150
persons will join, however the cost per person in excess of 150. How many persons will make
the profit a maximum?
A.75 C.225
B.150 D.250
9. Two cities A and B are 8km and 12km, respectively, north of a river which runs due east. City
B being 15km east of A. A pumping station is to be constructed (along the river) to supply water
for the two cities. Where should the station be located so that the amount of pipe is a minimum?
A.3 km east of A C.9 km east of A
B. 4 km east A D.6 km east of A
10. A boatman is at A, which is 4.5km from the nearest point B on a straight shore BM. He
wishes to reach, in minimum time, a point C situated on the shore 9km from B. How far from C
should he land if he can row at the rate of 6kph and walk at the rate of 7.5kph?
A. 1 km C. 5 km
B. 3 km D. 8 km
11. The shortest distance from the point (5,10) to the curve x²=12y is:
A.4.331 C. 5.127
B.3.474 D.6.445
12. A statue 3m high is standing on a base of 4m high. If an observer’s eye is 1.5m above the
ground, how far should he stand from the base in order that the angle subtended by the statue is a
maximum?
A.3.41m C.5.71m
B. 3.51m D.4.41m
13. An iron bar 20m long is bent to form a closed plane area. What is the largest area possible?
A.21.56 square meter C.28.56 square meter
B.25.58 square meter D.31.83 square meter
14. A normal window is in the shape of a rectangle surmounted by a semi-circle. What is the
ratio of the width of the rectangle to the total height so that it will yield a window admitting to
the most light for a given perimeter?
A.1 C.1/3
B.2/3 D.½

228
15. A rectangular field is to be fenced into four equal parts. What is the size of the largest field
that can be fenced this way with a fencing length of 1500 feet if the division is to parallel to one
side?
A.65,200 C.64,500
B. 65,2500 D.63,500
16. Three sides of a trapezoid are each 8 cm long. How long is the 4 th side, when the area of the
trapezoid has the greatest value?
A. 16 cm C. 12 cm
B. 15 km D. 10 cm
17. An open top rectangular tank with square bases is to have a volume of 10 cubic meters. The
material for it’s bottom cost P150.00 per square meter, and that for the sides is P60.00 per square
meter. The most economical height is:
A.2 meters C. 3 meters
B.2.5 meters D. 3.5 meters
18. A rectangular box having a square base and open at the top is to have a capacity of 16823 cc.
find the height of the box to use the least amount of material.
A.16.14 cm C. 18.41cm
B.32.28 cm D.28.74 cm
19. The altitude of a cylinder of maximum volume that can be inscribed in a right circular cone
of radius r and height h is:
A. h/3 C.3h/2
B. 2h/3 D.h/4
20. What is the least amount of tin sheet, in sq.inches, that can be made into a closed cylindrical
can having a volume of 108 cu.inches?
A.125 square meter C.150 square meter
B.137 square meter D.120 square meter
21. The volume of the closed cylindrical tank is 11.3 cubic meter. If the total surface area is a
minimum, what is its base radius, in m?
A.1.44 C.1.22 B.1.88 .1.66
22. A cylindrical steam boiler is to be constructed having a capacity of 1000 cu. m. The material
for the sides cost P2000.00 per square meter and for the ends P3000.00 per square meter. Find
the radius so that the cost is least.

229
A.3.52m C.4.73m
B. 4.12m D.5.25m
23. A box is to be constructed from a piece of zinc 20 inches square by cutting equal squares
from each corner and turning up the zinc to for the side. What is the volume of the largest box
that can be so constructed?
A. 599.95 cubic inches C. 592.59 cubic inches
B. 579.50 cubic inches D. 622.49 cubic inches
24.A load of 40 kN is to be raised by means of a lever weighing
250 N/m, which is supported at one end. If the load is place 1m from the support, how long
should the lever be so that the force required be a minimum?
A.13.43meters C. 18.56 meters
B.20.19 meters D. 17.89 meters
25. As x increases uniformly at the rate of 0.002 feet per second, at what rate is the expression
(1+x) to the 3rd power increasing when x becomes 8 feet?
A.430 cfs C. 0.486 cfs
B.0.300 cfs D.0.346 cfs
26. The distance a body travels is a function of time and is given by x(t)=16t+18t². Find its
velocity at t=3.
A. 64 C.54
B. 56 D.44
27. The distance traveled by a train is given by the equation x=3t²_2t+4 where x is the distance
in kilometers and t is the time in hours. Determine the velocity of the train after traveling 10km.
A.8.72 kph C.23.5 kph
B.18.72 kph D.75.3 kph
28. An object moves along a straight line such that, after t minutes, its distance from its starting
point is D=20t+5/(t+1)meters. At what speed, in m/minute will it be moving at the end of 4
minutes?
A.39.8 C.49.8 B.29.8 D.19.8
29. The speed of the traffic flowing past a certain downtown exit between the hours of 1;00 P.M.
and 6:00 P.M. is approximately v= t³-10.5t²+30t+20 miles per hour, where t=number of hours
past noon. What is the fastest speed of the traffic between 1:00 P.M. and 6:00 P.M. in mph?
A.50 C.32.5

230
B. 46 D.52
30. A van is 5 km due north of a bus at 2:00 p.m. if the van is traveling northward at the rate of
6o kph, how fast will the two be separating st 5:00 p.m.?
A. 85.36 kph C. 98.65 kph
B. 96.04 kph D. 102.54 kph
31. A car drives east from point A at 30 kph. Another car starting from B at the same time, drives
S 30°W toward A at 60 kph. B is 30 km away from A. How fast in kph is the distance between
the two cars changing after one hour?
A.76.94 kph C. 75.94 kph
B.78.94 kph D. 77.94 kph
32. A car starting at 12:00 noon travels west at a speed of 30 kph. Another car starting from the
same point at 2:00 pm travels north at 45 kph. Find how fast the two are separating at 4:00 p.m.?
A.55 C. 51
B.57 D.53
33. Two railroad tracts are perpendicular with each other. At 12:00 p.m. there is a train on each
track approaching the crossing at 50 kph, one being 100 km, the other 150 km away from the
crossing. How fast in kph is the distance between the two trains changing at 4:00 P.M.?
A. 68.08 C.69.08
B. 67.08 D.70.08
34. A balloon is rising vertically over a point A on the ground at the rate of 15 ft/sec. A point B
on the ground is level with and 30ft. from A. When the balloon is 40 ft. from A, at what rate is its
distancing from B changing?
A.10 ft/s C.12ft/s B.13ft/s D.15ft/s
35. A hemispherical dome has a diameter of 100m. A searchlight was placed at a point A located
at the circumference at the base. At the middle of the dome at B, a balloon was released
vertically at a velocity of 4 m/s. How fast is the shadow of the balloon move along the roof when
the balloon is 25m high?
A.6.4m/s C.7.2 m/s
B.4.6 m/s D.4 m/s
36.. If the distance y from the origin at time t is given by y=16t²+3000t+50000, Find the initial
velocity when t=0
A.0 C. 5300

231
B.3000 D. 50000
37. An airplane is flying horizontally at an altitude of 9000m. An observer on the ground noticed
that when the angle of elevation of the plane is 60°, the angle decreases at the rate of 0.15
radians/second. What is the velocity of the airplane?
A.600 m/s C. 1400 m/s
B.900 m/s D. 1800 m/s
38 .The surface are of the sphere (initially zero) increases uniformly at the rate of sq.cm per
second. Find the rate at which the radius is increasing after two seconds.
A.0.59 cm/sec C. 0.509 cm/sec
B.0.62 cm/sec D.0.52 cm/sec
39.The height of a right circular cylinder is 50 inches and decreases at the rate of 4 inches per
second, while the radius of the base is 20 inches and increases at the rate of one inch per second.
At what rate is the volume changing?
A.11310 cu.in/sec C.11130 cu.in/sec
B.1275 cu.in/sec D.1257 cu.in/sec
40. vessel 15 feet deep and 7.5 feet in diameter at the top. Water is rising at the rate of 2 feet per
minute when the water is 4 feet deep. What is the rate if inflow in cu.ft. per minute?
A.8.14 B.7.46 C.9.33 D.6.28
41. What is the curvature of the curve y²=16x at the point (4,8)?
A.-0.044 C.-0.066
B.=0.088 D.-0.033
42. Suppose that x years after founding in 1975, a certain employee association had a
membership of f(x)=100(2x³-45x²+264x), at what time between 1975 and 1989 was the
membership smallest?
A.1983 C. 1984
B.1985 D. 1986
43. A 3 meter long steel pipe has its upper and leaning against a vertical wall and lower end on a
level ground. The lower end moves away at a constant rate of 2cm/s. How fast is the upper end
moving down, in cm/s, when the lower end is 2 m from the wall?
A.1.81 C. 1.79
B.1.66 D.1.98
44. A particle moves according to the parametric equations:

232
y=2t² x=t³
Where x and y are displacements (in meters) in x and y direction, respectively and t is the time in
seconds. Determine the acceleration of the body after t=3 seconds.
A.12.85 m/s² C. 21.47 m/s²
B. 18.44 m/s² D. 5.21 m/s²
45.. Determine the shortest distance from point (4,2) to the parabola y²=8x.
A. 2.83 C. 2.41
B. 3.54 D. 6.32
46.. Water flows into a tank having the form of the frustum of a right circular cone. The tank is 4
m tall with upper radius of 1.5 m and the lower radius of 1 m, When the water in the tank is 1.2
m deep, the surface rises at the rate of 0.012 m/s, Calculate the discharge of water flowing into
the tank in m³/s.
A. 0.02 C. 0.08
B. 0.05 D.0.12
47.. The motion of a particle is defined by the parametric equation x=t³ and y=2t². Determine the
velocity when t=2
A.14.42 C. 12.74
B. 16.25 D. 18.63
48. The sum of two numbers is K, The product of one by the cube of the other is to be a
maximum. Determine one of the numbers.
A.3K/4 C. 3K/2
B.3k/8 D. 3K/7
49.Find the height of the largest right pyramid with a square base that can be inscribed in a
sphere of radius 10 cm.
A.12.2 cm C.13.3 cm
B.14.4 cm D.15.5 cm
50.Sand is pouring from a spout at the rate of 25cc/sec. It forms a cone whose heights is always
1/3 the radius of its base. At what rate in cm/sec is the height increasing when the cone is 50 cm
high?
A.0.000785 C.0.000447
B.0.000214 D.0.000354

233
ANSWER KEYS:
Problem Set 1: Conversion
1. D
2. A
3. C
4. A
5. B
6. A
7. D
8. B
9. B
10. B
11. C
12. C
13. B
14. B
15. D
16. D
17. C
18. D
19. C
20. C
21. B
22. C
23. A
24. D
25. C
26. D
27. D
28. C
29. C
30. B
31. C

Problem Set 2: Exponents and Radicals


1. C
2. D
3. C
4. D
5. B
6. D

234
7. C
8. D
9. C
10. A
11. A
12. C
13. B
14. D
15. A
16. C
17. C
18. A
19. C
20. A
21. A
22. C
23. D
24. B
25. D
26. A
27. A

Problem 3: Fundamentals in Algebra


1. C
2. A
3. D
4. C
5. D
6. B
7. D
8. D
9. B
10. D
11. C
12. A
13. B
14. D
15. A
16. C
17. D
18. C
19. B

235
20. B
21. C
22. C
23. D
24. B
25. A
26. B
27. C
28. C
29. D
30. D
31. D
32. C
33. D
34. B
35. C
36. C
37. C
38. A
39. C
40. C
41. A
42. B
43. D
44. A
45. B
46. A
47. C
48. B
49. A
50. A
Problem set 4:

1. B 26. C
2. C 27. A
3. D 28. D
4. A 29. D
5. C 30. C
6. C 31. B
7. C 32. B
8. A 33. D
9. B 34. C
10. A 35. B

236
11. C 36. A
12. A 37. D
13. C 38. A
14. B 39. B
15. C 40. A
16. C 41. D
17. C 42. C
18. C 43. C
19. A 44. C
20. C 45. A
21. B 46. C
22. A 47. B
23. B 48. B
24. D 49. A
25. A 50. D

Problem Set 5:
1. B 26. A
2. C 27. A
3. C 28. A
4. D 29. D
5. C 30. B
6. B 31. C
7. D 32. C
8. D 33. A
9. A 34. D
10. C 35. C
11. C 36. A
12. B 37. B
13. B 38. C
14. D 39. B
15. C 40. C
16. B 41. C
17. C 42. A
18. A 43. C
19. D 44. C
20. C 45. B
21. A 46. D
22. B 47. A
23. B 48. C
24. D 49. B
25. D 50. D

237
Problem Set 6:
1. B 26. D
2. A 27. A
3. A 28. C
4. B 29. D
5. A 30. C
6. D 31. A
7. D 32. C
8. C 33. A
9. D 34. C
10. A 35. C
11. A 36. C
12. B 37. B
13. D 38. C
14. C 39. C
15. D 40. A
16. B 41. C
17. D 42. D
18. A 43. B
19. C 44. C
20. C 45. A
21. D 46. D
22. C 47. C
23. B 48. C
24. C 49. B
25. D 50. C

Problem Set 7:
1. D
2. D
3. B
4. B
5. C
6. C
7. B
8. C
9. B
10. C
11. B
12. C
13. D
14. B
15. D
16. C
17. D

238
18. C
19. C
20. A
21. B
22. A
23. D
24. A
25. B
26. B
27. A
28. C
29. D
30. D
31. A
32. C
33. C
34. A
35. A
36. B
37. A
38. C
39. D
40. C
41. C
42. B
43. D
44. D
45. A
46. A
47. D
48. C
49. A
50. B

Problem Set 8:
1. A
2. B
3. B
4. B
5. A
6. B
7. C
8. A
9. A
10. D

239
11. D
12. A
13. D
14. C
15. D
16. C
17. B
18. C
19. D
20. A
21. B
22. C
23. A
24. D
25. A
26. C
27. C
28. C
29. B
30. B
31. C
32. A
33. C
34. A
35. B
36. A
37. C
38. A
39. B
40. A
41. D
42. A
43. D
44.
45.
46.
47. D
48. A
49. C
50. B

240
Problem Set 9:
1. C
2. C
3. B
4. C
5. A
6. B
7. B
8. A
9. D
10. D
11. D
12. C
13. B
14. C
15. B
16. B
17. D
18. B
19. A
20. B
21. B
22. B
23. D
24. B
25. A
26. C
27. D
28. C
29. D
30. C
31. B
32. B
33. A
34. D
35. C
36. B
37. C
38. C
39. B
40. A
41. B
42. A
43. A
44. C
45. A

241
46. C
47. C
48. C
49. C
50. A
51. A
52. A
53. A
54. D
55. B
56. A
57. C
58. B
59. A
60. B

Problem Set 10:


1. C 30. D
2. A 31. C
3. C 32. B
4. A 33. C
5. D 34. A
6. D 35. A
7. B 36. C
8. D 37. D
9. A 38. D
10. B 39. C
11. A 40. C
12. A 41. C
13. B 42. C
14. A 43. A
15. D 44. A
16. B 45. A
17. A 46. A
18. C 47. B

242
19. D 48. B
20. D 49. B
21. A 50. A
22. A
23. B
24. D
25. C
26. D
27. D
28. C
29. C

Problem Set 11:


1. B 30. C
2. C 31. A
3. A 32. C
4. B 33. A
5. C 34. A
6. B 35. C
7. C 36. D
8. B 37. C
9. C 38. C
10. D 39. A
11. C 40. D
12. C 41. D
13. A 42. A
14. C 43. B
15. C 44. A
16. B 45. A
17. C

243
18. B
19. C
20. D
21. B
22. A
23. B
24. B
25. D
26. D
27. C
28. B
29. A

Problem Set 12:


1. D 30. B
2. C 31. B
3. A 32. A
4. B 33. B
5. C 34. B
6. D 35. D
7. B 36. A
8. D 37. C
9. A 38. C
10. B 39. B
11. D 40. B
12. B 41. A
13. D 42. D
14. B 43. A
15. B 44. D
16. D 45. B

244
17. A 46. D
18. B 47. C
19. D 48. A
20. B 49. B
21. C 50. A
22. D
23. A
24. C
25. A
26. C
27. D
28. B
29. C

Problem Set 13:


1) D
2) C
3) A
4) B
5) C
6) D
7) B
8) D
9) A
10) B
11) D
12) B
13) D
14) B
15) B
16) D
17) A
18) B
19) D
20) B
21) C
22) D
23) A

245
24) C
25) A
26) C
27) D
28) B
29) C
30) B
31) B
32) A
33) B
34) B
35) D
36) A
37) C
38) C
39) B
40) B
41) A
42) D
43) A
44) D
45) B
46) D
47) C
48) A
49) B
50) A
Problem Set 14:
1) D
2) D
3) B
4) B
5) D
6) C
7) B
8) C
9) A
10) A
11) C
12) D
13) A
14) D
15) A
16) B
17) B

246
18) B
19) C
20) B
21) C
22) B
23) D
24) D
25) C
26) D
27) D
28) B
29) C
30) B
31) A
32) D
33) C
34) B
35) D
36) B
37) B
38) B
39) A
40) A
41) D
42) B
43) C
44) D
45) A
46) D
47) C
48) D
49) D
Problem Set 15:
1.A
2.B
3.B
4.B
5. D
6. B
7. D
8. A
9. D
10. A
11. A
12. B

247
13. B
14. C
15. B
16. B
17. A
18. A
19. C
20. A
21. B
22. C
23. B
24. A
25. B
26. C
27. A
28. D
29. A
30. C
31. A
32. D
33. D
34. B
35. B
36. D
37. D
38. B
39. D
40. D
41. A
42. D
43. D
44. B
45. B

Problem Set 16:

1. B
2. C
3. C
4. C
5. B
6. C

248
7. A
8. C
9. D
10. D
11. C
12. B
13. B
14. B
15. C
16. B
17. D
18. D
19. A
20. B
21. B
22. A
23. D
24. C
25. A
26. A
27. D
28. B
29. B
30. B
31. B
32. B
33. B
34. C

249
35. D
36. A
37. B
38. C
39. A
40. B
41. A
42. B
43. D
44. A
45. C
46. A
47. B
48. C
49. A
50. B
Problem Set 17:
1. D
2. A
3. D
4. A
5. D
6. D
7. D
8. C
9. D
10. D
11. A

250
12. B
13. A
14. C
15. B
16. D
17. B
18. A
19. C
20. A
21. B
22. B
23. C
24. A
25. A
26. D
27. A
28. D
29. B
30. A
31. A
32. C
33. D
34. D
35. A
36. D
37. B
38. B
39. A

251
40. C
41. D
42. A
43. C
44. D
45. B
46. D
47. A
48. C
49. C
50. C
51. C
52. B
53. D
54. C
55. B
56. C
57. A
58. C
59. D
60. B
Problem Set 18:
1. (4, 6)(-3, 8)
2. 12 & 3
3. 4√2
4. 108<xy<24
5. D
6. A
7. B
8. C
9. D

252
10. C
11. A
12. D
13. B
14. A
15. B
16. A
17. D
18. D
19. D
20. A
21. B
22. C
23. A
24. C
25. D
26. B
27. B
28. C
29. B
30. C
31. A
32. B
33. D
34. C
35. B
36. A
37. D
38. C
39. C
40. D
41. D
42. C
43. A
44. B
45. D
46. C
47. D
48. C
49. A
50. B
51. C
52. D

253
53. B
54. C
55. B
56. D
57. D
58. B
59. D
60. C
61. C
62. B
63. A
64. D
65. D
66. B
67. C
68. C
69. A
70. A
71. B
72. A
73. D
74. A
75. D
76. D
77. A
78. A
79. A
80. A

81. C
82. B
83. A
84. A
85. A
86. D
87. D
88. A
89. B
90. C
91. D
92. A
93. B

254
94. D
95. B
96. A
97. C
98. A
99. A
100. A
101. D
102. C
103. A
104. C
105. B
106. A
107. C
108. D
109. D
110. B

Problem Set 19:


1) C
2) B
3) B
4) D
5) A
6) B
7) C
8) A
9) D
10) A
11) C
12) A
13) D
14) B
15) A
16) C
17) C
18) C
19) A
20) C
21) A
22) D

255
23) A
24) D
25) A
26) D
27) B
28) C
29) D
30) C
31) B
32) A
33) C
34) B
35) D
36) A
37) B
38) A
39) B
40) D
41) A
42) B
43) C
44) C
45) D
46) C
47) A
48) C
49) B
50) C
51) B
52) A
53) B
54) A
55) C
Problem Set 20:
1-1. B
1-2. B
1-3. A
1-4. C
1-5. B
1-6. A
1-7. B
1-8. D

256
1-9. B
1-10. A
1-11. C
1-12. B
1-13. A
1-14. A
1-15. D
1-16. A
1-17. B
1-18. C
1-19. B
1-20. B
1-21. C
1-22. D
1-23. D
1-24. B
1-25. C
1-26. B
1-27. D
1-28. C
1-29. B
1-30. A
1-31. A
1-32. A
1-33. A
1-34. A
1-35. A
1-36. B
1-37. C
1-38. C
1-39. A
1-40. B
1-41. A
1-42. B
1-43. A
1-44. D
1-45. A

1-46. C
1-47. D

257
1-48. D
1-49. D

Problem Set 21:


2-1. D
2-2. B
2-3. B
2-4. B
2-5. D
2-6. D
2-7. A
2-8. C
2-9. D
2-10. B
2-11. C
2-12. C
2-13. D
2-14. A
2-15. B
2-16. A
2-17. B
2-18. B
2-19. A
2-20. A
2-21. C
2-22. C
2-23. C
2-24. D
2-25. A
2-26. A
2-27. D
2-28. B
2-29. B
2-30. D
2-31. C
2-32. B
2-33. B
2-34. A
2-35. B
2-36. D
2-37. C

258
2-38. D
2-39. D
2-40. A
2-41. D
2-42. C
2-43. B
2-44. A
2-45. A
2-46. A
2-47. C
2-48. D
2-49.

259
references

http://evlm.stuba.sk/~partner7/DBfiles/Modules/Differentiation/RulesDifferentiation.pdf
https://math.libretexts.org/Bookshelves/Calculus/Book%3A_Calculus_(OpenStax)/03%3A_Derivatives/3
.3%3A_Differentiation_Rules
https://www.math24.net/definition-derivative/
https://magoosh.com/hs/ap-calculus/2017/derivative-function-slope/
https://www.ugrad.math.ubc.ca/coursedoc/math100/notes/derivs/deriv5.html
https://www.mathwarehouse.com/algebra/linear_equation/slope-of-a-line.php
https://socratic.org/calculus/derivatives/slope-of-a-curve-at-a-point
https://math.stackexchange.com/questions/1405955/finding-the-slope-of-a-curve-with-a-given-point

https://www.varsitytutors.com/hotmath/hotmath_help/topics/rate-of-change
https://www.khanacademy.org/math/algebra/x2f8bb11595b61c86:functions/x2f8bb11595b61c86:aver
age-rate-of-change-word-problems/a/average-rate-of-change-review
https://byjus.com/instantaneous-rate-of-change
formula/#:~:text=The%20instantaneous%20rate%20of%20change,it%20is%20a%20curve%20slope.
https://www.sparknotes.com/math/calcab/applicationsofthederivative/section1/
https://www.emathzone.com/tutorials/calculus/examples-of-average-and-instantaneous-rate-of-
change.html

https://mathworld.wolfram.com/PolynomialCurve.html
https://mathcurve.com/courbes2d.gb/polynomiale/polynomiale.shtml
https://study.com/academy/lesson/what-is-a-straight-line-definition-examples.html
https://tutors.com/math-tutors/geometry-help/what-is-a-straight-line-definition-examples

https://www.intmath.com/applications-differentiation/1-tangent-normal.php
http://www.mathcentre.ac.uk/resources/uploaded/mc-ty-tannorm-2009-1.pdf
https://www.toppr.com/guides/maths/application-of-derivatives/tangents-and-normals/
https://my.georgebrown.ca/uploadedFiles/TLC/_documents/Solving%20for%20Tangent%20and%20Nor
mal%20Lines.pdf
https://www.cliffsnotes.com/study-guides/calculus/calculus/applications-of-the-derivative/tangent-
and-normal-lines

https://www.cliffsnotes.com/study-guides/calculus/calculus/applications-of-the-derivative/first-
derivative-test-for-local-
extrema#:~:text=If%20the%20derivative%20changes%20from,maximum%20at%20the%20critical%20po
int.&text=When%20this%20technique%20is%20used,Derivative%20Test%20for%20Local%20Extrema.
https://ltcconline.net/greenl/courses/115/applications/frsttst.htm
https://web.ma.utexas.edu/users/m408n/m408c/CurrentWeb/LM4-3-5.php
http://www.personal.kent.edu/~bosikiew/Math11012/first-derivative-test.pdf
http://archives.math.utk.edu/visual.calculus/3/graphing.5/index.html

http://www.evlm.stuba.sk/~velichova/xmathkluc/diff/theory/concav.xml

260
https://web.ma.utexas.edu/users/m408n/m408c/CurrentWeb/LM4-3-12.php

https://tutorial.math.lamar.edu/classes/calci/RelatedRates.aspx
Calculus 10th Edition
http://www.khanacademy.com.org/math/ap-calculus-ab/ab-diff-analytical-
applications-new/ab-5-6a/v/inflection-points
https://www.mathsisfun.com/calculus/inflection-points.html
https://www.varsitytutors.com/calculus_1-help/points/graphing-
functions/functions/points-of-inflection
https://www.math24.net/curve-sketching/#example1

https://byjus.com/maths/differential-calculus/
https://www.shelovesmath.com/calculus/differential-calculus/differentials-linear-
approximation/
http://georgeballinger.ca/math100/Differentials%20and%20Error%20Propagation.pdf
https://www.youtube.com/watch?v=R7t9Iv_lTc8
https://www.youtube.com/watch?v=4DuQh5oUsbQ
https://www.youtube.com/watch?v=ilFnSweYKzA
http://www.mathwords.com/a/approximation_by_differentials.htm#:~:text=Mathwords%3A%
20Approximation%20by%20Differentials&text=A%20method%20for%20approximating%20the,
to%20approximate%20the%20function%27s%20graph.
https://mathinsight.org/linear_approximation_differentials_refresher
https://www.math.utah.edu/lectures/math1210/15.5PostNotes.pdf
https://courses.lumenlearning.com/suny-openstax-calculus1/chapter/linear-approximations-
and-differentials/?fbclid=IwAR25Vw_C65kOexvrlZ0yKYDXrdEt2fwutPEfPBxbjNGfRbB31-
IZi8_tQC8

261

You might also like